Download as pdf or txt
Download as pdf or txt
You are on page 1of 195

INCOME TAX

VOLUME II

AY 2021-22

CA Raj K Agrawal
All India CA Rank Holder
© All rights reserved
CA. Raj K Agrawal

Every effort has been made to present this publication in the most authentic form without any
errors and omissions. In spite of this errors might have inadvertently crept in, or there may be a
difference of opinion on certain points. Any mistake, error or discrepancy noted may be kindly
brought to the notice of the Author, which shall be dealt with suitably. It is notified that the Author
does not guarantee the accuracy or completeness of any information published herein, and will not
be responsible for any damage or loss, of any kind, in any manner, arising out of use of this
information.

No Part of this publication may be reproduced or copied in any form or translated in any other
language without prior written permission of the Author.
About the Author

Raj K Agrawal, a Chartered Accountant devoted to the cause of CA students.


He Qualified Chartered Accountancy with all India 27th rank in CA Final
and all India 29th rank in CA PE-I. He has been consistent school and college
topper.

He is endowed with the passion of winning as evinced through demonstrated


excellence in Academics and Teaching Career. His distinguished teaching style
to face the challenges of tough professional exams has made him famous and
favourite amongst the students. He is promoter and educator of Study At
Home Pvt. Ltd. which is a recognized StartUp by Ministry of Commerce &
Industry, Government of India. He has authored several books for professional
courses published by renowned publishers of India.

His primary focus is on enhancing student’s knowledge theoretically and


practically as well as focused preparations to ensure success in the examinations
and to achieve professional expertise.

© CA. Raj K Agrawal


E-mail: support@studyathome.org
Website: www.studyathome.org
About the Book
I am pleased to commend to readers the Revised Edition of Taxation, which has been
revised and enlarged as per law applicable for AY 2021-22. It is a comprehensive
presentation of the subject matter in a lucid form understandable to the students. The
book also contains solved problems from key professional and academic
examinations. These will help students to maintain a meaningful focus on examination
requirements.

The book is intended to serve as a standard text for students pursuing their CA-
Intermediate, IPCC, CS-Executive, CMA-Inter, BBA, B.Com, B.Com (Hons) and many
more Professional Courses.

The following are the main features of the book:

• Simple Language
• Chart expressions
• Self-explanatory notes
• Illustrations
• Solved and Unsolved practical problems.

I hope this edition will endear itself to students and peers. I welcome comments and
suggestions for improving the utility of this book.

CA. Raj K Agrawal


Index
S. No. Topics Page No.
1 Capital Gain 1 - 58
2 Income From Other Sources 59 - 84
3 Clubbing of Income 85 - 105
4 Set off and Carry Forward of Losses 106- 120
5 Deductions 121- 136
6 Total Income 137 - 163
7 Return of Income 164 – 177
8 TDS & TCS 178 - 188
9 Alternate Minimum Tax (AMT) 189 - 190
1 CAPITAL GAINS
Basis of Charge [Sec. 45(1)]
Any profits or gains arising from the transfer of a capital asset effected in the PY, shall be
chargeable to income-tax under the head ‘Capital Gains’ unless such capital gain is exempt
u/s 54, 54B, 54D, 54EC, 54EE, 54F, 54G, 54GA or 54GB.
The essential conditions for taxing capital gains:
(a) There must be a capital asset;
(b) Such capital asset must have been transferred.

Capital Asset [Sec. 2(14)]


Capital asset means-
a) property of any kind held by the assessee, whether or not connected with his
business or profession,
b) any securities held by a foreign Institutional Investor which has invested in such
securities in accordance with the SEBI regulations.
But does not include:
1. Any stock in trade, consumable stores or raw materials held for the purposes of his
business or profession.
2. Movable Personal effects (i.e. assets held for personal use by the assessee or any
member of his family dependent on him). However, the following assets shall not
treated as personal effects though these assets are moveable and may be held for
personal use:
(a) Jewellery
(b) Archaeological collections
(c) Drawings
(d) Paintings
(e) Sculptures
(f) Any work of art
Note:
(i) The house property, in which the assessee lives, is used by him for personal
purposes, but it will not be a personal effect as it is an immovable property.
(ii) Jewellery includes:
(a) Ornaments made of gold, silver, platinum or any other precious metal or any
alloy containing one or more of such precious metals, whether or not containing
2 INCOME TAX

any precious or semi-precious stone and whether or not worked or sewn into
any wearing apparel;
(b) Precious or semi-precious stones, whether or not set in any furniture, utensil or
other article or worked or sewn into any wearing apparel.
3. Rural Agricultural land in India. In other words, it must not be an Urban agricultural
land.
Rural agricultural land means an agricultural land in India provided it is not situated in:
(a) Any area which is comprised within the jurisdiction of a municipality having a
population of 10,000 or more
(b) Any area within the distance, measured aerially:
 More than 2 kms from the local limits of any Municipality or Cantonment
Board having a population of more than 10,000 but not exceeding 1,00,000;
or
 More than 6 kms from the local limits of any Municipality or Cantonment
Board having a population of more than 1,00,000 but not exceeding
10,00,000; or
 More than 8 kms from the local limits of any Municipality or Cantonment
Board having a population of more than 10,00,000.
4. Specified Gold Bonds – 6½ GoldBonds 1977 or 7% Gold Bonds 1980 or National
Defense Gold Bond 1980 issued by the Central Government.
5. Special Bearer Bonds 1991 issued by the Central Government.
6. Gold Deposit Bonds issued under the Gold Deposit Scheme 1999 or Gold Deposit
certificates issued under the Gold Monetization Scheme, 2015.

Illustration 1: Ms. Vanya contends that sale of a work of art held by her is not eligible to
capital gains tax. Is she correct?
Solution: As per Section 2(14)(ii), the term “personal effect” excludes any work of art. As a
result, any work of art will be considered as a capital asset and sale of the same will attract
capital gains tax. Thus, the contention of Ms. Vanya is not correct.

Types of Capital Assets


(1) Short-term capital asset
(2) Long-term capital asset

Types of Capital Gains


(i) Short term Capital Gain - Gain arising on the transfer of short-term capital asset
(ii) Long-term Capital Gain - Gain arising on the transfer of long-term capital asset
CAPITAL GAINS 3

Period of Holding
It means the period for which the asset is held by the assessee. It starts from the date of
acquisition and ends on the day preceding date of transfer.

Date of Acquisition Day preceding the Date


Period of Holding
of Transfer

Capital Assets

Short Term 1. For: Long Term


(a) Listed Securities
(Equity, Preference
Debt)
(b) UTI Units
Upto 12 months More than 12 months
(c) Equity oriented
Mutual Fund
(d) Zero Coupon Bond

2. For:
Upto 24 months (a) Unlisted Shares More than 24 months
(b) Land & Building

Upto 36 months 3. Other Assets More than 36 months

Period of holding applicable to different assets


Nature of assets Period
Shares (equity or preference) or Debentures or bonds listed 12 months
Unlisted Shares (equity or preference) 24 months
Unlisted Bonds or Debentures 36 months
Units of UTI 12 months
Units of equity oriented MF 12 months
Units of Debt funds or money market mutual fund 36 months
Zero coupon Bonds 12 months
Land & Building 24 months
Any other Asset 36 months
4 INCOME TAX

Note:
 Period of holding in case of share of a company which went in liquidation shall be till
the date of liquidation and not till the date of payment by the company.
 There is a need to make the distinction between short-term and long-term capital gain
as short-term capital gain like any other incomes is taxable at normal rate of income-
tax, whereas long-term capital gain is taxed at 20%.

Capital asset must have been transferred


Transfer, in relation to capital assets, includes:
(i) Sale, Exchange or Relinquishment of the asset
(ii) Extinguishments of any rights therein; or
(iii) Compulsory acquisition under any law; or
(iv) Conversion of Capital asset into stock in trade; or
(v) Maturity or redemption of zero coupon bonds; or
(vi) Possession of any immovable property in part performance of a contract; or
(vii) Any transaction (whether by becoming member of or acquiring shares in a co-
operative society) which has the effect of transferring the enjoyment of any
immovable property; or
(viii) Insurance Claim due to damage or destruction of property.

Note:
1. Sale: Sale means voluntary conveyance of ‘property’ in the goods by one person to
another for consideration in ‘money’
2. Exchange: Exchange means voluntary conveyance of Property in the goods by one
person to another for consideration in kind.
Transfer of Gold
Mr. A Mr. B
Transfer of shares

The sale consideration shall be the FMV of the thing received in kind.
3. Extinguishment: To extinguish means to put a total end to something. It indicates a
complete wipe out, destruction or annihilation of contracts, rights, title, interest or a
debt or other obligation whether the effect is produced by the act of God or by
operation of Law or by the act of party.
For example, if a company is liquidated, the right of share of shareholder in the shares
of the company is extinguishing on the date of liquidation.
CAPITAL GAINS 5

Transactions not regarded as transfer:


1. Any distribution of capital assets on the total or partial partition of HUF.
2. Any transfer of a capital assets under a gift or will or an irrevocable trust.
Exception: The aforesaid rule is not applicable, if the following conditions are satisfied:
(a) Taxpayer is an employee;
(b) He has been allotted (directly or indirectly) shares/ debentures by the employer
company under Employees’ Stock Option Plan; and
(c) The aforesaid shares/ debentures are gifted by the concerned-employee to any
person.

Example:
Exercise Period of
Vesting Period
Period Holding
1st Year 2nd Year 3rd Year 4th Year 5th Year 6th Year

Option Acquired Date of Date of Sold


100 shares @ 20 Date of Exercise Allotment shares @
MP = 100 Vesting MP = 175 MP = 200 250 or
MP = 150 gifted

Perquisite = MV on the date of Exercise – Amount recovered from employee


= 100 Shares x 175 — 100 Shares x 20
= 17,500 — 2,000
= 15,500
But taxable in the year of allotment (i.e. 5th year).
If the same shares have been sold or gifted by the employee subsequently, the cost of
acquisition of the shares shall be the market value of the shares which was taken into
consideration for the purpose of perquisite value under the head salary. In case of gift, sale
consideration would be FMV on the date of gift.
Capital Gain Short Term Long Term
Sale Consideration (250 × 100) 25,000 25,000
Less: COA / Indexed COA 17,500 18,227
 17,500  301 
 
 289 
STCG / LTCG 7,500 6,773
6 INCOME TAX

3. Any transfer of a capital asset by a company to its 100% subsidiary company


provided the subsidiary company is an Indian company.
4. Any transfer of a capital asset by a 100% subsidiary company to its holding
company, if the holding company is an Indian company.
5. Any transfer in a scheme of amalgamation of a capital asset by the amalgamating
company to the amalgamated company, if the amalgamated company is an Indian
company.
6. Any transfer, in a demerger, of capital assets by the demerged company to the resulting
company, if the resulting company is an Indian company.

7. Any transfer, in a scheme of amalgamation of a banking company with a banking


institution (sanctioned and brought into force by the Central Government) of a capital
asset by the banking company to the banking institution.

8. Any transfer of a capital asset, being any work of art, archaeological, scientific or
art collection, book, manuscript, drawing, painting, photograph or print, to the
Government or a university or the National Museum, National Art Gallery, National
Archives or any such other public museum or institution, as may be notified by the
Central Government in the Official Gazette to be of national importance.

9. Any transfer by way of conversion of bonds or debentures, into shares of that


company.
When any debentures or part thereof of a company are converted into shares of that
company, the transaction is not considered as a transfer and hence no capital gain is
chargeable. However, when these shares are, thereafter, actually transferred, capital
gain shall arise and be chargeable in the previous year in which the shares are
transferred. The cost of acquisition of the shares shall be that part of the cost of
debenture in relation to which shares were acquired by the assessee.

Indexation
Conversion
Bond/ Debenture Equity Share Sold
No Capital Gain Capital Gain Arise
Holding Period

Illustration 2: Ramesh acquired 200 listed debentures of 100 each on 15-5-2013. 50%
value of the debentures was converted into 4 listed equity shares of the value of 10 each
on 20-8-2019. Ramesh therefore, received 800 shares of face value of 10 each and left
with 200 debentures of 50 each. The shares were sold on 15-6-2020 @ 100 per share
through recognized stock exchange and Ramesh paid 80 as securities transaction tax.
Compute the capital gain chargeable for the assessment year 2021-22.
CAPITAL GAINS 7

Solution:

Sale consideration of 800 shares 80,000


Less: Cost of acquisition (800 × 12.50) 10,000
Short-term capital gain 70,000
Note: The date of acquisition of the shares shall be the date on which such shares are
allotted to the assessee. In this case, the shares were allotted on 20-8-2019 and were sold
on 15-6-2020. As these were held for a period of not exceeding 12 months, they have been
treated as short-term capital assets. Securities transaction tax of 80 is not allowed as
deduction.

10. Any Transfer by way of conversion of preference shares into equity shares of that
Company.

Indexation
Conversion
Preference Share Equity Share Sold
No Capital Gain Capital Gain Arise

Holding Period

11. Allotment of shares in the resulting company to the shareholder’s of demerged


company.
Indexation

Share of Demerged Share of Resulting


Sold
Company No Capital Gain Company Capital Gain Arise

Holding Period

12. Allotment of shares in amalgamated company in lieu of shares held in


amalgamating company.
Indexation

Share of Amalgamating Share of Amalgamated


Capital Gain Arise Sold
Company No Capital Gain Company

Holding Period
8 INCOME TAX

13. Any transfer of a capital asset or intangible asset by a firm to a company as a


result of succession of the firm by a company in the business carried on by the firm
provided the following conditions are satisfied:
(a) All the assets and liabilities of the firm, relating to the business immediately before
the succession become the assets and liabilities of the company;
(b) All the partners of the firm immediately before the succession become the
shareholders of the company in the same proportion in which their capital
accounts stood in the books of the firm on the date of the succession;
(c) The partners of the firm do not receive any consideration or benefit, directly or
indirectly, in any form or manner, other than by way of allotment of shares in the
company;
(d) The aggregate of the shareholding in the company of the partners of the firm is not
less than 50% of the total voting power in the company and their shareholding
continues to be as such for a period of 5 years from the date of the succession.

14. Where a sole proprietary concern is succeeded by a company in the business


carried on by it as a result of which the sole proprietary concern transfers any
capital asset or intangible asset to the company provided the following
conditions are satisfied:
(a) All the assets and liabilities of the sole proprietary concern relating to the business
immediately before the succession become the assets and liabilities of the
company;
(b) The shareholding of the sole proprietor in the company is not less than 50% of the
total voting power in the company and his shareholding continues to remain as
such for a period of 5 years from the date of the succession; and
(c) The sole proprietor does not receive any consideration or benefit, directly or
indirectly, in any form or manner, other than by way of allotment of shares in the
company.
In case of conversion of sole proprietorship or firm into a company which is not
regarded as transfer, the cost of acquisition of asset in the hands of the company would
be the same as that in the hand of the sole proprietary concern or the firm, as the case
may be.

15. Any transfer under the Security Lending Scheme, 1997 for lending of any securities
under an agreement or arrangement, which the assessee has entered into with the
borrower of such securities and which is subject to the guidelines issued by the SEBI or
RBI, in this regard.

16. Any transfer of a capital asset in a transaction under reverse mortgage scheme shall
not be regarded as transfer and therefore shall not attract capital gain tax.
CAPITAL GAINS 9

Analysis
 Borrower is generally a senior citizen.
 He owns a house property but does not have a regular income.
 He mortgages his property with a schedule bank or a housing finance company.
 The lender in return pays periodic installments or lump sum to the borrower
during his life time.
 The borrower can continue to stay in the property during his life time and as well
continue to receive regular income from lender.
 The borrower does not pay the principal as well as interest to the lender during his
lifetime.
 The Borrower shall give possession of house to Bank, which shall not be regarded
as transfer.
 The lender will recover the loan along with the accumulated interest by selling the
house after the death of the borrower. This sale of mortgaged property by bank
would attract capital gain tax liability.
 However before disposal, an option will be given to the legal heirs to repay the loan
along with interest and to get the mortgaged property released.
 Any excess amount recovered on disposal will be remitted back to the legal heirs of
the borrower.

17. Conversion of small private companies and unlisted public companies into LLPs
to be exempt from capital gains tax subject to fulfillment of certain conditions:
 The total sales, turnover or gross receipts in business of the company should not
exceed 60 lakh in any of the three preceding previous years;
 The value of total assets in the books of accounts of the company in any of the
3 previous years preceding in the previous year in which conversion takes
place, should not exceed 5 Crore.
 The shareholders of the company become partners of the LLP in the same
proportion as their shareholding in the company;
 No consideration other than share in profit and capital contribution in the LLP
arises to the shareholders;
 The erstwhile shareholders of the company continue to be entitled to receive at
least 50% of the profits of the LLP for a period of 5 years from the date of
conversion;
 All assets and liabilities of the company become the assets and liabilities of the LLP;
and
 No amount is paid, either directly or indirectly, to any partner out of the
accumulated profit of the company for a period of 3 years from the date of
conversion.
10 INCOME TAX

However, if subsequent to the transfer, any of the above conditions are not complied
with, the capital gains not charged under section 45 would be deemed to be chargeable
to tax in the previous year in which the conditions are not complied with, in the hands
of the LLP or the shareholder of the predecessor company, as the case may be.
Further, the successor LLP would be allowed to carry forward and set-off the business
loss and unabsorbed depreciation and VRS Expenditure not written off the predecessor
company.

18. Any transfer, made outside India, of a capital asset being rupee denominated bond of an
Indian Company issued outside India, by a non-resident to another non-resident.

19. Transfer of a Government Security carrying a periodic payment of interest made


outside India from one non-resident to another non-resident.

20. Transfer of Sovereign Gold Bond issued by RBI under sovereign Gold Bond Scheme,
2015 by way of redemption by an assessee being as individual.

21. Any transfer by a unit holder of a capital asset being a unit or units held by him in the
consolidating plan of a mutual fund scheme, made in consideration of the allotment to
him of a capital asset, being a unit or units in the consolidated plan of that scheme of
the mutual fund.

Illustration 3: Ashok Pvt. Ltd. has converted into a LLP on 1.4.2020. The following are the
particulars of Ashok Pvt. Ltd. as on 31.3.2020:
(1) Unabsorbed depreciation 13.32 lakh
Business loss 27.05 lakh
(2) WDV of assets
Plant & Machinery (15%) 60 lakh
Building (10%) 90 lakh
Furniture (10%) 10 lakh
(3) Cost of land (acquired in the year 2001) 50 lakh
(4) VRS expenditure incurred by the company during the previous year 2018-19 is 50
lakh. The company has been allowed deduction of 10 lakh each for the P.Y. 2018-19
and P.Y. 2019-20 under section 35DDA.
Assuming that the conversion fulfils all the conditions specified explain the tax treatment of
the above in the hands of the LLP.
Solution:
(1) As per Section 72A(6A), the LLP would be able to carry forward and set-off the
unabsorbed depreciation and business loss of Ashok Pvt. Ltd. as on 31.3.2020.
However, if in any subsequent year, say previous year 2021-22, the LLP fails to fulfill
CAPITAL GAINS 11

any of the conditions mentioned in Section 47(xiiib), the set off of loss or depreciation
so made in the previous year 2020-21 would be deemed to be the income chargeable to
tax of P.Y. 2021-22.
(2) The aggregate depreciation for the P.Y. 2020-21 would be
Plant & Machinery 9 lakh (15% of 60 lakh)
Building 9 lakh (10% of 90 lakh)
Furniture 1 lakh (10% of 10 lakh)
In this case, since the conversion took place on 1.4.2020, the entire depreciation is
allowable in the hands of the LLP. Had the conversion taken place on any other date, say
1.7.2020, the depreciation shall be apportioned between the company and the LLP in
proportion to the number of days the assets were used by them. In such a case, the
depreciation allowable in the hands of Ashok Pvt. Ltd. and the LLP would be calculated as
given below:
In the hands of A Ltd. (for 91 days)
91
Plant and machinery ×9,00,000 2,24,384
365
91
Building ×9,00,000 2,24,384
365
91
Furniture ×1,00,000 24,932
365
In the hands of the LLP (for 274 days)
274
Plant and machinery ×9,00,000 6,75,616
365
274
Building ×9,00,000 6,75,616
365
274
Furniture ×1,00,000 75,068
365
(3) The cost of acquisition of land in the hands of the LLP would be the cost for which
Ashok Pvt. Ltd. acquired it, i.e., 50 lakh.
(4) The LLP would be eligible for deduction of 10 lakh each for the P.Y. 2020-21, P.Y.
2021-22 and P.Y. 2022-23 under section 35DDA.

Illustration 4: Mr. Abhik’s father, who is a senior citizen, had pledged his residential house
to a bank under a notified reverse mortgage scheme. He was getting loan from bank in
monthly installments. Mr. Abhik’s father did not repay the loan on maturity and gave
possession of the house to the bank to discharge his loan. How will the treatment of long-
term capital gain be made on such reverse mortgage transaction?
Solution: Section 47(xvi) provides that any transfer of a capital asset in a transaction of
reverse mortgage under a scheme made and notified by the Central Government shall not
be considered as a transfer for the purpose of capital gain.
12 INCOME TAX

Accordingly, the transaction made by Mr. Abhik’s father will not be regarded as a transfer.
Therefore, no capital gain will be charged on such transaction.
Further, Section 10(43) provides that the amount received by the senior citizen as a loan,
either in lump sum or in installment, in a transaction of reverse mortgage would be exempt
from income-tax.
However, capital gains tax liability would be attracted at the stage of alienation of the
mortgaged property by the bank for the purposes of recovering the loan.

Benefit of indexation is not available in the case of following Long-term Capital Asset
Capital Assets Who is the
transferor
Bonds or debentures (other than capital indexed bonds issued by the
Any person
Government & Sovereign Gold Bond issued by RBI)
Shares in or debentures of an Indian company acquired by utilizing
Non-Resident
convertible foreign exchange as mentioned in first proviso to Sec. 48
Depreciable asset (other than an asset used by a power generating
Any person
unit eligible for depreciation on straight line basis)
Undertaking/division transferred by way of slump sale as covered by
Any person
Sec. 50B

Capital gains exempt from tax under Section 10


1. Capital Gain on Transfer of US64 [Sec. 10(33)]
Any income arising from the transfer of a long term or short term capital asset being a
unit of US64 is not chargeable to tax where the transfer of such assets takes place on or
after April 1, 2002.

2. Capital Gain on Compulsory Acquisition of Urban Agriculture Land [Sec. 10(37)]


Any transfer of Urban Agricultural land on compulsory acquisition is exempt if the
following conditions are satisfied:
(i) The assessee is an individual or a HUF.
(ii) He or it owns an agriculture land situated in urban area.
(iii) There is transfer of the agriculture land by way of compulsory acquisition or the
consideration for transfer is approved or determined by the Central Government
(not by a State Government) or RBI.
(iv) The agriculture land was used by the assessee (and/ or his parents if the land was
owned by an individual) for agricultural purposes during 2 years immediately
prior to the date of transfer.
(v) The asset may be long-term capital asset or short-term capital asset.
CAPITAL GAINS 13

3. Capital Gain on transfer of immovable property in Amaravati [Sec 10(37A)]


Sec. 10(37A) inserted with retrospective effect from 1.4.2015 to provide that transfer
of capital asset being immovable property in Amaravati under land pooling scheme
shall be exempt.

Full Value of consideration [Sec. 48]


1. Full value of consideration is the consideration received or receivable by the transferor
in lieu of assets, which he has transferred. Asset acquired may be received in cash or in
kind. If it is received in kind, then fair market value of assets acquired is taken as full
value of consideration.

2. Adequacy or inadequacy of consideration is not a relevant factor for the purpose of


determining of full value of consideration (see, however, the provisions of Section 50C).

3. It makes no difference whether (or not) “full value of consideration” is received during
the previous year. Even if the full value of consideration is received in instalments in
different years, the entire value of consideration has to be taken into account for
computing the capital gains, which become chargeable in the year of transfer.

Expenditure on Transfer
“Expenditure incurred wholly and exclusively in connection with such transfer” means
expenditure incurred which was necessary to effect the transfer. Examples of such expenses
are brokerage or commission paid for securing a purchaser, cost of stamp, registration fees
borne by the vendor, travelling expenses incurred in connection with transfer, litigation
expenditure for claiming enhancement of compensation awarded in the case of compulsory
acquisition of assets.

Cost of Acquisition
Cost of acquisition of an asset is the value for which it is acquired by the assessee. It means
that whatever cost is incurred for getting an asset plus all expenses incurred to acquire it is
the cost of acquisition. Interest paid on money borrowed for the purchase of a capital asset
would constitute part of the cost of acquisition, provided such interest has not been
deducted under any other provision. Similarly, if any loan has been taken to purchase a plot,
the interest paid or payable on such loan, till the building is constructed on the plot, shall be
included in the cost of land.
However, in the following cases the above meaning of cost of acquisition does not hold
good:
14 INCOME TAX

1. The cost to the previous owner is deemed to be the cost of acquisition to the assessee in
cases where capital asset became the property of assessee under any mode of transfer
described below. [Sec. 49(1)]
(i) Acquisition of property on distribution of assets on the total or partial partition of
a HUF;
(ii) Acquisition of property under a gift or will;
(iii) In case of conversion of sole proprietorship or firm into a company which is not
regarded as a transfer.

2. Where the previous owner has acquired the property in the aforesaid manner, the
previous owner of the property means the last previous owner who acquired the
property by means other than those discussed above. Cost of any improvement of the
asset borne by the previous owner, or the assessee, will be added to such cost.

3. In order to find out whether the capital asset is short-term or long-term in the above
cases, the period of holding of the previous owner shall be taken into consideration.

4. Indexation benefit in respect of the gifted asset shall apply from the year in which
the asset was first acquired by the previous owner.
CIT v. Manjula J. Shah
In case the capital asset becomes the property of the assessee in the circumstances
mentioned in Section 49(1), the assessee must be treated to have held the asset from
the year the asset was first held by the previous owner and accordingly the CII for the
year the asset was first held by the previous owner would be considered for
determining the indexed cost of acquisition.

5. On June 1, 2016, X took a loan of 5 lakh by mortgaging his house property. X could not
repay the loan during his lifetime and after his death on July 2, 2019, the property (with
mortgage is transferred to Mrs. X. Mrs. X transfers the property on May 2, 2020 and
before transfer a sum of 7.2 lakh is paid to clear of the mortgage. 7.2 lakh will be
deductible as part of cost of acquisition of the property while calculating capital gains
in the hands of Mrs. X. If however, loan is taken by Mrs. X, then repayment of loan will
not be deductible as part of cost of acquisition of the property while calculating capital
gains in the hands of Mrs. X.

6. Cost of Acquisition of capital asset acquired by way of conversion of stock-in-trade into


capital asset will be the FMV of asset on the date of such conversion. Period of Holing &
Indexation benefit would arise from the date of such conversion.
CAPITAL GAINS 15

7. Where the capital gain arises from the transfer of an asset declared under the Income
Declaration Scheme, 2016 and the tax, surcharge and penalty have been paid in
accordance with the provisions of the scheme on the fair market value of the asset as on
the date of commencement of the scheme, the cost of acquisition of the asset shall be
deemed to be the fair market value of the asset which has been taken into account for
the purpose of said scheme.

Cost of Acquisition being the Fair Market Value as on April 1, 2001


In the following cases, the assessee may take at his option, either actual cost or the fair
market value of the asset, as on April 1, 2001 as cost of acquisition:
1. Where the capital asset became the property of the assessee before April 1, 2001.
2. Where the capital asset became the property of the assessee by any mode referred to in
section 49(1) and the capital asset became the property of the previous owner before
April 1, 2001.
3. The option is not available in the case of depreciable assets.
4. When option is available, the cost of the asset or fair market value as on April 1, 2001,
whichever is higher, is taken as the cost of acquisition.
5. In case of land or building or both, the stamp duty value shall be taken instead of fair
market value. However, where circle rates have not been notified by the states then the
fair market value shall be used.
6. The option is not available in respect of transfer of a capital asset being:
 goodwill of a business or profession
 right to manufacture, produce or process any article or thing; right to carry on
business;
 trade mark/ brand name associated with a business;
 tenancy right;
 route permits; or
 loom hours
whether self-generated or otherwise.

Cost of acquisition and improvement in case of Self-Generated Assets [Sec. 55]


Asset Cost of Acquisition Cost of Option of Expenses
Self- Purchased Improvement FMV of on Transfer
Generated 2001
Value
Goodwill of
Businessor Nil Actual Nil No Actual
Profession
Right to manu-
Nil Actual Nil No Actual
facture, produce
16 INCOME TAX

or Right to carry
on business
Trade Mark/
Nil Actual Actual No Actual
Brand Name
Tenancy Right Nil Actual Actual No Actual
Route Permit Nil Actual Actual No Actual
Loom Hours Nil Actual Actual No Actual

Advance Money Received & Retained [Sec. 51]


Any advance or other money received and retained by the assessee shall be deducted
from the COA or the WDV or the FMV, as the case may be. Amount forfeited by previous
owner shall not be deduced. Provided that where any sum of money, received as an advance
or otherwise in the course of negotiations for transfer of a capital asset, on or after 1-4-14
then, such sum shall not be deducted from the cost for which the asset was acquired or the
written down value or the fair market value, as the case may be, in computing the cost of
acquisition instead shall be income chargeable to tax under head “Income from other
Sources.
But if any advance money is forfeited against any rural agricultural land thenit cannot be
treated as income u/s 56(2)(ix) because rural agricultural land is not a capital asset.

Fair market value to be the full value of consideration where consideration cannot be
determined [Sec. 50D]
Where consideration cannot be ascertained or determined, the fair market value on the date
of transfer of the asset, shall be taken to be the full value of the consideration for computing
the capital gain.

Cost Inflation Index


Cost inflation index as notified by the Central Government for different previous years is
given below.
Previous Year Cost Inflation Index
2001-02 100
2002-03 105
2003-04 109
2004-05 113
2005-06 117
2006-07 122
2007-08 129
2008-09 137
2009-10 148
2010-11 167
CAPITAL GAINS 17

2011-12 184
2012-13 200
2013-14 220
2014-15 240
2015-16 254
2016-17 264
2017-18 272
2018-19 280
2019-20 289
2020-21 301

Capital gain on transfer of equity share allotted at time of corporation or


demutualization of a recognized stock exchange.
The following special provisions are applicable if a stock exchange is converted into
company and member of the old stock exchange are allotted:
(a) Equity shares, and
(b) Right to trade in the newly formed stock exchange.
Capital asset Cost of Acquisition Period of Holding
Allotment of equity Cost of acquisition of From the date of holding of
share in newly membership ticket in the membership ticket in the old
formed stock old stock exchange. exchange.
exchange.
Right to trade in the Nil From the date of holding of
new exchange. membership ticket in the old
exchange.
Note: The benefit of indexation will be from the year in which the asset was first held by the
assessee.
Period of Holding (LT/ST)
Indexation

Share in Stock
Sold
Exchange
Membership in
AOP
Right to Trade Sold

Indexation
(although not required
because COA is Nil)

Period of Holding (LT/ST)


18 INCOME TAX

Right Shares
Shares Particulars Cost of Acquisition Period of Holding
1. Original On the basis of which Amount actually From the date of
Shares the assessee entitled paid for acquiring allotment.
to right shares. shares.
2. Right Which is renounced Nil From the date of offer
entitlement by the assessee in from the company to the
favour of a third date of renouncement.
person.
3. Right By exercising his Actual amount paid From the date of
Shares rights entitlement. for the same. allotment of right shares.
4. Right Who purchase the Price to renouncer + From the date of
Shares right entitlement amount paid to allotment of right shares.
from the renouncer. company.

Bonus Shares
Shares Allotment date Cost of Acquisition
1. Bonus Shares Before April 1, 2001 FMV as on April 1, 2001
2. Bonus Shares On or After April 1, 2001 Nil

Capital gains in the case of non-resident [First proviso to Sec. 48]


Conditions: The following conditions should be satisfied:
1. The assessee is a non-resident.
2. He acquires shares in (or debentures of) an Indian company by utilizing foreign
currency.
3. The asset may be short-term or long-term.
Rule of Computation:
1. The benefit of indexation shall not be available.
2. Method of computing capital gain.

Conversion rate
Conversion
of which date is Computation
rate
applicable
Sale Average Date of transfer Sale Consideration (Indian )/ Avg.
Consideration exchange rate exchange rate at date of transfer.
Less: COA Average Date of acquisition Cost of acquisition (Indian )/
exchange rate Average exchange rate at date of
acquisition.
Less: Transfer Average Date of transfer Expenditure on sale (Indian )/ Avg.
expenses exchange rate exchange rate at date of transfer.
CAPITAL GAINS 19

Capital gain Buying rate Date of transfer Capital Gain (Foreign Curr.) X
Buying rate at Date of transfer.

Capital gain on conversion of capital asset into stock-in-trade [Section 45(2)]


W.e.f. the assessment year 1985-86 the conversion of capital asset into stock-in-trade is
treated as a ‘transfer’ within the meaning of Section 2(47). However, Section 45(2) provides
that although such a conversion will be a transfer of the previous year in which the asset is
so converted, but the capital gain will arise in the previous year in which such converted
asset is sold or otherwise transferred. Indexation of cost of acquisition and improvement, if
required, will be done till the previous year in which such conversion took place. Further,
the fair market value of the asset, as on the date of such conversion, shall be deemed to be
full value of the consideration of the asset. The sale price minus market value as on the date
of conversion shall be treated as business income and taxed under the head ‘Profit and
gains of business and profession’.

Illustration 5: Rana invested 80,000 on the purchase of gold ornaments on 4-1-1994. He


holds the gold ornaments as investments. Market Value as on 1-4-2001 is 1,60,000. On 12-
1-2019 he started a business of dealing in jewellery and converts his holding into his stock-
in-trade. The market value of the gold ornaments as on the date of conversion was
5,00,000 and therefore, Rana credited his capital account by 5,00,000 and debited his
stock account by 5,00,000. The gold ornaments are now reflected in the business of Rana
as stock-in-trade. These gold ornaments were sold in the previous year 2020-21 for a sum
of 6,00,000.
(a) Compute the capital gain and business income.
(b) What would be the answer if the gold ornaments are held by the assessee till 31-3-
2021?

Solution:
(a) The conversion of capital asset into stock-in-trade is treated as a transfer within the
meaning of Section 2(47). In this case, conversion took place on 12-1-2019 i.e. in the
previous year 2018-19. Therefore, it will be treated as transfer of the previous year
2018-19. But the capital gain will only arise in the previous year in which such asset is
sold i.e. previous year 2020-21.

Capital Gain of Assessment Year 2021-22

Full value of consideration (Market value as on the date of conversion) 5,00,000


Less: Indexed cost of acquisition – 1,60,000 ×280/100 4,48,000
Long-term capital gain 52,000
20 INCOME TAX

Note:
1. Cll has been taken as 280 i.e. the index for the previous year 2018-19 in which the asset
is converted as it will be treated as the year of transfer.
2. As the asset was acquired by the assessee on 4-1-1994 i.e. prior to 1-4-2001. CII has
been taken as of PY 2001-02 i.e. 100.

Business Income AY 2020-21

Sale price 6,00,000


Less: Market value as on the date of conversion 5,00,000
Business income 1,00,000

(b) If the gold ornaments are still held: There will neither be business income nor capital
gain because the asset has, no doubt, been converted into stock-in-trade, but it has not
yet been sold or otherwise transferred.

Capital gain in case of Joint Development Agreement


Capital gain is chargeable to tax in the year in which transfer takes place except in certain
cases. ‘Transfer’ includes any arrangement where rights are handed over in execution of
part performance of contract. In such scenario, execution of JDA between owner of
immovable property and the developer triggers the capital gains tax liability in the hands of
the owner in the year in which the agreement is executed for developing the projects. There
were many issues on the interpretation of this section, to clarify on this issue and to reduce
the hardship of paying tax in year of transfer:

 Individual/HUF who enters into JDA, capital gains shall be chargeable to in year in
which certificate of completion for the project is issued by the competent
authority.
 Stamp duty value of land owner’s share on the date of issuing of certificate of
completion plus any monetary consideration received, if any, be full value of
consideration.
 Tax @ 10% to be deductible on payments made in above transaction.
 Assessee who Transfers the asset before the issue of completion certificate are
excluded from this section and the capital gains shall be deemed to be the income
of the previous year in which such transfer took place and the provisions of the
Act, other than the provisions of this sub-section, shall apply for the determination
of the full value of consideration received or accruing as a result of such transfer.
 This section is applicable in case of Individual and HUF only.
CAPITAL GAINS 21

Capital gain on transfer of capital asset by a partner/ member of AOP/ BOI as capital
contribution [Sec. 45(3)]
The profits or gains arising from the transfer of capital asset held by a person, to a firm or
AOP or BOI in which he becomes a partner or member, by way of capital contribution or
otherwise, shall be chargeable to tax as his income of the previous year, in which such a
transfer takes place and for the purposes of computation of capital gain, in the hands of the
partner/member, the amount recorded in the books of account of the firm, AOP or BOI for
such capital asset shall be deemed to be the full value of the consideration. It may be
observed that the sale consideration in this case shall be the amount as recorded in the
books of the firm/AOP, etc. and not the market value of the asset as on the date of the
transfer.

Illustration 6: Rahul acquired a property by way of gift from his father in the previous year
2006-07 when its FMV was 3,00,000. The father had acquired the property in the previous
year 2003-04 for 2,00,000. This property was introduced as capital contribution to a
partnership firm in which Rahul became a partner on 5-6-2020. The market value of the
asset as on 5-6-2020 was 15,00,000, but it was recorded in the books of account of the
firm at 10,00,000. Is there any capital gain chargeable in the hands of Rahul? If yes,
compute the amount.
Solution:

Full value of consideration 10,00,000


Less: Indexed cost of acquisition – 2,00,000×301/105 5,73,333
Long- term capital gain 4,26,667

Note:
1. Full value of consideration is taken as the value at which it is recorded in the books of
account of the firm.
2. Cost of acquisition is taken as cost to the previous owner but indexation has been done
from the date it was first held by the assessee.
3. Indexation benefit in respect of the gifted asset shall apply from the year in which asset
was first acquired by the previous owner referring the case of CIT Vs Manjula J. Shah.

Illustration 7: Ajay has 2 motor cars which are used by him exclusively for his personal
purposes. The cost of the cars was 1,50,000 and 1,80,000. The first car was transferred
by him on 5-1-2021 to a firm in which he is a partner as his capital contribution. The market
value of the car as on 5-1-2021 is 1,00,000, but it was recorded in the books of account of
the firm at 2,00,000. Compute the capital gain if any, chargeable for the AY 2021-22.
22 INCOME TAX

Solution: Since the car is a movable property and was used by Ajay for his personal
purposes only, it will be treated as a personal effect. As personal effect is not a capital asset,
there will be no capital gain.

Illustration 8: In illustration No. 7 What would be the answer if Ajay had converted the
first car into stock-in-trade of his business?
Solution: The answer would be the same because personal effect is not a capital asset and
transfer of asset other than capital asset is not covered under capital gains and thus not
taxable.

Capital gain on transfer of a capital asset by way of distribution on the dissolution of a


firm, AOP/ BOI [Sec. 45(4)]
The profits or gains arising from the transfer of a capital asset to the partners/members
thereof by way distribution on the dissolution of a firm or other AOP or BOI (not being a
company or a cooperative society) or otherwise, shall be chargeable to tax as the income of
the firm, AOP/BOI, of the previous year in which the said transfer takes place. In such a case,
there will be a capital gain to the firm/ AOP/BOI. For the purposes of computation of capital
gain, the fair market value of the asset on the date of such transfer shall be deemed to be the
full value of the consideration received or accruing as a result of transfer, instead of the
value at which it is given to the partner/member.
Note:
1. Where depreciable asset is distributed, there will always be short-term capital
gain/loss based upon the particular block of assets. On the other hand, if non-
depreciable asset is distributed, it will be long-term capital gain or short-term capital
gain, depending upon the period of holding by the firm.
2. Although, for the purpose of computation of capital gains in the hands of firm/ AOP, the
sale consideration shall be the market value of the asset as on the date of its
distribution but the cost of acquisition of this asset to the partner/member shall be the
value at which it was transferred to partner/member.

Distribution of stock in trade amongst partners at the time of dissolution [Sec. 45(4)]
Stock in trade is not a capital asset and as such if stock in trade is distributed amongst the
partners, then as per A.L.A. Firm v CIT, the stock in trade is to be valued at market price, the
surplus, if any, has to be taxed as business income of the firm.
However, the Supreme Court in the case of Sakthi Trading Co. v CIT, observed that valuation
of closing stock is to be done at the market value only when there was dissolution and also
discontinuance of the business of the firm. But, where there is no discontinuance of the
business, the closing stock should be valued at cost or market price whichever is lower.
CAPITAL GAINS 23

Illustration 9: PQR & Company is a partnership firm, consisting of 3 partners P, Q and R the
firm is dissolved on 31-3-2021. The assets of the firm were distributed to the partners as
under:
Block of Machinery Stock Land
Particulars
(Given to P) (Given to Q) (Given to R)
Year of acquisition 2011-12 2017-18 1999-00

Cost of acquisition 7,20,000 4,00,000 90,000


Market value as on 31-3-2021 5,00,000 5,00,000 5,00,000
WDV as on 31-3-2021 4,40,000 -- --
Value at which given to partners
as per agreement 3,00,000 4,10,000 3,00,000
Market value as on 1-4-2001 -- -- 1,40,000
Compute the income taxable in the hands of the firm for the assessment year 2020-21.
Further what shall be the cost of acquisition of such assets to the partners of the firm.

Solution:

Short-term capital gain on block of machinery


Sale consideration (i.e. the market value) 5,00,000
Less: Cost of acquisition (i.e. the written down value of block) 4,40,000
Short-term capital gain 60,000
Business income on transfer of stock
Market value of stock 5,00,000
Less: Book Value of Stock 4,00,000
PGBP 1,00,000
Long-term capital gain on land
Sale consideration 5,00,000
Less: Indexed cost of acquisition – 1,40,000 × 301/100 4,21,400
Long-term capital Gain 78,600

Cost of acquisition of assets to the partners


Partner ‘P’ 3,00,000 for block of machinery
Partner ‘Q’ 4,10,000 for stock
Partner ‘R’ 3,00,000 for land

Illustration 10: Aswani firm consists of 3 partners namely Ravi, Giri and Sanjay. Sanjay
retires from the firm on 15-10-2020, His capital balance and the profits till the date of
retirement stood at 15,00,000. The firm transferred its land to Sanjay in settlement of his
24 INCOME TAX

account. The market value of the land as on that date was 25,00,000. The land was
acquired of by the firm on 1-5-2007 for 5,00,000.
Compute capital gain in the hands of the firm.

Solution:

Consideration price 25,00,000


Less: Indexed cost of acquisition – 5,00,000×301/129 11,66,667
Long-term capital gain 13,33,333

Capital Gain on Transfer by way of compulsory acquisition of an asset [Sec. 45(5)]


Where a capital asset, other than urban agricultural land, has been compulsorily acquired
under any law and compensation fixed by CG or RBI, it will be treated as a transfer of the
previous year in which the asset is compulsorily acquired. Indexation, if required, will be
done till the previous year of compulsory acquisition. However, the capital gain will be
taxable in the previous year in which the compensation is received.
 Initial compensation: Initial compensation shall be taken to be the sale consideration
of the asset and the capital gain shall be computed accordingly.
 Enhanced compensation: At times, the assessee is not satisfied with the
compensation/consideration determined and may go in for an appeal against the
amount determined. If on appeal the compensation/consideration is enhanced, it shall
be fully taxable as capital gain in the year in which the final order of such court
tribunal or other authority is made.The cost of acquisition and improvement thereto
will be taken as nil, since it has already been deducted at the time of computation of
capital gain for initial compensation. Such capital gain shall be long-tem or short-term
depending upon the original capital gain. It is possible that the person may die before
the enhanced compensation is received and the enhanced compensation is received by
his legal heirs. Such enhanced compensation will be taxable in the hands of the person
who receives the same. The expenses for realization (litigation expenses) of such
enhanced compensation may be claimed as a deduction as expenses on transfer.

Where right to receive the compensation is in dispute [Section 45(5)]


Where the amount of the compensation or consideration is subsequently reduced by any
court, tribunal or other authority, the capital gain of that year, in which the compensation or
consideration received was taxed, shall be recomputed by the Assessing Officer accordingly.
Hence, the capital gain shall be taxable even if the compensation amount is in dispute as the
assessee shall be entitled to get the assessment amended if compensation is later on
reduced.
CAPITAL GAINS 25

Capital gains on distribution of assets by companies in liquidation [Sec. 46]


Where the assets of a company are distributed to its shareholder on its liquidation, such
distribution shall not be regarded as a transfer by the company. Therefore, there will be no
capital gain to the company. However, where a shareholder on the liquidation of a company,
receives any money or other assets from the company in lieu of the shares held by him, such
a shareholder shall be chargeable to income-tax under the head ‘Capital gains’ in respect of
the money and the asset so received. In this case, the consideration price for capital gain
purposes shall be money received and/ or the market value of the other assets on the date
of distribution.
Note: In determining whether the capital gain in the above case is short term or long term,
the period subsequent to the date on which the company goes into liquidation shall not be
considered.

Capital gain on purchase by a company of its own shares/securities [Sec. 46A]


Any consideration received by a shareholder from any company on purchase of its own
shares held by such shareholder shall be chargeable to tax on the difference between the
cost of acquisition and the value of consideration received by the holder of shares, as capital
gains. The computation of capital gains shall be made in accordance with the provisions of
Section 48.
A domestic company will be liable to additional income tax at the rate of 20% (plus
applicable surcharge and cess) on buy back of its shares, not listed on a recognised stock
exchange, on the distributed income. Consequentially the amount received on account of
buy back of shares will be exempt in the hands of the shareholder.

Capital gains in the case of slump sale [Sec. 50B]: The provisions of Section 50B,
applicable for computation of capital gains in the case of slump sale, are given below:
 Any profits or gains arising from the slump sale effected in the PY shall be chargeable as
LTCG and shall be deemed to be the income of the PY in which the transfer took place.
Where, however, an undertaking owned and held by the assessee for not more than 36
months is transferred under the slump sale, then capital gain shall be deemed to be
STCG.
 In the case of slump sale of an undertaking, the “net worth” of the undertaking shall be
taken as cost of acquisition and cost of improvement.
 “Net worth” for this purpose is the aggregate value of total assets of the undertaking or
division as reduced by the value of liabilities of such undertaking or division as
appearing in the books of account. However, the following points should be noted:
1. Any change in the value of assets on account of revaluation of assets shall be
ignored for the purpose of computing the net worth.
2. In the case of depreciable asset, the depreciated value of assets of such undertaking
or division shall not exceed be the written down value of block of assets.
26 INCOME TAX

3. In the case of non-depreciable assets, book value shall be taken.


4. Net worth cannot be negative.
 The benefit of indexation will not be available.
 Every assessee, in the case of slump sale, shall furnish along with the return of income,
a report of a chartered accountant in Form No. 3CEA indicating the computation of the
net worth of the undertaking or division, as the case may be and certifying that the net
worth of the undertaking or division, as the case may be, has been correctly arrived at.

Illustration 11: Janta Janardan Ltd. is engaged in manufacture of chemical (since 1969) and
paper (since 2015). Following data is noted from the balance sheet of Janta Janardan Ltd. as
on 31.3. 2020.
( in thousands)
Equity Share Capital 6,000
Revaluation Reserve 600
Total 6,600

Chemical Paper division Total


division
Land 10,00 20,00 30,00
Plant and machinery 11,00 17,00 28,00
Debtors and other current assets 7,00 11,00 18,00
Less: Creditors 2,00 8,00 10,00
Total 26,00 40,00 66,00
Revaluation Reserve was created by making upward revision of land belonging to paper
division. The Company wants to transfer paper division on April 1, 2020 by way of slump
sale for a total consideration of 58 lakh (transfer expenses being 28,000). By taking into
consideration, the following additional information, find out the amount of capital gains and
other tax consequences.
1. Transfer agreement does not specify value of individual assets/liabilities.
2. The rate of depreciation on plant and machinery owned by Janta Janardan Ltd. is 15%.
The depreciated value of the block (consisting of chemical division and paper division)
on April 1, 2020 is 20 lakh for income tax purpose. Apart from transferring plant and
machinery of paper division, the company sells plant Q for 11 lakh on September 1,
2020. Plant Q belongs to chemical division. Plant and machinery of the paper division
was purchased in May 2015 for 30 lakh. The division started commercial production
in June 2016. No other asset for paper division is purchased/sold between May 2016
and March 2020.

Solution: Janta Janardan Ltd. transfers paper division for a lump sum consideration.
Transfer satisfies all conditions of Section 2(42C). Paper division was set up in 2016 and it
CAPITAL GAINS 27

is transferred on April 1, 2020. The capital gain (or loss) will be long-term. The sale
consideration is 58 lakh. The cost of acquisition is net worth of paper division which will
be determined as follows:

Computation of written value for the purpose of computing


depreciation
Depreciated value of the block of assets of chemical and paper division on
April 1, 2020 20,00,000
Less: Sale proceeds of Plant Q (-) 11,00,000
Balance (a) 9,00,000
Less: Depreciated value of assets of paper division(i.e. 15,66,019) [See
Note]; it cannot exceed (a) (-) 9,00,000
Written down value Nil
Depreciation available to X Ltd. for the previous year 2020-21 Nil

Computation of depreciated value of assets of paper division (as if


paper division is the only division) owned by Janta Janardan Ltd.
Depreciated value on April 1, 2016 Nil
Add: Cost of assets acquired and put to use during 2016-17 30,00,000
Written down value on March 31, 2017 30,00,000
Less: Depreciation for 2016-17 4,50,000
Depreciated value on April 1, 2017 25,50,000
Less: Depreciation for 2017-18 3,82,500
Depreciated value on April 1, 2018 21,67,500
Less: Depreciation for 2018-19 3,25,125
Depreciated value on April 1, 2019 18,42,375
Less: Depreciation for 2019-20 2,76,356
Depreciated value on April 1, 2020 15,66,019
Computation of net worth paper division
Land (excluding 6 lakh which was added by revaluation) 14,00,000
Plant or machinery (i.e. the amount actually deducted while computing
written down value of the block) 9,00,000
Debtors and other current assets 11,00,000
Total 34,00,000
Less: Creditors 8,00,000
Net worth 26,00,000
28 INCOME TAX

Computation of capital gain on transfer of paper division


Sale consideration 58,00,000
Less: Cost of acquisition (being net worth) 26,00,000
Expenses on transfer 28,000
Long-term capital gain 31,72,000

Transfer of security in Demat Form [Sec. 45(2A)]


1. Beneficial owner of shares/securities is chargeable to tax.
2. For computing capital gain chargeable to tax, the cost of acquisition and period of
holding of any security shall be determined on the basis of first-in-first-out (FIFO)
method.
3. FIFO method will be applied only in respect of the demat holdings because in case of
sale of demat securities, the securities held in physical form cannot be construed to
have been sold.
4. In the depository system, the investor can open and hold multiple accounts. In such a
case, where an investor has more than one demat account, FIFO method will be applied
account wise.
5. If in an existing demat account of old physical stock is dematerialized and entered at a
later date, under the FIFO method, the basis for determining the movement out of the
account is the date of entry into the account.

Illustration 12:
Date of Credit Particulars Quantity
June 1, 2020 Purchased directly in dematerialized form on May 25, 2020 4,000
June 5, 2020 Dematerialized share originally purchased in November, 1990 5,000
June 10, 2020 Purchased directly in dematerialized form of June 10, 2020 4,000
June 15, 2020 Dematerialized shares originally purchased in May, 1970 3,000
If say, 4,500 shares were sold from this account, then the period of holding and the cost of
acquisition of the first 4,000 shares should be as from May 25, 2020 and the cost thereof,
whereas the balance 500 shares will be treated as having been acquired in November 1990,
at the relevant cost. This is the effect of FIFO method.

Insurance claim received for damage or destruction of capital asset is treated as


capital gain [Sec. 45(1A)]
The following two conditions should be satisfied:
1. The compensation is received because of ‘damage to’ or ‘destruction of’ any capital
assets.
2. The damage or destruction is a result of four categories of circumstances, viz.,
(i) Flood, typhoon, hurricane, cyclone, earthquake or other convulsion of nature; or
(ii) Riot or civil disturbance; or
CAPITAL GAINS 29

(iii) Accidental fire explosion; or


(iv) Action by an enemy or action taken in combating an enemy (whether with or
without a declaration of war).
Where a person receives at any time during the previous year any money or other assets
under any insurance from an insurer and the above two conditions are satisfied, then the
receipt of such money or other assets shall be liable for “Capital gains” of the previous year
in which such money or other asset is received. The fair market value of other asset (on the
date of receipt) shall be deemed to be the full value of the consideration received.

When Sec. 45(1A) is not Applicable i.e. damage is not a result of 4 categories given
above:
1. When insurance compensation is a capital receipt: If the conditions mentioned are
not satisfied, the Section 45(1A) is not applicable and consequently, insurance
compensation (if it is a capital receipt) will not be chargeable to tax. For e.g. a road
accident takes place in which vehicles and machinery or furniture being carried are
destroyed. A ship, being overweight, is sunk and assets are lost. The receipt of
insurance compensation in such circumstances is not chargeable to tax u/s 45(1A).
2. When insurance compensation is a revenue receipt: If the conditions mentioned are
not satisfied and insurance compensation is a revenue receipt, then Section 45(1A) is
not applicable but the receipt may be taxable as a trading receipt under section 28 or
56. For instance, insurance compensation for theft of stock-in-trade is not taxable
under section 45(1A) but it will be taxable as PGBP.

Computation of capital gains in the case of land and building [Sec. 50C]
Sec. 50C is applicable if the following conditions are satisfied:
1. There is a transfer of land or building or both. The asset may be long-term capital asset
or short-term capital asset. It may be depreciable or non-depreciable asset.
2. The sale consideration is less than value adopted by “Stamp duty authority”. However,
it shall not apply if the difference is upto 10% of sale consideration or 50,000
whichever is higher.

Consequences if the above Conditions are Satisfied:


Full value of consideration for the purpose of
Different situations
capital gains
Where the assessee accepts the value Value adopted by Stamp duty authority is taken
adopted by Stamp duty authority. as full value of consideration.
Where the assessee has disputed value The stamp duty valuation as finally accepted for
adopted by Stamp duty authority under stamp duty purpose is taken as full value of
the Stamp Act (i.e., stamp duty consideration.
proceedings).
30 INCOME TAX

Where the assessee claims that value  Fair market value determined by the
adopted by Stamp duty authority is Valuation officer (if it is less than the stamp
more than the fair market value (but he duty valuation) is taken as full value of
has not disputed such valuation in consideration.
stamp duty proceedings).  Stamp duty valuation (if the fair market value
determined by the Valuation officer is more
than the stamp duty valuation) is taken as full
value of consideration.
It is also provided that if there is a time gap between the date of agreement and the date of
registration, the value for the purpose of the aforesaid comparison can be taken as the value
assessable for stamp duty on the date of the agreement, provided some part of the
consideration has been paid on or before the date of agreement by way of an account payee
cheque or account payee bank draft or use of electronic clearing system through a bank
account.

Illustration 13: Vishwanath owns a piece of land situated in Noida (date of acquisition:
March 1, 2007, cost of acquisition: 68,264, value adopted by Stamp duty authority at the
time of purchase: 95,000). On March 30, 2020 the piece of land is transferred for 4 lakh.

Find out the capital gains chargeable to tax in the following situations:
1. The value adopted by Stamp duty authority is 5.5 lakh. Vishwanath does not dispute
it.
2. The value adopted by the Stamp duty authority is 5.75 lakh. Vishwanath files an
appeal under the Stamp Act and Stamp duty valuation has been reduced to 4.90 lakh
by the Allahabad High Court.
3. The value adopted by the Stamp duty authority is 5.60 lakh. Vishwanath does not
challenge it under the Stamp Act. However, he claims before the Assessing Officer that
5.60 lakh is more than the fair market value of the land. The Assessing Officer refers it
to the Valuation Officer who determines 5.25 lakh as fair market value.
4. In Situation (3), suppose the value adopted by the Valuation Officer is 6.10 lakh.

Solution:
Situations
(1) (2) (3) (4)

Full value of consideration 5,50,000 4,90,000 5,25,000 5,60,000


Less: ICOA [68,264 × 301÷ 122] 1,68,422 1,68,422 1,68,422 1,68,422
Long- term capital gains 3,81,578 3,21,578 3,56,578 3,91,578
CAPITAL GAINS 31

Full value of consideration for transfer of share other than quoted share [Sec. 50CA]
Where the consideration received or accruing as a result of the transfer by an assessee of a
capital asset, being share of a company other than a quoted share, is less than the fair
market value of such share determined in such manner as may be prescribed, the value so
determined shall, for the purposes of section 48, be deemed to be the full value of
consideration received or accruing as a result of such transfer.

Exemptions available in computation of capital gain


Assessee to
Quantum of
Sec. whom Conditions to be satisfied
exemption
allowed
54 Individual 1. Transfer should be of a residential Actual amount invested
/HUF house, income of which is in new asset or the
chargeable under the head ‘income capital gain whichever
from house property’. is less.
2. It must be a long-term capital
asset.
3. Another residential house (can be
2 house for person having CG
upto 2 Crore) situated in India
should be purchased within one
year before or 2 years after, or
construction should be within 3
years after the date of transfer.
54 B Individual 1. Transfer (excluding compulsory Actual amount invested
/HUF acquisition) should be of in new asset or the
agricultural land. capital gain whichever
2. It must have been used in the 2 is less.
years immediately preceding the
date of transfer for agricultural
purposes either by the assessee or
his parent.
3. Another agricultural land should
be purchased within 2 years after
the date of transfer.
32 INCOME TAX

54D Any assessee 1. There must be compulsory Actual amount invested


which is an acquisition. in new asset or the
industrial 2. The property compulsorily capital gain whichever
undertaking acquired should be land and is less.
building forming part of an
industrial undertaking.
3. The asset must have been used in
the 2 years immediately preceding
the date of transfer of the assessee
for the purpose of the business of
the undertaking.
4. Within a period of 3 years after the
date of compulsory acquisition any
other land or building should be
purchased or constructed for the
use of industry.

54EC Any assessee 1. The asset transferred should be a Actual amount invested
long-term capital assets being subject to maximum
land or building or both. 50 lakhs, whether
2. Within a period of 6 months after such investment is
the date of transfer, the capital made in the same
gain must be invested in any bonds financial year or in the
specified by Central Government next financial year or
redeemable after 5 years (like partly in the same
NHAI, REC, PFC & IRFC). financial year and partly
in the next financial
year.

54EE Any Assessee 1. The asset transferred should be a Actual amount invested
long term capital asset. subject to maximum
2. The Capital Gain must be invested 50 lakh
in unit of specified fund (fund
financing Start Up India Project)
3. Investment within 6 months from
the date of transfer.
4. Units should not be transferred for
a period of 3 years.
CAPITAL GAINS 33

54 F Individual/ 1. The asset transferred should be a


HUF long-term capital asset, not being
residential house. LTCG× Amount invested
2. Within a period of 1 year before or Net Sale consideration
2 years after the date of transfer, a
residential house situated in
India should be purchased or
constructed within 3 years after
the date of transfer.
3. The assessee should not own more
than one residential house on the
date of transfer.
4. The assessee should not within a
period of 2 years purchase or
should not within a period of 3
years construct any residential
house other than the new house.
54G Any assessee 1. Machinery, plant, building, or land Actual amount invested
being an used for the business of an in new asset or the
industrial industrial undertaking situated in capital gain whichever
undertaking an urban area should have been is less.
transferred.
2. Transfer should be due to shifting
to any area other than an urban
area.
3. Within a period of 1 year before or
3 years after the date of transfer,
purchased machinery, plant or
acquired building or land or
constructed building and
completed shifting to the new area.
54GA Any assessee 1. Machinery, plant, building, or land Actual amount invested
being an used for the business of an in new asset or the
industrial industrial undertaking situated in an capital gain whichever
undertaking urban area should have been is less.
transferred
2. Transfer should be due to shifting
to any Special Economic Zone
whether developed in any urban
area or any other area.
34 INCOME TAX

3. Within a period of 1 year before or


3 Years after the date of transfer
purchased machinery, plant or
acquired building or land or
constructed building and
completed shifting to the new area.
54GB Individual 1. Long term Residential Property
/HUF (being house or plot of land) Actual amount invested
should have been transferred. in new asset or the
2. Net consideration is utilized for capital gain whichever
subscribing to more than 25% is less.
equity shares of startup
company.
3. The company invests the amount
received in new asset including
computer & computer software
before due date of filing ROI.

Note:
1. Capital gain Scheme: If the new asset is not acquired under section 54, 54B, 54D, 54F,
54G and 54GA or the full amount could not be invested upto the due date of furnishing
the return of income, the assessee can deposit the desired amount under the Capital
Gain Scheme on or before the due date of return and thus can acquire the asset within
the stipulated time out of money withdrawn from such scheme at a later date. In the
case of Section 54EC & 54EE the Capital Gain Scheme is not applicable.

2. Consequences if the new asset acquired is transferred within 3 years of its


acquisition:
(i) Under section 54, 54B, 54D, 54G and 54GA: For computation of new Capital
Gain, the cost of acquisition of such new asset shall be reduced by the amount of
Capital Gain exempt under section 54, 54B 54D, 54G and 54GA earlier.
(ii) Under section 54F: Besides the new capital gain, the Capital Gain exempt
earlier under section 54F, shall be long-term capital gain of the previous year in
which new asset is transferred.
(iii) Under section 54EC & 54EE: If such security acquired is converted into money
or any loan is taken against such securities within 5 years, the Capital Gain
exempt under sections 54EC or 54EE for such securities earlier shall be long-
term Capital Gain of the previous year in which such conversion takes place or
the loan is taken.
CAPITAL GAINS 35

3. Consequences if the amount deposited in Capital Gain Scheme is not utilized


within the stipulated time of 3 years (2 years in case of section 54B): The
unutilized amount shall be Capital Gain (short-term or long-term depending upon
original transfer) of the previous year in which such period has expired. However, in
case of Section 54F, proportionate amount shall be taxable.

4. Capital Gain exempt u/s 54GB earlier would be taxed if:


 Assessee transfers his shareholding in the start up company within a period of 5
year from the date of acquisition.
 The said company does not invest the amount in new asset before due date of
filing ROI.
 The said company transfers the new asset acquired within a period of 5 year from
the date of acquisition.

Computation of tax on long-term capital gains [Sec. 112]: Long-term capital gain is
taxable as under:
Asset Rate of Tax
Listed Securities (other than unit) or 10% (without benefit of indexation)
Zero Coupon Bond 20% (with benefit of indexation)
Other Assets 20% (with benefit of indexation

Incentives Under Sections 80C to 80U are not Available: Deductions under sections 80C
to 80U are not available in respect of long-term capital gains and STCG u/s 111A. Available
from other STCG.

Exemption Limit: Exemption Limit will be availed first for other income & then for capital
gain. Among capital gain, LTCG u/s 112 would be availed first, then STCG u/s 111A & then
LTCG u/s 112A.

Illustration 14: Harsh (28 years) is a resident individual. For the assessment year 2021-22,
he has the following incomes:
Particulars
Long-term capital gain (LTCG) 33,000
Other income 2,27,000
Net income (NI) 2,60,000
Solution: In this case, the long-term capital gain chargeable to tax will be 10,000.
Effectively, tax payable would be Nil as rebate of 12,500 subject to tax payable shall be
available u/s 87A.
36 INCOME TAX

Tax on LTCG from of Unlisted Securities [Sec. 112(1)(c)]


In case of non-residents, long term capital gains arising from unlisted securities shall be
chargeable to tax @ 10% without giving benefit of first & second proviso to Sec. 48.

Tax on Long term Capital gain [Sec. 112A] & short-term capital gain [Sec. 111A]
chargeable to STT
1. The taxpayer is an individual, HUF, firm, company or any other taxpayer.
2. During the previous year, he has generated short-term capital gain on transfer of equity
shares or units in equity-oriented mutual fund.
3. Such transaction is chargeable to securities transaction tax at the time of transfer or
transaction is undertaken in foreign currency through a recognized stock
exchange located in an IFSC (International Financial Services Centres) even if STT
is not paid.
4. If shares were acquired on or after 1-10-2004. STT must have been paid at the time of
acquisition.
5. STCG is taxable at the rate of 15% +SC + Cess. No deduction is available under section
80C to 80U from the above-noted short term capital gains.
6. LTCG exceeding 1,00,000 is taxable at the rate of 10%+SC + Cess. The indexation
benefit in respect of cost of acquisition and cost of improvement shall not be allowed.
No deduction is available u/s 80 C to 80 U & rebate u/s 87A from the above-noted Long
term capital gains.
7. Here cost of acquisition in respect of LTCA acquired by assessee before the 1st day of
February, 2018 shall be deemed to be the higher of (a) or (b)
(a) Actual Cost of Acquisition
(b) Lower of –
(i) The fair market value of such asset (highest price on exchange on 31.1.2018)
(ii) The full value of consideration received or accruing as a result of the transfer
of the capital asset.

Illustration 15: Mr. A converts his capital asset acquired for an amount of 50,000 in June,
2006 into stock-in-trade in the month of November, 2018. The fair market value of the asset
on the date of conversion is 4,50,000. The stock-in-trade was sold for an amount of
6,50,000 in the month of September, 2020. What will be the tax treatment?

Financial Year Cost Inflation Index


2006-07 122
2018-19 280
2020-21 301
CAPITAL GAINS 37

Solution: The capital gains on the sale of the capital asset converted to stock-in-trade is
taxable in the given case. It arises in the year of conversion (i.e. P.Y. 2018-19) but will be
taxable only in the year in which the stock-in-trade is sold (i.e. P.Y. 2020-21). Profits from
business will also be taxable in the year of sale of the stock-in-trade (P.Y. 2020-21).
The long-term capital gains and business income for the A.Y. 2021-22 are calculated as
under:
Particulars
Profit and Gains from Business or Profession
Sale proceeds of the stock-in-trade 6,50,000
Less: Cost of the stock-in-trade (FMV on the date of 4,50,000 2,00,000
conversion)
Long Term Capital Gains
Full value of the consideration (FMV on the date of the 4,50,000
conversion)
Less: Indexed cost of acquisition ( 50,000 x 280/122) 1,14,754 3,35,246
Note: For the purpose of indexation, the cost inflation index of the year in which the asset is
converted into stock-in-trade should be considered.

Illustration 16: Ms. Usha purchases 1,000 equity shares in X Ltd., at a cost of 30 per
share (brokerage 1%) in January 1996. She gets 100 bonus shares in August 2000. She
again gets 1100 bonus shares by virtue of her holding on February 2007. Fair market value
of the shares of X Ltd. on April 1, 2001 is 80. In January 2021, she transfers all her shares
@ 200 per share (brokerage 2%).
Compute the capital gains taxable in the hands of Ms. Usha for the A.Y. 2021-22
Cost inflation Index for F.Y. 2001-02: 100, F.Y. 2006-07: 122 & F.Y. 2020-21: 301.

Solution:
Computation of capital gains for the A.Y. 2021-22
Particulars
1000 Original shares
Sale proceeds (1000 × 200) 2,00,000
Less: Brokerage paid (2% of 2,00,000) 4,000
Net sale consideration 1,96,000
Less: Indexed cost of acquisition [ 80 × 1000 × 301/100] 2,40,800
Long term capital loss (A) (44,800)
100 Bonus shares
Sale proceeds (100 × 200) 20,000
Less: Brokerage paid (2% of 20,000) 400
38 INCOME TAX

Net sale consideration 19,600


Less: Indexed cost of acquisition [ 80 × 100 × 301/100] [See Note below] 24,080
Long term capital loss (B) (4,480)

1100 Bonus shares


Sale proceeds (1100 × 200) 2,20,000
Less: Brokerage paid (2% of 2,20,000) 4,400
Net sale consideration 2,15,600
Less: Cost of acquisition NIL
Long term capital gain (C) 2,15,600
Long term capital gain (A+B+C) 1,66,320
Note: Cost of acquisition of bonus shares acquired before 1.4.2001 is the FMV as on
1.4.2001 (being the higher of the cost or the FMV as on 1.4.2001).

Illustration 17: Mr. R holds 1000 shares in Star Minus Ltd., an unlisted company, acquired
in the year 2001-02 at a cost of 75,000. He has been offered right shares by the company
in the month of August, 2020 at 160 per share, in the ratio of 2 for every 5 held. He retains
50% of the rights and renounces the balance right shares in favour of Mr. Q for 30 per
share in September 2020. All the shares are sold by Mr. R for 300 per share in January
2021 and Mr. Q sells his shares in December 2020 at 280 per share.
What are the capital gains taxable in the hands of Mr. R and Mr. Q?
Financial Year Cost Inflation Index
2001-02 100
2020-21 301

Solution: Computation of capital gains in the hands of Mr. R for the A.Y. 2021-22
Particulars
1000 Original shares
Sale proceeds (1000 × 300) 3,00,000
Less: Indexed cost of acquisition [ 75,000 × 301/100] 2,25,750
Long term capital gain (A) 74,250
200 Right shares
Sale Proceeds (200 × 300) 60,000
Less: Cost of acquisition [ 160 × 200] [ Note 1] 32,000
Short term capital gain (B) 28,000
Sale of Right Entitlement
Sale proceeds (200 × 30) 6,000
Less: Cost of acquisition [Note 2] NIL
Short term capital gain (C) 6,000
CAPITAL GAINS 39

Note 1: Since the holding period of these shares is less than 24 months, they are short term
capital assets and hence cost of acquisition will not be indexed.
Note 2: The cost of the rights renounced in favour of another person for a consideration is
taken to be nil. The consideration so received is taxed as short-term capital gains in full. The
period of holding is taken from the date of the rights offer to the date of the renouncement.

Computation of capital gains in the hands of Mr. Q for the A.Y. 2021-22
Particulars
200 shares
Sale proceeds (200 × 280) 56,000
Less: Cost of acquisition [200 shares × ( 30 + 160)] [See Note below] 38,000
Short term capital gain 18,000
Note: The cost of the rights is the amount paid to Mr. R as well as the amount paid to the
company. Since the holding period of these shares is less than 24 months, they are short
term capital assets.

Illustration 18: X & sons, HUF, purchased a land for 1,20,000 in the P.Y. 2003-04. In the
P.Y. 2007-08, a partition takes place when Mr. A, a coparcener, is allotted this plot valued at
1,50,000. In P.Y. 2008-09, he had incurred expenses of 2,35,000 towards fencing of the
plot. Mr. A sells this plot of land for 15,00,000 in P.Y. 2020-21 after incurring expenses to
the extent of 20,000.
You are required to compute the capital gain for the A.Y. 2021-22.
Financial Year Cost Inflation Index
2003-04 109
2007-08 129
2008-09 137
2020-21 301
Solution:
Computation of taxable capital gains for the A.Y. 2021-22
Particulars
Sale consideration 15,00,000
Less: Expenses incurred for transfer 20,000
14,80,000
Less: (i) Indexed cost of acquisition ( 1,20,000 × 301/109) 3,31,376
(ii) Indexed cost of improvement ( 2,35,000 × 301/137) 5,16,314 8,47,690
Long term capital gains 6,32,310
Note – As per the view expressed by Bombay High Court in CIT v. Manjula J. Shah, in case
the cost of acquisition of the capital asset in the hands of the assessee is taken to be cost of
such asset in the hands of the previous owner, the indexation benefit would be available
from the year in which the capital asset is acquired by the previous owner. If this view is
40 INCOME TAX

considered, the indexed cost of acquisition would have to be calculated by considering the
Cost Inflation Index of F.Y. 2003-04.

Illustration 19: Mr. B purchased convertible debentures for 5,00,000 during August
2002. The debentures were converted into shares in September 2012. These shares were
sold for 15,00,000 in August, 2020. The brokerage expenses are 50,000. Market Price of
share on 31.1.20 was 12,00,000. You are required to compute the capital gains in case of Mr.
B for the assessment year 2021-22.
Financial Year Cost Inflation Index
2002-03 105
2012-13 200
2020-21 301

Solution:
Computation of Capital Gains of Mr. B for the A.Y. 2021-22
Particulars
Sale consideration 15,00,000
Less: Expenses on transfer i.e. Brokerage paid 50,000
Net consideration 14,50,000
Less: Cost of acquisition 12,00,000
Long term capital gain 2,50,000

Illustration 20: Mr. C purchases a house property for 1,06,000 on May 15, 1975. The
following expenses are incurred by him for making addition/alternation to the house
property:
Particulars
a. Cost of construction of first floor in 1982-83 3,10,000
b. Cost of construction of the second floor in 2002-03 7,35,000
c. Reconstruction of the property in 2012-13 5,50,000
Fair market value of the property on April 1, 2001 is 8,50,000. The house property is sold
by Mr. C on August 10, 2020 for 68,00,000 (expenses incurred on transfer: 50,000).
Compute the capital gain for the assessment year 2021-22.

Financial Year Cost Inflation Index


2001-02 100
2002-03 105
2012-13 200
2020-21 301
CAPITAL GAINS 41

Solution:
Computation of capital gain of Mr. C for the A.Y. 2021-22
Particulars
Gross sale consideration 68,00,000
Less: Expenses on transfer 50,000
Net sale consideration 67,50,000
Less: Indexed cost of acquisition (Note 1) 25,58,500
Less: Indexed cost of improvement (Note 2) 29,34,750 54,93,250
Long-term capital gain 12,56,750
Notes:
Indexed cost of acquisition; 8,50,000 × 301/100 = 25,58,500
Fair market value on April 1, 2001 (actual cost of acquisition is ignored as it is lower than
market value on April 1, 2001.)

Indexed cost of improvement is determined as under:


Particulars
Construction of first floor in 1982-83 Nil
(expenses incurred prior to April 1, 2001 are not considered)
Construction of second floor in 2002-03 (i.e., 7,35,000 × 301/105) 21,07,000
Alternation/reconstruction in 2012-13 (i.e., 5,50,000 × 301/200) 8,27,750
Indexed cost of improvement 29,34,750

Illustration 21: Mr. A is a proprietor of Akash Enterprises having 2 units. He transferred on


1.4.2020 his Unit 1 by way of slump sale for a total consideration of 25 lacs. Unit 1 was
started in the year 2005-2006. The expenses incurred for this transfer were 28,000. His
Balance Sheet as on 31.3.2020 is as under:
Liabilities Total Assets Unit 1 Unit 2 Total
( ) ( ) ( ) ( )
Own Capital 15,00,000 Building 12,00,000 2,00,000 14,00,000
Revaluation Reserve (for 3,00,000 Machinery 3,00,000 1,00,000 4,00,000
building of unit 1)
Bank loan (70% for unit 1) 2,00,000 Debtors 1,00,000 40,000 1,40,000
Trade creditors (25% for
unit 1) 1,50,000 Other 1,50,000 60,000 2,10,000
assets
Total 21,50,000 Total 17,50,000 4,00,000 21,50,000
Other information:
(i) Revaluation reserve is created by revising upward the value of the building of Unit 1.
(ii) No individual value of any asset is considered in the transfer deed.
42 INCOME TAX

(iii)
Other assets of Unit 1 include patents acquired on 1.7.2018 for 50,000 on which no
depreciation has been charged.
Compute the capital gain for the assessment year 2021-22.

Solution: Computation of capital gains on slump sale of Unit 1


Particulars
Sale value 25,00,000
Less: Expenses on sale 28,000
Net sale consideration 24,72,000
Less: Net worth (See Note 1 below) 12,50,625
Long-term capital gain 12,21,375

Notes:
1. Computation of net worth of Unit 1 of Akash Enterprises
Particulars
Building (excluding 3 lakhs on account of revaluation) 9,00,000
Machinery 3,00,000
Debtors 1,00,000
Patents (See Note 2 below) 28,125
Other assets ( 1,50,000 - 50,000) 1,00,000
Total assets 14,28,125
Less: Creditors 37,500
Bank Loan 1,40,000 1,77,500
Net worth 12,50,625
2. Written down value of patents as on 1.4.2020
Value of patents:
Cost as on 1.7.2018 50,000
Less: Depreciation @ 25% for Financial Year 2018-19 12,500
WDV as on 1.4.2019 37,500
Less: Depreciation for Financial Year 2019-20 9,375
WDV as on 1.4.2020 28,125
For the purposes of computation of net worth, the written down value determined as
per section 43(6) has to be considered in the case of depreciable assets. The problem
has been solved assuming that the Balance Sheet values of 3 lakh and 9 lakh ( 12
lakh - 3 lakh) represent the written down value of machinery and building,
respectively, of Unit 1.
3. Since the Unit is held for more than 36 months, capital gain arising would be long term
capital gain. However, indexation benefit is not available in case of slump sale.
CAPITAL GAINS 43

Illustration 22: Mr. Dinesh received a vacant site as gift from his friend in November 2006.
The site was acquired by his friend for 7,00,000 in April 2002. Dinesh constructed a
residential building during the year 2010-11 in the said site for 15,00,000. He carried out
some further extension of the construction in the year 2012-13 for 5,00,000.
Dinesh sold the residential building for 55,00,000 in January 2020 but the State stamp
valuation authority adopted 65,00,000 as value for the purpose of stamp duty.
Compute his long term capital gain, for the assessment year 2021-22 based on the above
information. The cost inflation indices are as follows:
Financial year Cost inflation index
2002-03 105
2005-06 117
2010-11 167
2012-13 200
2020-21 301
Solution:
Computation of long term capital gain of Mr. Dinesh for the A.Y. 2020-21
Particulars
Full value of consideration (Note 1) 65,00,000
Less: Indexed cost of acquisition-land ( 7,00,000 × 301/105) 20,06,667
(Note 2 & 3)
Indexed Cost of acquisition-building ( 15,00,000 × 301/167) 27,03,593
(Note 3)
Indexed Cost of improvement-building ( 5,00,000 × 301/200) 7,52,500 54,62,760
Long-term capital gain 10,37,240

Notes:
1. As per section 50C, where the consideration received or accruing as a result of
transfer of a capital asset, being land or building or both, is less than the value adopted
by the Stamp Valuation Authority, such value adopted by the Stamp Valuation
Authority shall be deemed to be the full value of the consideration received or
accruing as a result of such transfer, Accordingly, full value of consideration will be
65 lakhs in this case.
2. Since Dinesh has acquired the asset by way of gift, therefore, as per section 49(1), cost
of the asset to Dinesh shall be deemed to be cost for which the previous owner
acquired the asset i.e., 7,00,000, in this case.
3. In the case of CIT v. Manjula J. Shah 16 Taxmann 42 (Bom.), the Bombay High court
held that indexation cost of acquisition in case of gifted asset can be computed with
reference to the year in which the previous owner first held the asset.
44 INCOME TAX

Illustration 23: Mr. Kay purchases a house property on April 10, 1992 for 65,000. The
fair market value of the house property on April 1, 2001 was 2,70,000. On August 31,
2003. Mr. Kay enters into an agreement with Mr. Jay for sale of such property for 3,70,000
and received an amount of 60,000 as advance. However, as Mr. Jay did not pay the balance
amount, Mr. Kay forfeited the advance. In May 2008, Mr. Kay constructed the first floor by
incurring a cost of 2,35,000. Subsequently, in January 2009, Mr. Kay gifted the house to his
friend Mr. Dee. On February 10, 2021, Mr. Dee sold the house for 12,00,000. CII for F.Y.
2003-04: 109; 2008-09: 137; 2020-21: 301. Compute the capital gains in the hands of Mr.
Dee for A.Y. 2021-22

Solution:
Computation of taxable capital gains of Mr. Dee for A.Y. 2021-22
Particulars
Sale consideration 12,00,000
Less: Indexed cost of acquisition (See Note below) 5,93,212
Indexed cost of improvement (See Note below) 5,16,314 11,09,526
Long-term capital gain 90,474
Note:-
For the purpose of capital gains, holding period is considered from the date on which the
house was purchased by Mr. Kay, till the date of sale. However, indexation of cost of
acquisition is considered from the date on which the house was gifted by Mr. Kay to Mr. Dee,
till the date of sale. i.e. from January 2009 (P.Y. 2008-09) to February, 2021 (P.Y. 2020-21).
Indexed cost of acquisition = ( 2,70,000 × 301/137) = 5,93,212
Indexed cost of improvement = ( 2,35,000 × 301/137) = 5,16,314 Amount forfeited by
previous owner, Mr. Kay, shall not be deducted from cost of acquisition.

Illustration 24: Mr. X purchases a house property in December 1993 for 5,25,000 and an
amount of 1,75,000 was spent on the improvement and repairs of the property in March,
1997. The property was proposed to be sold to Mr. Z in the month of May, 2006 and an
advance of 40,000 was taken from him. As the entire money was not paid in time, Mr. X
forfeited the advance and subsequently sold the property to Mr. Y in the month of March,
2021 for 52,00,000. The fair value of the property on April 1, 2001 was 11,90,000. What
is the capital gain chargeable in the hands of Mr. X for the A.Y. 2021-22?
Financial year Cost Inflation Index
2001-02 100
2006-07 122
2020-21 301
CAPITAL GAINS 45

Solution: Capital gains in the hands of Mr. X for the A.Y. 2021-22 is computed as under:
Particulars
Sale proceeds 52,00,000
Less: Indexed cost of acquisition [Note 1] 33,23,500
Indexed cost of improvement [Note 2] --
Long term capital gains 18,76,500
Note 1: Computation of indexed cost of acquisition
Cost of acquisition (higher of fair market value as on April 1, 2001 and the 11,90,000
actual cost of acquisition
Less: Advance taken and forfeited 40,000
Cost for the purposes of indexation 11,50,000
Indexed cost of acquisition ( 11,50,000 × 301/100) 33,23,500
Note 2: Any improvement cost incurred prior to 1.4.2001 is to be ignored when fair market
value as on 1.4.2001 is taken into consideration.

Illustration 25: Mr. Cee purchased a residential house on July 20, 2018 for 10,00,000 and
made some additions to the house incurring 2,00,000 in August 2018. He sold the house
property in April 2020 for 20,00,000. Out of the sale proceeds, he spent 5,00,000 to
purchase another house property in September 2020.
What is the amount of capital gains taxable in the hands of Mr. Cee for the A.Y. 2021-22?

Solution: The house is sold before 24 months from the date of purchase. Hence, the house is
a short-term capital asset and no benefit of indexation would be available.

Particulars
Sale consideration 20,00,000
Less; Cost of acquisition 10,00,000
Cost of improvement 2,00,000
Short-term capital gains 8,00,000

Note: The exemption of capital gains under section 54 is available only in case of long-term
capital asset. As the house is short-term capital asset, Mr. Cee cannot claim exemption under
section 54. Thus, the amount of taxable short-term capital gains is 8,00,000.

Illustration 26: Mr. Roy, aged 55 years owned a Residential House in Ghaziabad. It was
acquired by Mr. Roy on 10-10-2007 for 24,00,000. He sold it for 65,00,000 on 4-11-
2020. The stamp valuation authority of the State fixed value of the property at 72,00,000.
The assessee paid 2% of the sale consideration as brokerage on the sale of the said
property.
46 INCOME TAX

Mr. Roy acquired a residential house property at Kolkata on 10-12-2020 for 7,00,000 and
deposited 3,00,000 on 10-4-2021 and 5,00,000 on 15-6-2021 in the capital gains bonds
of Rural Electrification Corporation Ltd. He deposited 4,00,000 on 6-7-2021 and
9,00,000 on 1-11-2021 in the capital gain deposit scheme in a Nationalized Bank for
construction of an additional floor on the residential house property in Kolkata.
Compute the Capital Gain chargeable to tax for the Assessment Year 2021-22.
Cost Inflation Index for Financial Year 2007-08: 129 and Financial Year 2020-21: 301

Solution:
Computation of Capital Gains chargeable to tax in the hands of Mr. Roy for the A.Y.
2021-22
Particulars
Gross Sale Consideration on transfer of residential house 72,00,000
[As per section 50C, in case the actual sale consideration is
lower than the stamp duty value fixed by the stamp valuation
authority, the stamp duty value shall be deemed as the full
value of consideration]
Less: Brokerage @ 2% of actual sale consideration of
65,00,000 1,30,000
Net Sale Consideration 70,70,000
Less: Indexed cost of acquisition [ 24 Lakh × 301/129] 56,00,000
Long-term capital gain 14,70,000
Less: Exemption under section 54
Acquisition of residential house property at Kolkata on 7,00,000
10.12.2020 (i.e., within the prescribed time of two years from
4.11.2020, being the date of transfer of residential house at
Ghaziabad).
Amount deposited in Capital Gains Accounts Scheme on or
before the due date of filing return of income for construction
of additional floor on the residential house property at Kolkata.
Since Mr. Roy has no other source of income, his due date for
filing return of income is 31st July, 2021
[Therefore, 4,00,000 deposited on 6.7.2021 will be eligible
for exemption whereas 9,00,000 deposited on 1.11.2021 will
not be eligible for exemption under section 54] 4,00,000 11,00,000
Exemption under section 54EC
Amount deposited in capital gains bonds of RECL within six
months from the date of transfer (i.e., on or before 3.5.2021)
would qualify for exemption. 3,00,000
[Therefore, in this case, 3,00,000 deposited in capital gains
CAPITAL GAINS 47

bonds of RECL on 10.4.2021 would be eligible for exemption


under section 54EC, whereas 5,00,000 deposited on
15.6.2021 would not qualify for exemption]
Long-term capital gain 2,93,256
Total Income (Rounded off) 2,93,260

Illustration 27: Aarav converts his plot of land purchased in July, 2003 for 80,000 into
stock-in-trade on 31st March, 2020. The fair market value as on 31.3.2020 was 3,00,000.
The stock-in-trade was sold for 3,25,000 in the month of January, 2021.
Find out the taxable income, if any, and if so under which ‘head of income’ and for which
Assessment Year?
Cost Inflation Index: F.Y. 2003-04: 109; F.Y. 2019-20: 289; F.Y. 2020-21: 301.

Solution: Conversion of a capital asset into stock-in-trade is a transfer within the meaning
of section 2(47) in the previous year in which the asset is so converted. However, the capital
gains will be charged to tax only in the year in which the stock-in-trade is sold.
The cost inflation index of the financial year in which the conversion took place should be
considered for computing indexed cost of acquisition. Further, the fair market value on the
date of conversion would be deemed to be the full value of consideration for transfer of the
asset as per section 45(2). The sale price less the fair market value on the date of conversion
would be treated as the business income of the year in which the stock-in-trade is sold.

Therefore, in this problem, both capital gains and business income would be charged to tax
in the A.Y. 2021-22.
Particulars
Capital Gains
Sale consideration (Fair market value on the date of conversion) 3,00,000
Less: Indexed cost of acquisition ( 80,000 × 289/109) 2,12,110
Long-term capital gain 87,890
Profits & Gains of Business of Profession
Sale price of stock-in-trade 3,25,000
Less: Fair market value on the date of conversion 3,00,000
25,000

Computation of taxable income of Mr. Aarav for A.Y. 2021-22


Particulars
Profits and gains from business or profession 25,000
Long term capital gains 87,890
Taxable income 1,12,890
48 INCOME TAX

Illustration 28: Mr. Malik owns a factory building on which he had been claiming
depreciation for the past few years. It is the only asset in the block. The factory building and
land appurtenant thereto were sold during the year. The following details are available:
Particulars
Building completed in September, 2009 for 10,00,000
Land appurtenant thereto purchased in April, 2002 for 12,00,000
Advance received from a prospective buyer for land in May, 2003, 50,000
Forfeited in favour of assessee, as negotiations failed
WDV of the building block as on 1.4.2020 8,74,800
Sale value of factory building in November, 2020 8,00,000
Sale value of appurtenant land in November, 2020 40,00,000
The assessee is ready to invest in long-term specified assets under section 54EC, within
specified time.
Compute the amount of taxable capital gain for the assessment year 2021-22 and the
amount to be invested under section 54EC for availing the maximum exemption.
Cost inflation indices are as under:
Financial Year Cost inflation Index
2002-03 105
2003-04 109
2020-21 301

Solution:
Computation of taxable capital gain of Mr. Malik for A.Y. 2021-22
Particulars
Factory building
Sale price of building 8,00,000
Less: WDV as on 1.4.2020 8,74,800
Short-term capital loss on sale of building (-) 74,800
Land appurtenant to the above building
Sale value of land 40,00, 000
Less: Indexed cost of acquisition ( 11,50,000 × 301/105) 32,96,667
Long-term capital gains on sale of land 7,03,333
Chargeable long term capital gain 6,28,533

Investment under section 54EC


In this case, both land and building have been held for more than 24 months and hence, are
long-term capital assets. Exemption under section 54EC is available if the capital gains
arising from transfer of a long-term capital asset are invested in long-term specified assets
like bonds of National Highways Authority of India and Rural Electrification Corporation
Ltd. or bonds notified by Central Government in this behalf, within 6 months from the date
CAPITAL GAINS 49

of transfer. As per section 54EC, the amount to be invested for availing the maximum
exemption is the net amount of capital gain arising from transfer of long-term capital asset,
which is 8,58,533 (rounded off to 8,58,530) in this case.
Notes:
1. Where advance money has been received by the assessee, and retained by him, as a
result of failure of the negotiations, section 51 will apply. The advance retained by the
assessee will go reduce the cost of acquisition. Indexation is to be done on the cost of
acquisition so arrived at after reducing the advance money forfeited i.e. 12,00,000 -
50,000 = 11,50,000. It may be noted that in cases where the advance money is
forfeited during the previous year 2014-15 or thereafter, the amount forfeited would
be taxable under the head “Income from Other Sources” and such amount will not be
deducted from the cost of acquisition of such asset while calculating capital gains.
2. Factory building on which depreciation has been claimed, is a depreciable asset.
Profit/loss arising on sale is deemed to be short-term capital gain/loss as per section
50, and no indexation benefit is available.
3. Land is not a depreciable asset, hence section 50 will not apply. Being a long-term
capital asset (held for more than 24 months), indexation benefit is available.
4. As per section 74, short term capital loss can be set-off against any income under the
head “Capital gains”, long-term or short-term. Therefore, in this case, short-term capital
loss of 74,800 can be set-off against long-term capital gain of 8,34,762.

Illustration29: Mr. ‘X’ furnishes the following data for the previous year ending 31.3.2021:
(a) Unlisted Equity Shares of AB Ltd., 10,000 in number were sold on 31.5.2020, at 500
for each share.
(b) The above shares of 10,000 were acquired by ‘X’ in the following manner:
(i) Received as gift from his father on 1.6.2000 (5,000 shares) the fair market value
on 1.4.2001 200 per share.
(ii) Bonus shares received from AB Ltd. on 21.7.2008 (2,000 shares).
(iii) Purchased on 1.2.2011 at the price of 350 per share (3,000 shares).
(c) Purchased one residential house at 25 lakhs, on 1.5.2021 from the sale proceeds of
shares.
(d) ‘X’ is already owning a residential house, even before the purchase of above house. You
are required to compute the taxable capital gain. He has no other source of income
chargeable to tax.
(Cost Inflation Index – F.Y. 2008-09: 137; 2010-11; 167; F.Y. 2020-21: 301)
50 INCOME TAX

Solution: Computation of taxable capital gain of Mr. ‘X’ for A.Y. 2021-22
Particulars
Sale consideration received on sale of 10,000 shares @ 500 50,00,000
each
Less: Indexed cost of acquisition
(a) 5,000 shares received as gift from father on 1.6.2000
Indexed cost 5,000 × 200 × 301/100 30,10,000
(b) 2,000 bonus shares received from AB Ltd Nil
Bonus shares are acquired on 21.7.2008 i.e. after 01.04.2001.
Hence, the cost is Nil.
(c) 3000 shares purchased on 1.2.2011 @ 350 per share.
The indexed cost is 3000 × 350 × 301/167 18,92,515 49,02,515
Long term capital gain 97,485
Less: Exemption under section 54F (See Note below)
97,485 × 25,00,000 / 50,00,000 48,742
Taxable long term capital gain 48,743

Note: Exemption under section 54F can be availed by the assessee subject to fulfillment of
the following conditions:
(a) The assessee should not own more than one residential house on the date of transfer of
the long-term capital asset;
(b) The assessee should purchase a residential house within a period of 1 year before or 2
years after the date of transfer or construct a residential house within a period of 3
years from the date of transfer of the long-term capital asset.
In this case, the assessee has fulfilled the two conditions mentioned above. Therefore, he is
entitled to exemption under section 54F.

Illustration 30: Mr. Kumar, aged 50 years, is the owner of a residential house which was
purchased in September, 2004 for 9,50,000. He sold the said house on 5th August, 2020 for
24,00,000. Valuation as per stamp valuation authority of the said residential house was
43,00,000. He invested 5,00,000 in NHAI Bonds on 12th January, 2021. He purchased a
residential house on 5th July, 2021 for 10,00,000. He gives other particulars as follows:
Interest on Bank Fixed Deposit 32,000
Investment in public provident fund 50,000
You are requested to calculate the taxable income for the assessment year 2021-22.
Cost inflation index for F.Y. 2004-05 and 2020-21 are 113 and 301, respectively.
CAPITAL GAINS 51

Solution:
Computation of total income of Mr. Kumar for the A.Y. 2021-22
Particulars
Capital Gains:
Sale price of the residential house 24,00,000
Valuation as per Stamp Valuation authority 43,00,000
(Value to be taken is the higher of actual sale price or valuation
adopted for stamp duty purpose as per section 50C)
Therefore, Consideration for the purpose of Capital Gains 43,00,000
Less: Indexed Cost of Acquisition
9,50,000 × 301/113 25,30,531
17,69,469
Less: Exemption under section 54 10,00,000
Exemption under section 54EC 5,00,000 15,00,000
Long-term capital gains 2,69,469
Income from other sources:
Interest on bank deposits 32,000
Gross Total Income 3,01,469
Less: Deduction under Chapter VI-A
Section 80C – Deposit in PPF (restricted to 32,000) 32,000
Total Income 2,69,469
Total Income (Rounded off) 2,69,470

Notes:
1. The basic exemption limit of 2,50,000 can be adjusted against long term capital gains.
2. Deduction under section 80C should be restricted to gross total income excluding long
term capital gain.

Illustration 31: Mrs. Harshita purchased a land at a cost of 35 lakhs in the financial year
2003-04 and held the same as her capital asset till 31st March, 2011. She started her real
estate business on 1st April, 2011 and converted the said land into stock-in-trade of her
business on the said date, when the fair market value of the land was 210 lakhs.
She constructed 15 flats of equal size, quality and dimension. Cost of construction of each
flat is 10 lakhs. Construction was completed in February, 2021. She sold 10 flats at 30
lakhs per flat in March, 2021. The remaining 5 flats were held in stock as on 31st March,
2021.
She invested 50 lakhs in bonds issued by National Highways Authority of India on 31st
March, 2021 and another 50 lakhs in bonds of Rural Electrification Corporation Ltd. in
April, 2021.
52 INCOME TAX

Compute the amount of chargeable capital gain and business income in the hands of Mrs.
Harshita arising from the above transactions for Assessment Year 2021-22 indicating
clearly the reasons for treatment for each item.
[Cost Inflation Index: FY 2003-04: 109; FY 2011-12: 184; FY 2020-21: 301].

Solution:
Computation of capital gains and business income of Harshita for A.Y. 2021-22
Particulars
Capital Gains
Fair market value of land on the date of conversion deemed as the full value 2,10,00,000
of consideration for the purposes of section 45(2)
Less: Indexed cost of acquisition [ 35,00,000 × 184/109] 59,08,257
1,50,91,743
Proportionate capital gains arising during A.Y. 2021-22 [ 1,50,91,743 × 2/3] 1,00,61,162
Less: Exemption under section 54EC 50, 00,000
Capital gains chargeable to tax for A.Y. 2021-22 50,61,162

Business Income
Sale price of flats [10 × 30 lakhs] 3,00,00,000
Less; Cost of flats
Fair market value of land on the date of conversion [ 210 lacs × 2/3] 1,40,00,000
Cost of construction of flats [10 × 10 lakhs] 1,00,00,000
Business income chargeable to tax for A.Y. 2021-22 60,00,000

Notes:
(1) The conversion of a capital asset into stock-in-trade is treated as a transfer under
section 2(47). It would be treated as a transfer in the year in which the capital asset is
converted into stock-in-trade.

(2) However, as per section 45(2), the capital gains arising from the transfer by way of
conversion of capital assets into stock-in-trade will be chargeable to tax only in the
year in which the stock-in-trade is sold.

(3) The indexation benefit for computing indexed cost of acquisition would, however, be
available only up to the year of conversion of capital asset into stock-in-trade and not
up to the year of sale of stock-in-trade.

(4) For the purpose of computing capital gains in such cases, the fair market value of the
capital asset on the date on which it was converted into stock-in-trade shall be
CAPITAL GAINS 53

deemed to be the full value of consideration received or accruing as a result of the


transfer of the capital asset.
In this case, since only 2/3rd of the stock-in-trade (10 flats out of 15 flats) is sold in the
P.Y. 2020-21, only proportionate capital gains (i.e., 2/3rd) would be chargeable to tax
in the A.Y. 2021-22.

(5) On sale of such stock-in-trade, business income would arise. The business income
chargeable to tax would be the difference between the price at which the stock-in-
trade is sold and the fair market value on the date of conversion of the capital asset
into stock-in-trade.

(6) In case of conversion of capital asset into stock-in-trade and subsequent sale of stock-
in-trade, the period of 6 months is to be reckoned from the date of sale of stock-in-
trade for the purpose of exemption under section 54EC [CBDT Circular No. 791 dated
2.6.2000]. In this case, since the investment in bonds of NHAI has been made within 6
months of sale of flats, the same qualifies for exemption under section 54EC. With
respect to long-term capital gains arising in any financial year, the maximum
deduction under section 54EC would be 50 lakhs, whether the investment in bonds
of NHAI or RECL are made in the same financial year or next financial year or partly in
the same financial year and partly in the next financial year.
Therefore, even though investment of 50 lakhs has been made in bonds of NHAI
during the P.Y. 2020-21 and investment of 50 lakhs has been made in bonds of RECL
during the P.Y. 2021-22, both within the stipulated six month period, the maximum
deduction allowable for A.Y. 2021-22, in respect of long-term capital gain arising on
sale of long-term capital asset(s) during the P.Y. 2020-21, is only 50 lakhs.

Illustration 32: Mr. Martin, a resident individual, sold his residential house property on 08-
06-2020 for 70 lakhs which was purchased by him for 20,50,000 on 05-05-2006.
He paid 1 lakh as brokerage for the sale of said property. The stamp duty valuation
assessed by sub registrar was 80 lakhs.
He bought another house property on 25-12-2020 for 15 lakhs.
He deposited 5 lakhs on 10-11-2020 in the capital gain bond of National Highway
Authority of India (NHAI).
He deposited another 10 lakhs on 10-07-2021 in the capital gain deposit scheme with SBI
for construction of additional floor of house property.
Compute income under the head “Capital Gains” for A.Y. 2021-22 as per Income-tax Act,
1961.
Cost inflation index for Financial Year 2006-07: 122 and 2020-21: 301.
54 INCOME TAX

Solution:
Computation of income under the head “Capital Gains” of Mr. Martin for A.Y. 2020-21
Particulars ₹ ₹
Long-term capital gain
Full value of consideration 80,00,000
(As per section 50C, in case the actual sale consideration (i.e.,
70 lakhs, in this case) is less than the stamp duty value (i.e., 80
lakhs, in this case) assessed by the stamp valuation authority
(Sub-registrar, in this case), the stamp duty value shall be
deemed as the full value of consideration]
Less: Expenses in connection with transfer (brokerage paid for
sale of property) 1,00,000
79,00,000
Less: Indexed cost of acquisition [ 20,50,000 × 280/122] 47,04,918 31,95,082
Less Exemption under section 54:
-Purchase of new residential house property within two years
from the date of sale of residential house 15,00,000
-Deposit in Capital Gains Accounts Scheme on or before the due
date of filing of return of income u/s 139(1) for construction of
additional floor on such house property. 10,00,000
25,00,000
Exemption under section 54EC:
Investment in capital gains bond of NHAI within 6 months from
the date of transfer (i.e., before 8.12.2020) 5,00,000 30,00,000
Taxable Capital Gains/Total Income 1,95,082
Total Income (rounded off) 1,95,080

Notes:
Exemption under section 54 is available is respect of reinvestment of capital gains on sale of
residential house in one residential house in India. In this case, exemption would be
available for amount invested in purchase of new residential house and amount deposited
for construction of additional floor in the same house, since they together constitute one
residential house.

Illustration 33: Mr. A is an individual carrying on business. His stock and machinery were
damaged and destroyed in a fire accident.
The value of stock lost (total damaged) was 6,50,000. Certain portion of the machinery
could be salvaged. The opening WDV of the block as on 1-4-2020 was 10,80,000.
CAPITAL GAINS 55

During the process of safeguarding machinery and in the fire fighting operations, Mr. A lost
his gold chain and a diamond ring, which he had purchased in April, 2004 for 1,20,000.
The market value of these two items as on the date of fire accident was 1,80,000.
Mr. A received the following amounts from the insurance company:
(i) Towards loss of stock 4,80,000
(ii) Towards damage of machinery 6,00,000
(iii) Towards gold chain and diamond ring 1,80,000
You are requested to briefly comment on the tax treatment of the above three items under
the provisions of the Income-tax Act, 1961.
Solution:
(i) Compensation towards loss of stock: Any compensation received from the
insurance company towards loss/damage to stock in trade is to be construed as a
trading receipt. Hence, 4,80,000 received as insurance claim for loss of stock has to
be assessed under the head “Profit and gains of business or profession”.
Note – The assessee can claim the value of stock destroyed by fire as revenue loss,
eligible for deduction while computing income under the head “Profits and gains of
business or profession”.
(ii) Compensation towards damage to machinery: The question does not mention
whether the salvaged machinery is taken over by the Insurance company or that the
salvaged machinery is taken over by the Insurance company, Assuming that the
salvaged machinery is taken over by the Insurance company, and there was no fresh
addition of machinery during the year, the block of machinery will cease to exist.
Therefore, 4,80,000 being the excess of written down value (i.e. 10,80,000) over
the insurance compensation (i.e. 6,00,000) will be assessable as a short-term capital
loss.
Note – If new machinery is purchased in the next year, it will constitute the new block
of machinery, on which depreciation can be claimed for that year.
(iii) Compensation towards loss of gold chain and diamond ring: Gold chain and
diamond ring are capital assets as envisaged by section 2(14). They are not “personal
effects”, which alone are to be excluded. As per section 45(1A), if any profit or gain
arises in a previous year owing to receipt of insurance claim, the same shall be
chargeable to tax as capital gains. The capital gains has to be computed by reducing
the indexed cost of acquisition of jewellery from the insurance compensation of
1,80,000.
56 INCOME TAX

MCQ

1. Which of the following would be regarded as transfer –


(a) Transfer of a capital asset in a scheme of reverse mortgage
(b) Transfer of a capital asset under a gift or will or an irrevocable trust
(c) Transfer by way of conversion of equity shares from preference shares
(d) Redemption of Zero coupon bond

2. Short-term capital gains arising on transfer of listed shares on which STT is paid at
the time of transfer, would be chargeable to tax –
(a) At the rate of 10%
(b) At the rate of 20%
(c) At the rate of 15%
(d) At the rate of 5%

3. Distribution of assets at the time of liquidation of a company –


(a) Is not a transfer in the hands of the company or the shareholders
(b) Is not a transfer in the hands of the company but capital gains it chargeable to
tax on such distribution in the hands of the shareholders
(c) Is not a transfer in the hands of the shareholders but capital gains is chargeable
to tax on such distribution in the hands of the company
(d) Is a transfer both in the hands of shareholders and company

4. Land or building would be long term capital asset only if it is


(a) Held for more than 12 months immediately preceding the date of transfer
(b) Held for more than 24 months immediately preceding the date of transfer
(c) Held for more than a30 months immediately preceding the date of transfer
(d) Held for more than 36 months immediately preceding the date of transfer

5. Capital gain on transfer of depreciable asset would be –


(a) Long term capital gain, if held for more than 36 months
(b) Long term capital gain, if held for more than 24 months
(c) Long term capital gain, if held for more than 12 months.
(d) Short term capital gain, irrespective of the period of holding

6. For an assessee, who is a salaried employee who invests in shares, what is the
benefit available in respect of securities transaction tax paid by him on sale and
CAPITAL GAINS 57

acquisition of 100 listed shares of X Ltd. which has been held by him for 14 months
before sale?
(a) Rebate under section 88E is allowable in respect of securities transaction tax
paid
(b) Securities transaction tax paid is treated as expenses of transfer and deducted
from sale consideration.
(c) Capital gains is completely exempt under section 10(38)
(d) Capital gains is taxable at concessional rate of 10% (if capital gain exceeds
1 lac)

7. Under section 50C, the guideline value for stamp duty is taken as the full value of
consideration only if-
(a) The asset transferred is building and the actual consideration is less than the
guideline value
(b) The asset transferred is either land or building or both and the actual
consideration is less than the guideline value
(c) The asset transferred is building, irrespective of the actual consideration
(d) The asset transferred is land and the actual consideration is less than the
guideline value

8. Where there is a transfer of a capital asset by a partner to the firm by way of capital
contribution or otherwise, the consideration would be taken as –
(a) The market value of the capital asset on the date of transfer
(b) The cost less national depreciation of the capital asset
(c) The value of the asset recorded in the books of the firm.
(d) Any of the above, at the option of the assessee

9. Under section 54F, capital gains are exempted if


(a) Long-term capital gain arising on transfer of residential house is invested in
acquisition of one residential house situated in or outside India.
(b) Long-term capital gain arising on transfer of a capital asset other than a
residential house is invested in acquisition of one residential house situated in or
outside India.
(c) Net sale consideration on transfer of a capital asset other than a residential
house is invested in acquisition of one residential house situated in India.
(d) Short term or long-term capital gain arising on transfer of a capital asset other
than a residential house is invested in acquisition of one residential house
situated in India
58 INCOME TAX

10. Under section 54EC, capital gains are exempted if invested in the bonds issued by
NHAI & RECL or other notified bond –
(a) Within a period of 6 months from the date of transfer of the asset
(b) Within a period of 6 months from the end of the relevant previous year
(c) Within a period of 6 months from the end of the previous year or the due date for
filing the return of income under section 139(1), whichever is earlier
(d) At any time before the end of the relevant previous year.
INCOME FROM OTHER
2
SOURCES
Basis of charge [Sec. 56]
Income from other sources is the last and residual head of income.
Sec. 56(1) covers income which does not fall under any other head of income.
Sec. 56(2) specifies nine incomes which are always taxable under the head “Income from
other sources”.

The following nine incomes are always taxable under the head “Income from other
sources”-
1. Dividend from company or dividend of co-operative society.
2. Winnings from lotteries, crossword puzzles, races including horse races, card games
and other games of any sort or from gambling or betting of any from or nature
whatsoever.
3. Sum received from employees as contribution to any staff welfare scheme (if it is not
taxable as business income under sec. 28), if it is not deposited before due date of such
act.
4. Interest on debentures, Government securities / bonds (if it is not taxed as business
income under sec. 28).
5. Rental income from machinery, plant or furniture let on hire (if the same is not taxed as
business income under sec. 28).
6. Rental income of letting out of plant, machinery or furniture along with letting out of
building and the two letting are not separable (if the same is not taxed as business
income under sec. 28).
7. Any sum received under a Keyman insurance policy including bonus (if the same is not
taxable as salary income or business income).
8. Money or property received without consideration or for inadequate consideration.
9. Interest received on compensation or on enhanced compensation.

Other incomes which are normally included under the head ‘Income from other
Sources’
Following are some other incomes which are normally chargeable to tax under this head
because these are not covered under any other heads:
(i) Income from sub-letting of a house property by a tenant;
(ii) Casual income;
(iii) Insurance commission;
60 INCOME TAX

(iv) Family pension (payments received by the legal heirs of a deceased employees);
(v) Director’s sitting fee for attending board meetings;
(vi) Interest on bank deposits/deposits with companies/ loans;
(vii) Interest received on delayed refund;
(viii) Income from royalty, if it is not income from business or profession;
(ix) Director’s commission for standing as a guarantor to bankers;
(x) Income from undisclosed sources;
(xi) Remuneration received by Members of Parliament;
(xii) Interest on securities of foreign governments;
(xiii) Director’s commission for underwriting shares of a new company;
(xiv) Examinership fees received by a teacher from an institution other than his
employer;
(xv) Rent from a vacant piece of plot of land;
(xvi) Agricultural income from agricultural land situated outside India;
(xvii) Income from granting of mining rights;
(xviii) Interest paid by the Government on excess payment of advance tax, etc
(xix) Amount withdrawn from NSS (National Saving Scheme). Incase of NSS 1987 Scheme
both principal and interest is taxable. But in case of NSS 1992 only interest being
taxable on withdrawl.
(xx) Advance received and retained on or after 1.4.2014 in the course of negotiation for
transfer of a capital assets which did not materialize.

Deemed Dividend [Sec. 2(22)]


1. Sec. 2(22)(a)
Dividends include any distribution of assets by a company to its shareholders to the
extent of accumulated profits of the company, whether capitalized or not.
Note: - When assets are distributed u/s 2(22)(a) the market value of the asset on the
date of distribution has to be taken for computation of the dividend.

2. Sec. 2 (22)(b)
• Any distribution to its shareholders by a company of debentures or deposit
certificate in any form;
• Any distribution to its preference shareholders of share by way of bonus;
To the extent of accumulated profits of the company, whether capitalized or not.

3. Sec. 2(22)(c)
Any distribution to the shareholders of a company on its liquidation to the extent of
accumulated profits of the company immediately before its liquidation, whether
capitalized or not.
INCOME FROM OTHER SOURCES 61

4. Sec. 2(22)(d)
Dividends include any distribution to its shareholders by a company on reduction of its
capital to the extent to which the company possesses accumulated profits whether
capitalized or not.

5. Sec. 2(22)(e)
Dividends include
(a) Any payment by a company being a closely held Co.
(b) of any sum by way of loan or advance
• To a shareholder being the beneficial owner of shares
• Holding not less than 10% of voting power
Or
• To any concern
• in which such a shareholder
• is a member or a partner
• and in which he has a substantial interest
Or
• To any person
• on behalf of such shareholder
(c) To the extent of accumulated profits of the company.

Summary
Section Payment by Payment in Payment to To the extent of
Equity Shares holder Accumulated Profit
2(22)(a) Any company Asset Preference Share whether
holder Capitalised or not
Debenture, Equity Share holder
Deposit Preference Share
Certificate holder
2(22)(b) Any company ,,
Preference Share
Bonus holder

Any company
2(22)(c) Asset Equity Share holder ,,
in liquidation
Any company
2(22)(d) reducing Assets Equity Share holder ,,
capital
62 INCOME TAX

Equity Share holder


(holding ≥ 10%
Voting Power)
Closely held Loan/ Concern (in which
2(22)(e) Accumulated Profit
Company Advance equity share holder
holding ≥ 10% Voting
Power has substantial
interest in concern)
Note:
(a) Security Premium is not accumulated profit.
(b) Accumulated Profit is to be taken upto the date of distribution. However, for Sec.
2(22)(c) profit upto the date of winding is taken.
(c) Trade advances would not fall within the ambit of the word ‘advance’ in Section
2(22)(e) and therefore would not be treated as deemed dividend.
(d) Sec. 2(22)(e) is not applicable to any advance or loan given by a company during the
normal course of its business and if the money lending is the major part of the business
of the company.
(e) If a Loan/ Advance is deemed as dividend and further the company declares actual
dividend which is adjusted against the aforesaid loan/ advance, then the amount so
adjusted is not liable for tax again.

Illustration 1: Z Ltd. is a company in which the public are not substantially interested. K is
a shareholder of the company holding 15% of the equity shares. The accumulated profits of
the company as on 31.03.2020 amounted to ` 10,00,000. The company lent ` 2,00,000 to K
by an account payee bank draft on 01.10.2020. The loan was not connected with the
business of the company. K repaid the loan to the company by an account payee bank draft
on 30.03.2021. Examine the effect of the borrowal and repayment of the loan by K on the
computation of his total income for the assessment year 2021-22.
Solution: U/s 2(22)(e), payment of any sum by a Company in which the public are not
substantially interested, by way of advance or loan, to the extent the Company possesses
accumulated profits, to a shareholder, who is the beneficial owner of shares with not less
than 10% voting power, shall be deemed to be dividend. In the given case Mr. K is a
beneficial owner of 15% of voting power. Therefore, the amount of ` 2 Lakh so paid to him
shall be treated as deemed dividend u/s 2(22) (e) even if the amount is repaid during the
year.

Illustration 2: Mr. Jugal took a loan of ` 2,20,000 from a company in which the public is not
substantially interested. He holds 25% equity shares of the company. On the date of loan
and paying the premium, the accumulated profits of the company were ` 1,80,000.
INCOME FROM OTHER SOURCES 63

Subsequently in the same year, the company declared dividends. On his shareholding Mr.
Jugal was entitled to a dividend amounting to ` 35,000 which was set-off against the
amount of loan.
Compute the amount that should be included in the income of Mr. Jugal.
Solution:
`
Amount of loan taken ` 2,20,000
Accumulated profit of the company ` 1,80,000
Hence, the deemed divided shall be (Loan taken or the amount of accumulated
profits, whichever is less) 1,80,000
The amount of actual dividend ` 35,000 is set-off against loan, hence, such dividend is not to
be treated as dividend and Mr. Jugal shall not have to pay any tax on this dividend of
` 35,000.

Special rate of tax in case of winnings from lotteries, crossword puzzles, races, etc.
[Sec. 115BB]
Winnings from lotteries, etc. is taxable at a special rate of Income-tax, which at present, is
30% + Cess @ 4%. It may, however, be mentioned that according to section 58(4), no
deduction in respect of any expenditure or allowance, in connection with such income, shall
be allowed. In other words, the entire income of winnings, without any expenditure or
allowance, will be taxable. In fact, deduction under sections 80C to 80U will also not be
available from such income although such income is a part of the total income. As lottery
income is taxed at flat rate, the basic exemption limit (say ` 2,50,000) is not available to the
assessee.

Interest on Securities [Sec. 56(2)(id)]


Income by way of interest on securities, is chargeable under the head “income from other
sources”. If such income is not chargeable to income-tax under the head, “Profits and Gains
of Business or Profession”.
Interest on securities may be taxed on receipt basis or on due basis, depending upon the
system of accounting if any, adopted by the assessee. If the assessee follows the cash system
of accounting, interest is taxable on receipt basis otherwise it shall be taxable on due basis.
If no system of accounting is followed, it will always be taxable on ‘due’ basis.
Interest on securities accrues or becomes due on a specified date and not on a day-to-day
basis. The date on which the interest shall become due is specified by the issuing company,
which may be quarterly basis, half yearly basis or annual basis. For example, if a company
issues 12% debentures and specifies that interest shall become due on 31st December every
year, the due date is 31st December and the interest for the entire year shall become due
only on 31st December every year. The person, who is the registered owner of the
64 INCOME TAX

debentures as on 31st December, shall be entitled to receive the interest of the full year
irrespective of his period of holding.

Interest received by an assessee on compensation or on enhanced compensation will be


taxed on the basis of receipt irrespective of accounting system followed.

Avoidance of tax by certain transaction in securities (also known as Bond Washing


Transaction) [Sec. 94]
Interest on securities becomes due on specified dates and is treated as an income of the
person who owns the securities on the due date. Therefore, if the owner of any security sells
the securities just before the due date, he will be able to avoid the payment of tax on the
interest by shifting the burden to other who may have no other income or very little income.
Section 94(1) takes care of such practice. As per section 94(1) if the owner of any securities
sells or transfers any securities, and buys back or re-acquires the same or similar securities
and if the result of the transaction is that any interest becoming payable in respect of such
securities is receivable by any other person, such interest would be deemed to be the
income of the transferor and not of the transferee. The provisions of section 94(1) are
however, not applicable, if the owner proves that:
(a) There has been no avoidance of Income-tax; or
(b) The avoidance of Income-tax was exceptional and not systematic.

Grossing Up
1. Lottery income, horse races and crossword puzzles etc
As in the case of some other incomes, there is also a provision for tax to be deducted at
source from income from winning of lotteries, horse races and crossword puzzles. The
rate of TDS in the case of such incomes is 30% if the income exceeds ` 10,000. Such tax
deducted at source is income and the amount received is net income after deduction of
tax at source. In this case, such net income will have to be grossed up as under. If a
person wins a lottery of ` 1,00,000, tax must have been deducted @ 30% and net
amount received by the assessee would be ` 70,000 (1,00,000 – 30,000). Grossing up
would be done as ` 70,000 × 100/100-30 = ` 1,00,000.

2. Interest Income
Tax is also to be deducted at source on interest on securities at the prescribed rates of
tax. For income-tax purposes what is to be charged to tax is the gross amount of
interest. Therefore, if the net-interest is given, it has to be grossed up to arrive at the
taxable amount. In the case of government securities other than GOI Saving (Taxable)
Bonds, grossing up is not required as there is no deduction of tax at source. However,
grossing up is required in the case of the following securities:-
INCOME FROM OTHER SOURCES 65

(i) 8% GOI Saving (Taxable) Bonds 2003


(ii) 7.75% GOI Saving (Taxable) Bond, 2018
(iii) Tax-free non-government securities;
(iv) Less tax non-government securities;
Net Interest can be grossed up as under:
Net Interest
× 100
100 − Rate of TDS
The rates of T.D.S. are as under:
(i) In case of GOI saving bonds if interest exceeds ` 10,000 p.a. – 10%
(ii) Government Securities Whether listed or unlisted (Although Taxable) Nil
(iii) Interest on Non-government securities whether listed or unlisted exceeds
` 5,000 p.a. − 10%

Deductions for expenses from Interest on Securities [Sec. 57]


The following deductions will also be allowed from the gross interest on securities:
(a) Collection charges [Sec. 57(i)] Any reasonable sum paid by way of commission or
remuneration to a banker, or any other person for the purpose of realising the interest.
(b) Interest on loan [Sec. 57(ii)] Interest on money borrowed for investment in securities
can be claimed as a deduction.
(c) Any other expenditure [Sec. 57(iii)] Any other expenditure, not being a expenditure
of a capital nature, expended wholly and exclusively for the purpose of making or
earning such income can be claimed as a deduction.
(d) Deduction from interest on compensation or enhanced compensation [Sec.
57(iv)] In the case of above interest which is taxable under the head income from other
sources, a deduction of a sum equal to 50% of such income shall be allowed to the
assessee and no deduction shall be allowed under any other clause of section 57.
(e) The only deduction that will be allowed against dividend income will be of interest and
that too shall be restricted to 20% of the dividend income.

Interest exempt from tax [Sec. 10(15)]


• Railway Bond
• 7 % Capital Investment Bonds
• 9 % Relief Bonds, 1987
• Interest on notified bonds/debentures of a public sector company
• Interest on securities held by the Welfare Commissioner, or interest on deposits for
the benefit of the victims of the Bhopal gas leak disaster
• Notified bonds issued by local authority or by State Pooled Finance Entity
• Interest on Gold Deposit Bonds issued under the Gold Deposit Scheme, 1999
notified by the Central Government
66 INCOME TAX

• Interest on Gold Deposit Certificate issued under Gold Monetization Scheme, 2015.
• Interest on Public Provident Fund
• Interest on Sukanya Samriddhi Account Scheme
• Interest on Post Office Saving Bank Account. Interest beyond ` 3,500 & ` 7,000 (in
Case of Joint Post office saving bank A/c) shall be taxable.
Note:
(i) Interest on NSC is taxable. But accrued interest of initial 5 year is eligible for
deduction u/s 80C.
(ii) Deduction for interest on Saving Account u/s 80TTA upto ` 10,000 & any bank
interest u/s 80TTB upto ` 50,000 shall be available.

Income from letting out of machinery, plant or furniture [Sec. 56(2)(ii)]


Income from machinery, plant or furniture, belonging to the assessee and let on hire, is
chargeable as income from other sources, if the income is not chargeable to income-tax
under the head “Profits and Gains of Business or Profession”.

Income from composite letting of machinery, plant or furniture and buildings [Sec.
56(2)(iii)]
Where an assessee lets on hire the machinery, plant or furniture belonging to him and also
buildings, and the letting of the buildings is inseparable from the letting of the said
machinery, plant or furniture, the income from such letting, known as composite rent, if it is
not chargeable to tax under the head PGBP, shall be chargeable as income from other
sources.

GIFT

Money Moveable Immoveable Property

1. Cash 1. Share & Securities 1. Land


2. Cheque 2. Jewellery 2. Building
3. Draft 3. Bullion 3. Land & Building
4. Archeological Collection
5. Drawings
6. Paintings
7. Sculptures
8. Any work of art
INCOME FROM OTHER SOURCES 67

Receipts without consideration to be treated as income [Sec. 56(2)(x)]


Money or property received by any person without consideration or for inadequate
consideration is income if it does not fall in the exempted category.

Receipt of sum of money/ property without consideration or for inadequate


consideration should fall in any of the following categories:
Whether single or
Different all transaction be
Tax Treatment
categories considered for the
ceiling of ` 50,000
Category 1 – If aggregate amount of sum of money received by
Any sum of any person without consideration from one or
money (gift in more persons during a previous year exceeds
All transactions
cash or by ` 50,000, the whole of such aggregate value will be
cheque or chargeable to tax.
draft).
Category 2 : If aggregate fair market value of movable
Movable properties received without consideration during
property a previous year exceeds ` 50,000, the whole of All transactions
without aggregate fair market value of movable property
consideration or properties will be chargeable to tax.
Category 3 : If movable property is received for a consideration
Movable which is less than the aggregate fair market value
property for of the property by an amount exceeding ` 50,000, All transactions
inadequate then the difference between aggregate fair market
consideration value and the consideration is chargeable to tax.
Category 4 : If any immovable property (without any
Immovable consideration) is received and the stamp duty
property value of which exceeds ` 50,000, stamp duty value Single transaction
without will be chargeable to tax.
consideration
If any immovable property is received for a
Category 5 :
consideration which is less than the stamp duty
Immovable
value of the property by an amount exceeding
property Single transaction
higher of ` 50,000, or 10% of consideration then
inadequate
the difference between Stamp duty value and the
consideration
consideration is chargeable to tax.

In a case where the date of the agreement to purchase the property fixing the consideration
and the date of registration are different, the taxability will be determined with
68 INCOME TAX

reference to the stamp duty value on the date of agreement and not registration. This
exception will apply only where a part of the consideration has been paid by any mode
other than cash, on or before the date of such agreement.

Exempted Categories
While calculating the above monetary limit of ` 50,000 in any of the categories, any sum of
money or property received from the following shall not be considered.
1. Money/ property received from a relative.
2. Money/ propery received on the occasion of the marriage of the individual.
3. Money/ property received by way of will/ inheritance.
4. Money/ property received in contemplation of death of the payer.
5. Money/ property received from a local authority.
6. Money/ property received from any fund, foundation, university, other educational
institution, hospital, medical institution, any trust or institution referred to in section
10(23C).
7. Money/ property received from a charitable institute registered under section 12AA.

Other Relevant Points


1. Where the capital gain arises from the transfer of a property, the value of which has
been subject to income tax u/s 56(2)(vii), the cost of acquisition of such property shall
be deemed to be the value which has been taken into account for the purpose of
56(2)(vii). Here the period of holding of previous owner shall also be not considered.

2. Sec 56(2) shall apply if property is in nature of “Capital Asset” in hands of recipient. If
property is “Stock-in-trade” in the hands of recipient, then Sec. 56(2)(vii) has no
application. For example, if an individual who is a jeweller buys jewellery from a
customer of ` 80 lakhs and FMV of jewellery is ` 100 lakhs. Now, section 56(2) (vii)
shall not apply since jeweller receives jewellery as stock-in-trade. As and when he will
sell the jewellery say for ` 105 lakhs then ` 25 lakhs shall be taxable as his Business
Income.

3. If cash gift of ` 2 lac or more are received, apart from tax penalty u/s 269ST would get
attract equivalent to the amount involved. Even cash gifts received from parents on the
occasion of marriage would attract penalty.

4. Relative for the purpose of HUF shall mean any member of the HUF. Accordingly, gift
received by a HUF from its members will not be taxed as income.

5. Purchase from Registered Dealer – Invoice Value shall be FMV. Therefore, no income is
liable to tax u/s 56(2)(vii).
INCOME FROM OTHER SOURCES 69

6. The term “relative” in respect of an individual means.

Relative

Father Mother
Father Mother

Brother Brother
Father Mother
Father Mother
Sister Sister

Brother Brother
Mr. A
(Assesse) Spouse of Mr. A

Sister Sister

Son Daughter

Son Daughter

& Spouse of all the above

Note : Maternal Grandfather & Grandmother are not relative for the purpose of Sec. 56.

As a measure of tax planning, one should avoid giving gift to spouse or son’s wife because in
that case clubbing provisions u/s 64 will be attracted that is income from the gifted amount
will be added in the income of the donor. Fiance & Fiancee is not relative.
70 INCOME TAX

Illustration 3: Flora received the following gifts during the year ending 31.03.2020:
(a) ` 40,000 from her elder sister.
(b) ` 60,000 from the daughter of her elder sister.
(c) ` 1,25,000 from various friends on the occasion of her marriage,
Discuss the taxability or otherwise of these gifts in the hands of Flora.
Solution:
(a) ` 40,000 received from elder sister, is not taxable, as elder sister is a relative.
(b) ` 60,000 received from the daughter of her elder sister, is taxable, as she is not a
relative as per the definition of the Act.
(c) ` 1,25,000 is not taxable as it is received on the occasion of her marriage.

Illustration 4: Check the taxability of the following gifts received by Mrs. Kumkum during
the previous year 2020-21 and compute the taxable income from gifts for Assessment Year
2021-22.
(i) On the occasion of her marriage on 14.8.20, she has received ` 90,000 as gift out of
which ` 70,000 are from relatives and balance from friends.
(ii) On 12.9.20, she has received cash gift of ` 18,000 from cousin of her mother.
(iii) A cell phone worth ` 21,000 is gifted by her friend on 15.10.2020.
(iv) She gets a cash gift of ` 25,000 from the elder brother of her husband’s grandfather
on 25.10.2020.
(v) She has received a cash gift of ` 12,000 from her friend on 14.12.2020.
Solution:
Computation of taxable income of Mrs. Kumkum from gifts for A.Y. 2021-22
Sl. Particulars Taxable Reason for taxability or otherwise of
No. amount each gift
`
1. Relatives and friends Nil Gifts received on the occasion of marriage are
not taxable
2. Cousin of Mrs. Kumkum’s 18,000 Cousin of Mrs. Kumkum’s mother is not a
mother relative. Hence, the cash gift is taxable.
3. Friend -- Cell phone is not included in the definition of
property. Hence, it is not taxbale.
4. Elder brother of 25,000 Brother of husband’s grandfather is not a
husband’s grandfaher relative. Hence, the cash gift is taxable.
5. Friend 12,000 Cash gift from friend is taxable.
Total 55,000
Since the sum of money received by Mrs. Kumkum without consideration during the
previous year 2020-21 exceeds ` 50,000, the whole of the amount is chargeable to tax
under section 56(2)(vii) of the Income-tax Act, 1961.
INCOME FROM OTHER SOURCES 71

Illustration 5: X gives the following information about movable properties (being capital
assets)

Movable properties Fair market value Purchase Difference (4) [i.e.,


(1) (2) price excess of (2) over
(3) (3)]
` ` `
Received by gift-
Gold Ring from A 22,000 Nil 22,000
Painting from B 16,000 Nil 16,000
Sculptures from C 12,000 Nil 12,000
Total 50,000 Nil 50,000
Purchase for inadequate
consideration-
Drawing from D 80,000 65,000 15,000
Silver from E 85,000 75,000 10,000
Archeological Collection from F 60,000 35,000 25,000
Total 2,25,000 1,75,000 50,000

Solution: In this case, the aggregate fair market value of movable properties received by
gift is not more than ` 50,000. Consequently, nothing is taxable in respect of movable
properties received by gift. The aggregate fair market value of movable properties
purchased is ` 2,25,000 and the aggregate purchase price is ` 1,75,000. The difference is not
more than ` 50,000. Nothing is taxable under section 56(2)(vii) on account of purchase of
movable properties for inadequate consideration. If, however, the fair market value of gold
ring is ` 25,000 and silver is ` 90,000, then ` 1,08,000 will be taxable under the head
“Income from other sources” in the hands of X.

Illustration 6: From the following information given below determine the income of Mr.
Manish who is Chartered Accountant by profession for the AY 2021-22 if his spouse has no
income.
(a) He purchases jewellery from a person (other than registered dealer) for ` 3,00,000
(Fair market value ` 4,20,000).
(b) He purchases Mona Lisa Painting from Raj Art Emporium (registered GST dealer). The
painting is purchased under a tax invoice at ` 3,50,000. However, the same painting
shall not be available for less than ` 5,00,000 from any other Art Emporium.
(c) Purchases Bullion from Varanasi Swarna Kala Kendra (a registered GST dealer) at
Invoice Price of ` 2,50,000. However, its FMV is 2,90,000.
72 INCOME TAX

(d) His Minor Son receives a gold chain of ` 27,000 from maternal uncle & a gold ring of
` 57,000 from maternal grandfather.

Solution: Computation of income of Mr. Manish


Nature of Gift Amount
Purchase of Jewellery for inadequate consideration (4,20,000 – 3,00,000) 1,20,000
Purchase of painting from registered dealer (Invoice price is taken as FMV) Nil
Purchase of Bullion from registered dealer (Invoice price taken as FMV) Nil
Gift received from Maternal Uncle (Relative) Nil
Gift Received from Maternal Grandfather (Not Relative) 57,000
Total 1,77,000
Note: Income of Minor child clubbed in the hand of parent whose income is higher.

Receipt of share premium by a company treated as income [Sec. 56(2)(viib)]


Where a closely held company issues shares to a resident for amount exceeding the face
value of the shares, then the amount received in excess of the fair market value of the shares
is deemed to be the income of the company. The fair market value for this purpose is the
higher of the value arrived at on the basis of the method to be prescribed and value as
substantiated by the company to the satisfaction of the Assessing Officer. The company can
substantiate the value based on the value of the tangible and intangible assets and various
types of commercial rights.
The provision does not apply to amounts recevied by a venture capital undertaking from a
venture capital fund or a venture capital company.

Family pension payments received by the legal heirs of a deceased employee


After the death of the employee, if there is any family pension received by the legal heirs of
the deceased, it will deemed to be the income of the legal heir and will be taxable under the
head ‘Income from other Sources’. On such pension, as per section 57 a standard deduction
shall be allowed to the legal heir @33 % of such pension, or ` 15,000, whichever is less.
Note:
(a) Ex-gratia payment made to the widow or other legal heirs of an employee, who dies
while still in active service, will not be taxable, if it is paid by Central Government/ State
Government/ Local Authority.
(b) Lump sum payment made gratuitously or by way of compensation or otherwise to the
widow or other legal heirs of an employee who dies while in active service is not
taxable.
(c) Family pension received by a widow of a member of the armed forces where the death
of the member has occurred in the course of the operational duties is totally exempt
under section 10(19).
INCOME FROM OTHER SOURCES 73

Amounts not deductible in computing income under the head ‘Income from Other
Sources’ [Sec. 58]
The following payments shall not be deductible in computing the income chargeable under
the head ‘Income from Other Sources’:
1. Personal expenses of the assessee.
2. Interest & Salaries paid outside India on which tax has not been deducted at source.
3. Expenses paid to a resident in India where Sec. 40(a)(ia) not complied would result in
30% disallowance.
4. Any expenditure referred to in section 40A like excessive or unreasonable payments to
certain specified persons [Sec. 40A(2)] and payments exceeding ` 10,000 otherwise
than by way of account payee cheque [Sec. 40A(3)].
5. Income-tax paid.
6. Any expenditure or allowance in connection with winning of lottery, crossword
puzzles, etc. However, expenditure incurred by the assessee for the activity of owning
and maintaining race horses shall be allowed as a deduction while computing the
income from this activity.

Illustration 7: A Company, incorporated for the manufacture of steel, had not commenced
production. The plant and machinery was in the stage of erection. During the previous year
ending 31.3.2021, it paid interest on borrowings, amounting to ` 20 Lakhs. It also received
interest of ` 1.50 Lakhs on investment in Short-Term deposits of moneys not immediately
required for business. The Assessing Officer assessed the interest income under other
sources. Discuss the correctness of the assessment.
Solution: Interest income earned on deposits made out of surplus funds before
commencement of business is taxable as "Income from other sources". No part of the
interest paid on the loan borrowed shall be allowed as deduction u/s 57 as the same was
not borrowed wholly and exclusively for the purpose of earning such interest. Whole of
such interest paid shall be capitalised. Therefore, the action of the Assessing Officer is
correct.

Illustration 8: The following incomes are received by Mr. Ronaldo during financial year
2020-21
`
1. Income from agricultural land in Afganistan 5,000
2. Interest on Postal Savings Bank A/c 100
3. Director’s fees 2,000
4. Rent from sub-letting a house 26,250
5. Dividend from a foreign company 700
6. Ground rent for land in Uganda 10,000
7. Winnings from horse-race (Gross) 12,300
74 INCOME TAX

8. Interest on deposits with Industrial Finance Corporation of India 500


9. Rent payable by Mr. Ronaldo for the sub-let house 12,000
10. Other expenses on sub-let-house 1,000
11. Interest on Securities (gross) 4,000
You are required to calculate Income from other Sources of Mr. Ronaldo for the AY 2021-22.

Solution:
` ` `
1. Income from agricultural land in Afganistan 5,000
2. Director’s Fees 2,000
3. Rent-from sub-letting a house 26,250
Less: Rent payable for the sub-let house 12,000
Other Expenses 1,000 13,000 13,250
4. Dividend from a foreign company 700
5. Ground rent for land in Uganda 10,000
6. Winnings from horse-race 12,300
7. Interest on Deposits with IFCI 500
8. Interest on Securities 4,000
Income from other sources 47,750
Note: Interest on Postal Saving Bank account is exempt u/s 10.

Illustration 9: Mr. A, a dealer in shares, received the following without consideration


during the P.Y. 2020-21 from his friend Mr. B -
1. Cash gift of ` 75,000 on his anniversary, 15th April, 2020.
2. Bullion, the fair market value of which was ` 60,000, on his birthday, 19th June, 2020.
3. A plot of land at Faridabad on 1st July, 2020, the stamp value of which is ` 5 lakh on that
date. Mr. B had purchased the land in April, 2008.
Mr. A purchased from his friend C, who is also a dealer in shares, 1,000 shares of X Ltd. @
` 400 each on 19th June, 2020, the fair market value of which was ` 600 each on that date.
Mr. A sold these shares in the course of his business on 23rd June, 2020.
Further, on 1st November, 2020, Mr. A took possession of property (building) booked by
him two years back at ` 20 lakh. The stamp duty value of the property as on 1st November,
2020 is ` 32 lakh.
On 1st March, 2021, he sold the plot of land at Faridabad for ` 7 lakh.
Compute the income of Mr. A chargeable under the head “Income from other sources” and
“Capital Gains” for A.Y. 2021-22.
INCOME FROM OTHER SOURCES 75

Solution: Computation of “Income from other sources” of Mr. A for the A.Y. 2021-22
Particulars `
(1) Cash gift is taxable under section 56(2) (vii), since it exceeds ` 50,000 75,000
(2) Bullion has been included in the definition of property therefore, the
same is taxable, since the aggregate fair market value exceeds ` 50,000 60,000
(3) Stamp value of plot of land at Faridabad, received without consideration,
is taxable under section 56(2)(vii) 5,00,000
(4) Difference of ` 2 lakh in the value of shares of X Ltd. purchased from Mr.
C, a dealer in shares, is not taxable as it represents the stock-in-trade of
Mr. A. Since Mr. A is a dealer in shares and it has been mentioned that the
shares were subsequently sold in the course of his business, such shares
represent the stock-in-trade of Mr. A. --
(5) Appreciation in the value of immovable property between the time of its
booking and its actual registration is outside the scope of section
56(2)(vii). --
Income from other Sources 6,35,000

Computation of “Capital Gains” of Mr. A for the A.Y. 2021-22


Sale Consideration 7,00,000
Less: Cost of acquisition [deemed to be the stamp value charged to tax
under section 56(2)(vii) as per section 49(4)] 5,00,000
Short-term capital gains 2,00,000
Note: The resultant capital gains will be short-term capital gains since for calculating the
period of holding, the period of holding of previous owner is not to be considered.

Illustration 10: From the following particulars of Pankaj for the previous year ended 31st
March, 2021. Compute the Income under the head “Income from other Sources”:
(i) Directors Fee from a Company 10,000
(ii) Interest on Saving bank Deposits 3,000
(iii) Income from undisclosed source 12,000
(iv) Winnings from Lotteries (Net) 33,500
(v) Royalty on a book written by him 9,000
(vi) Lectures in Seminars 5,000
(vii) Interest on loan given to a relative 7,000
(viii) Interest on Debentures of a Company 3,987
(ix) Interest on Post Office Savings Bank Account 500
(x) Interest on Government Securities 2,200
(xi) Interest on Monthly Income Scheme of Post office 33,000
He paid ` 1,000 for typing the manuscript of book written by him.
76 INCOME TAX

Solution: Computation of income of Pankaj chargeable under the head “Income from other
sources” for the A.Y. 2021-22
Particulars `
1. Director’s fees 10,000
2. Interest on SB A/c [Deduction u/s 80TTA Available] 3,000
3. Income from undisclosed source 12,000
4. Royalty on books written 9,000
Less: Expenses 1,000 8,000
5. Lectures in seminars 5,000
6. Interest on loan given to a relative 7,000
7. Interest on listed debentures 3,987
8. Interest on PO SB A/c[exempt under Sec. 10 (15)] Nil
9. Interest on Government Securities 2,200
10. Interest on Post Office Monthly Income Scheme 33,000
11. Winning from lotteries 33,500 × 100/70 47,857
1,32,044
Note: Royalty income would be charged to tax under the head “Income from Other Sources”,
only if it is not chargeable to tax under the head “Profits and gains of business or profession.
This problem has been solved assuming that the same is not taxable under the head “Profits
and gains of business or profession” and hence, is chargeable to tax under the head “Income
from other sources”.

Illustration 11: Compute the income under the head other sources and TDS deductible in
the case of following investments made by Abhi.
1. Purchase of 12% IDBI debentures of ` 1,00,000 on 1-10-2020 directly from IDBI. Due
date of interest is 31st of March every year.
2. Purchased 100, 12% IDBI debentures on 1-3-2021 from market @ ` 1,050 per
debenture. Face value of debenture is ` 1,000. Due date of interest is 31st of March
every year. IDBI had issued these debenures in 2013.
3. Purchased 100, 14% debentures of A Ltd. listed on stock exchange from a broker at
` 100 each on 30-11-2020. Due dates of interest are 30th of June and 31st of December
every year.
4. Purchased 100, 12% debentures of B Ltd. @ ` 100 each by subscribing to the company
directly. Date of issue was 31-10-2020. Interest due on 30th June and 31st of December
of every year.
5. Purchased 1,000 shares of face value of ` 10 of Y Ltd. @ 60 per share from market on
15-6-2020. The company declared a dividend @ 20% on 30-9-2020.
INCOME FROM OTHER SOURCES 77

Solution: Income under the head ‘Income from Other Sources’ for the AY 2021-22
Income TDS
deductible
1. 12% IDBI debentures purchased directly from IDBI for 6
months 6,000 600
2. 12% IDBI debentures bought from market 12,000 1,200
3. 14% debentures of A Ltd. for 6 months 700 --
4. 12% debentures of B Ltd. for 2 months 200 --
5. Dividend from Y Ltd. 200 --
Income under the head ‘Income from other sources’ 19,100 1,800

Illustration 12: Compute gross interest/dividend and net interest/dividend on securities


and shares in the following cases:
1. 10% Bonds of Industrial Development Bank of India of ` 4,60,000.
2. 10% Debentures of ABC Ltd. listed on Ahmedabad Stock Exchange purchased at ` 93
(Face Value ` 100) ` 1,30,200.
3. Interest received from debentures issued by Yash Ltd. listed on Stock Exchange –
` 31,500.
4. Interest received from debenture issued by Fransco Ltd. Company not listed on Stock
Exchange – ` 21,600.
Solution:
Gross Net
Interest / Rate of TDS Interest
Dividend TDS Amount after TDS
` `
1. 10% Bonds of IDBI 46,000 10% 4,600 41,400
2. 10% Debenture of ABC Ltd. (Listed) 14,000 10% 1,400 12,600
3. Interest received on debenture of Omi
Ltd. (listed) ` 31,500 × 100/90 35,000 10% 3,500 31,500
4. Interest received on debenturs (not
listed) ` 21,600 × 100/90 24,000 10% 2,400 21,600

Illustration 13: Find out the income chargeable to tax in the given case for the assessment
years 2021-22 and 2022-23. On May 12, 2020, X borrows ` 1,00,000 at the rate of 11% p.a.
from a bank to invest in public issue of 12% debentures of A Ltd. A Ltd. allots debentures on
May 18, 2020 (as per terms of allotment, interest is payable every year on December 31.
However, the first interest would be for the period commencing May 18, 2020 to December
31, 2020).
78 INCOME TAX

Solution: Computation of Income for the AY 2021-22


`
Amount of interest income [see Note 2] 7,496
Less: Interest on borrowed capital [see Note 1] 9,764
Income from other sources -2,268
Computation of Income for the AY 2022-23
`
Amount of interest income 12,000
Less: Interest on borrowed capital 11,000
Income from other sources 1,000
Working Note:
1. Interest to be paid for period commencing from May 12, 2020 to March 31, 2021.
Days = 324
324
Interest = 1,00,000 × 11% × = 9,764
365
2. Interest Received for 228 days
228
Interest = 1,00,000 × 12% × = 7,496
365

Illustration 14: Rahul received the following income as interest during the PY 2020-21.
(i) ` 3,482 as interest on debentures issued by local authority.
(ii) ` 7,164 as interest on debentures of Meghdoot Ltd. (not listed at any recognized
stock exchange).
(iii) ` 12,776 as interest on debentures of X Ltd. (Listed on Delhi stock exchange)
(iv) ` 6,000 as interest on Government Securities on 5.6.20
(v) ` 3,852 as interest on tax free debentures of Gunjan Ltd. (Not listed)
Determine income from other sources for the AY 2021-22 assuming bank charges 2% on
account collected.
Solution: Income from other Sources (for assessment year 2021-22)
`
(i) Interest on debentures issued by local authority (Exempt) --
(ii) Interest on debentures of Meghdoot Ltd. (7,164 × 100/90) 7,960
(iii) Interest on debenture of X Ltd. (12,776 × 100/90) 14,196
(iv) Interest on Government securities 6,000
(v) Interest on tax free debentures of Gunjan Ltd. (3,852 × 100/90) 4,280
Total income 32,436
Less: Expenses on collection (2% on ` 29,792 i.e. 7,164 + 12,776 + 6,000 + 3,852) 596
31,840
INCOME FROM OTHER SOURCES 79

Note: Although the collection of charges on ` 3,482 will also be paid but no deduction shall
be allowed of such charges as its income is exempt.

Illustration 15: Compute income under the head other sources from the following
particulars:
11. 6% Tax-free Railway’s Bonds of ` 2,80,000.
12. 9% Bonds of Industrial Development Bank of India of ` 3,60,000.
13. 12% Debentures of ABC Ltd. listed on Kolkata Stock Exchange purchased at ` 96 (Face
Value ` 100) ` 1,92,000.
14. Interest received from debentures issued by X Ltd. listed on Stock Exchange ` 59,268.
15. 6% Tax-free Relief Bonds of Reserve Bank of India ` 4,00,000.
16. Interest received from debenture issued by A Ltd. Company not listed on Stock
Exchange ` 9,552
17. Dividend received from A Ltd. on 27-6-2020 – ` 17,900.
18. Dividend received on 5-8-2020 on shares of B Ltd. @ 50% on 1,000 shares of ` 10 each,
which were purchased at ` 40 share.
19. 10% Dividend on preference shares of ` 10 each amounting to ` 2,50,000 paid on 31-3-
2021.
Solution:
1. 6% Tax-free Railway’s Bonds (Exempt) --
2. 9% Bonds of Industrial Development Bank of India (3,60,000 × 9%) 32,400
3. 12% Debentures of ABC Ltd. (1,92,000 × 100/96 × 12%) 24,000
4. Interest received on debentures issued by X Ltd. (` 59,268 × 100/90) 65,853
5. 6% Tax-free Relief Bonds of Reserve Bank of India (Exempt) --
6. Interest received on debentures issued by a Ltd. company not listed on
Stock Exchange – (` 9,552 × 100/90) 10,613
7. Dividend received from A Ltd. 17,900
8. Dividend on shares of B Ltd. 5,000
9. 10% Dividend on preference Share 25,000
1,80,766
80 INCOME TAX

MCQ

1. Income from letting of machinery, plant and furniture is –


(a) Always chargeable to tax under the head “profits and gains of buisness and
profession”.
(b) Always chargeable to tax under head “Income from other sources”
(c) Chargeable under the head “Income from other sources” only if not chargeable
under the head “Profits and gains of business and profession”.
(d) Chargeable to tax under the head “Income from house property”

2. In respect of winnings from lottery, crossword puzzle or race including horse race or
card game etc.
(a) No deduction under Chapter VI-A is allowed and basic exemption limit cannot
be exhausted.
(b) No deduction under Chapter VI-A is allowed but unexhausted basic exemption
can be exhausted.
(c) Both deduction under Chapter VI-A and basic exemption are allowed.
(d) Deduction under Chapter VI-A is allowed but basic exemption limit cannot be
exhausted.

3. The deduction allowable in respect of family pension taxable under “Income from
other sources” is
(a) 33-1/3% of the pension
(b) 30% of the pension or ` 15,000, whichever is less
(c) 33-1/3% of the pension or ` 15,000, whichever is less
(d) 30% of the pension

4. Deemed dividend under section 2(22)(e) is chargeable to tax –


(a) On the basis of method of accounting regularly employed by the assessee
(b) On the basis of mercantile system of accounting only
(c) On payment basis as prescribed under section 8 of the Income-tax Act, 1961.
(d) On cash basis only.

5. Ganesh received ` 60,000 from his friend on the occasion of his birthday.
(a) The entire amount of ` 60,000 is taxble.
(b) ` 50,000 is taxable
(c) The entire amaount is exempt
(d) ` 10,000 is taxable
INCOME FROM OTHER SOURCES 81

6. Mr. X aged, 61 years, received dividend of ` 12,00,000 from a domestic company in


P.Y. 2020-21. Tax chargeable under section 115BBDA is @ 10% on –
(a) The entire amount of ` 12,00,000
(b) ` 2,00,000
(c) Nil
(d) ` 9,00,000

7. In respect of dividend received from domestic companies in excess of ` 10,00,000 by


an individual –
(a) No deduction under Chapter VI-A is allowed but loss under other heads can be
set-off against such income.
(b) No deduction under Chapter VI-A is allowed and no loss can be set-off against
such income.
(c) Both deduction under Chapter VI-A and set-off of losses against such income are
allowed.
(d) Deduction under Chatper VI-A is allowed but set-off of losses under other heads
against such income is not allowed.

8. Mr. Y has received a sum of ` 51,000 on 24.10.2020 from relatives on the accasion of
his marriage.
(a) Entire ` 51,000 is chargeable to tax.
(b) Only ` 1,000 is chargeable to tax
(c) Entire ` 51,000 is exempt from tax
(d) Only 50% i.e. ` 25,500 is chargeable to tax

9. Mr. Mayank has received a sum of ` 75,000 on 24.10.2020 from his friend on the
occasion of his marriage anniversar
(a) Entire ` 75,000 is chargeable to tax.
(b) Entire ` 75,000 is exempt from tax
(c) Only ` 25,000 is chargeable to tax
(d) Only 50% i.e, ` 37,500 is chargeable ta tax

10. The deduction in respect of interest on enhanced compensation of ` 1,50,000


received during the previous year 2020-21, would be –
(a) ` 1,50,000, being 100% of ` 1,50,000
(b) ` 75,000, being 50% of ` 1,50,000
(c) ` 45,000, being 30% of ` 1,50,000
(d) Nil
82 INCOME TAX

Unsolved Exercise

Q1. Girdhar receives the following gifts during the previous year 2020-21
1. He gets a cash gift of ` 25,000 from his friend A.
2. He gets another gift of ` 5,000 by account payee cheque from his friend A.
3. He gets bullion of ` 46,000 without consideration from C, who is cousin of his
father.
4. He gets Silver coin of ` 5,800 from D, who is elder brother of his grandfather.
5. He gets a drawing of ` 45,000 from his grandmother.
6. On the occasion of marriage of Girdhar, he gets ` 2,80,000 as gift (out of which
` 1,80,000 is received from different relatives of Girdhar and Mrs. Girdhar and
remaining amount is received from friends of Girdhar and Mrs. Girdhar).
7. A computer received from his employer (it was purchased for ` 42,000 by the
employer on May 4, 2020 and given as gift on August 22, 2020).
8. Girdhar purchases a house property from his friend for ` 65,000 (stamp duty value
of the property is ` 6 lakh).
9. He gets ` 60,000 from a notified public charitable institution.
10. Girdhar receives ` 2,40,000 under will of a person known to him.
11. He gets a gift of ` 20,000 in cash from his friend.
12. He purchases a work of art for ` 72,000 from an exhibition in Chennai (the fair
market value of the work of art on the date of purchase is ` 2,00,000). Organizer of
exhibition is not a registered dealer.
13. He purchase a commercial property for ` 8,00,000 (stamp duty value is
` 9,60,000).
14. He gets a birthday gift by cheque of ` 11,000 from his friend.
Compute the amount chargeable to tax in the hands of Girdhar under the head
“Income from other sources” for the assessment year 2021-22.
[Ans. ` 9,35,800]

Q2. Jayant receives the following gifts during the previous year 2020-21.
July 3, 2020 Purchase of a house property from a friend for ` 8,00,000 (Stamp
duty value is ` 32,00,000)
October 30, 2020 Cash gift of ` 1,51,000 from a friend on marriage anniversary
December 8, 2020 Received house property as gift from Mrs. Jayant (stamp duty value
is ` 80,00,000)
December 29, 2020 Cash gift of ` 60,000 from an American friend
Feb 10, 2021 Cash gift of ` 20,000 from a colleague
INCOME FROM OTHER SOURCES 83

March 16, 2021 Purchase of a second hand car for ` 2,00,000 (fair market value is `
8,50,000)
March 20, 2021 Purchase of a painting from a shop for ` 80,000 (fair market value
is ` 35,000)
Find out the amount chargeable to tax under the head “Income from other sources” for
the assessment year 2021-22.
[Ans. ` 26,31,000]

Q3. Shri Joseph is a businessmen dealing in cloth. On 1-4-2020 the position of his
investments was as under:
(1) ` 38,000, 9% (Tax free) Debentures of Birla Jute Mills Ltd. (listed).
(2) ` 20,000, 12% Debenture of G Ltd,
(3) Interest on National Savings Certificate (VIII issue) due ` 3,860
(4) ` 22,000, 7 ½ % Government of India Loan 2003.
(5) ` 35,000, 7% Rajasthan Investment Loan
(6) ` 18,000, 7% Capital Investment Bonds.
The due dates of interest of all the above securities are 1st May and 1st November.
Calculate income from other sources for the assessment year 2021-22.
[Ans. ` 14,160]

Q4. Santosh holds the following securities on April 1, 2020:


` 49,000 7% securities of UP Government (date of payment of interest:
December 15 every year)
` 68,000 9% securities of Bihar Government (date of payment of interest:
March 31 every year)
` 88,000 6% debentures of Elite Ltd. (data of payment of interest: July 15 every
year).
On August 3, 2020 he sells ` 30,000 6% debentures of Elite Ltd. and invests the sale
proceeds in listed debentures of Concepts Ltd. During the previous year 2020-21, he
receives ` 5,747 (net) as interest on listed debentures of Concepts Ltd. Interest on 9%
Bihar Government securities, which has accrued on March 31, 2021, is received by
Santosh on April 15, 2021.
Calculate the income under head other sources of Santosh for the assessment year
2021-22. Santosh maintains books of account on the basis of mercantile system of
accounting.
[Ans. ` 21,216]
84 INCOME TAX

Q5. During the previous year 2020-21 Shri Gurudar Singh had following securities.
(i) ` 33,000, 12% tax-free debentures of S Ltd. (Listed on stock exchange).
(ii) ` 20,000, 9% relief bonds.
(iii) ` 66,000, 10% tax free debentures of Malwa Textile.
(iv) ` 20,500, 6% tax-free bonds issued by notified public sector company.
The interest is paid on 31st December annually. He paid 2% commission to his bank for
collection of interest.
Calculate his taxable income for the assessment year 2021-22.
[Ans. ` 11,522]

Q6. Ramdin a resident individual submits the following particulars of his income for the
year ended 31-3-2021.
(i) Royalty from a coal mine ` 30,000.
(ii) Salary as member of Parliament ` 26,000
(iii) His residential house has been taken on a rent of ` 10,000 p.a. half of which he has
sublet at ` 1,200 p.m.
(iv) Dividend received from a cooperative society ` 8,000
(v) Agricultural income in Pakistan ` 25,000
(vi) He has incurred the following expenses:
(a) Paid collection charges ` 100 for collecting dividends
(b) ` 3,000 spent for earning and collecting royalty income.
Compute Ramdin’s income from other sources for the assessment year 2021-22.
[Ans. ` 95,300]
CLUBBING OF
3
INCOME
Income transferred without transfer of assets [Sec. 60]
If a person transfers income to another person, without transfer of the asset from which the
income arises, then such income shall be taxable in the hands of transferor.

Revocable transfer of assets [Sec. 61]


If there is revocable transfer of an asset by one person to another, then Income from such
assets shall be taxable in the hands of transferor.

Income from assets transferred to spouse [Sec. 64(1) (iv)]


When any assets other than house property is transferred by an individual to his / her
spouse directly or indirectly, any income from such assets shall be deemed to be the income
of transferor.
However clubbing shall not be done if:
(i) Asset is sold for adequate consideration, or
(ii) If relationship of husband and wife does not exist either at the time of transfer or at
the time of accrual of income, or
(iii) Transfer is under an agreement to live apart.
If house property is transferred by an individual to the spouse otherwise than for adequate
consideration, then Sec. 64(1)(iv) shall not apply but Sec. 27 shall apply.

Income from assets transferred to son’s wife [Sec. 64(1)(vi)]


If a individual transfers any asset to his daughter-in-law, without adequate consideration,
income from the asset will be included in the total income of transferor. The relationship of
father-in-law (or mother-in-law) and daughter in law should subsist both at the time of
transfer of asset and at the time of accrual of income.

If transferred asset is invested by spouse/son’s wife in any business following amount shall
be included:
Amount invested out of asset transferred on the first day of the p/y
Income/ Interest from business x Total investment of transferee on the the first day of the p/y
86 INCOME TAX

When an individual is assessable in respect of income from assets transferred to a


person for the benefit of spouse [Sec. 64(1) (vii)]
1. The taxpayer is an individual.
2. He/she has transferred an asset.
3. The transfer may be direct or indirect.
4. The asset is transferred to a person or an association of persons.
5. It is transferred for the immediate or deferred benefit of his/her spouse.
6. The transfer is without adequate consideration.

When an individual is assessable in respect of income from assets transferred to a


person for the benefit of son’s wife [Sec. 64(1) (viii)]
1. The taxpayer is an individual.
2. He/ she has transferred an asset.
3. The asset is transferred to any person or an association of persons.
4. Transfer may be direct or indirect.
5. The asset is transferred for the immediate or deferred benefit of his/her son’s wife.
6. The asset is transferred otherwise than for adequate consideration.

Income from self-acquired property converted into joint-family property [Sec. 64(2)]
If an individual, who is a member of the HUF, converts his self-acquired property as the
property of the HUF then income derived by HUF from such property shall be included in
the income of transferor. Advance/Loan by an Individual to HUF is not throwing of assets in
common stock of the family.

Clubbing of income after partition


If the converted property is subsequently transferred among the members of family, the
income derived from such converted property received by spouse & minor children
(including minor married daughter) of transferor will be included in the income of
transferor. Advance/loan by an Individual to HUF in not throwing of asset in common stock
of the family.

Remuneration of Spouse [Sec. 64(1) (ii)]


An individual is chargeable to tax in respect of any salary, commission, fees or any other
remuneration received by the spouse from a concern in which the individual has substantial
interest.
(i) But that portion of salary etc. of spouse which is due to application of technical or
professional knowledge or experience shall not be clubbed.
(ii) If husband and wife both have substantial interest in the concern and
• both are receiving remuneration because of interest in the concern
CLUBBING OF INCOME 87

• Then the remuneration of both shall be clubbed in the hands of that spouse
whose total income is greater, before clubbing such income.
(iii) Substantial Interest
(a) For company: If individual along with relatives hold not less than 20% equity
shares beneficially.
(b) For others: If individual along with relatives is entitled to at least 20% of
profits.
Note:
1. Relative means the husband, wife, brother or sister or any lineal ascendant or
descendant of the individual.
2. Any income arising from the accumulated income of such property is not includible in
the income of the transferor. Income on original transfer only has to be clubbed.
3. Under the above provisions, if income of one person is to be clubbed in the hands of
another person, then in the case of loss, the loss shall also be clubbed.
4. Under the above provisions, if income of one person is clubbed in the hands of
transferor, then tax on income from such assets can also be demanded from the
transferee.

Income of minor child [Sec. 64(1A)]


(i) All incomes that accrue to minor child (including minor married daughter) will be
included in the total income of that parent whose total income is greater (before
including income of minor child).
(ii) If marriage of parents does not exist, then included in income of that parent who
maintains the child.
(iii) Where any such income is once included in the total income of either parent, any
such income of any succeeding year shall be included in the income of other parent
only if AO is satisfied.
(iv) In the following cases income of minor shall not be clubbed:
(a) Child is suffering from any disability of the nature specified in Section 80U, like
physically disabled, totally blind etc.
(b) Income of child on account of manual work or activity involving skill, talent or
specialized knowledge etc.
(v) Any income to minor on investing income of manual work or special talent shall be
clubbed.
(vi) If income of child is so included, the parent shall be entitled to an exemption of
maximum ` 1,500 in respect of each minor child.
(vii) If the minor child becomes major during the PY, then the incomes which have
accrued to him till the date he attains majority shall be clubbed u/s 64(1A).
88 INCOME TAX

Case Study

Assets Assets

Mr. X Income
Mrs. X

Revenue Both Incomes


will be clubbed
Income Re-Invested Capital Gain

Income

Will not be clubbed


CLUBBING OF INCOME 89

Income 6 lacs Income 8 lacs

Mr. X Mrs. X

Minor child Earns ` 3 lacs from his personal skill.

Income ` 3,00,000 Will not be clubbed

` 3,00,000 is invested in FD @ 12%

Intt. Income ` 36,000

Will be clubbed with Income of Mrs. X


90 INCOME TAX

Cash 10 lacs

Mr. X Mrs. X

Invested out of ` 10 lacs in

Share Debenture Jewellery

Dividend CG Intt. CG CG

Upto ` 10 lac Exceeding ` 10 lac Will be clubbed

Exempt
CLUBBING OF INCOME 91

Cash 10 lacs 10 lacs

Invested in her business (Firm)

Income

Share of Profit – Exempt

Salary, Bonus, Commission – will not be clubbed

Interest on Capital – will be Clubbed


92 INCOME TAX

Cash 10 lacs

Mr. X
Mrs. X

10 Lacs is gifted

2019 2020 2021

Profit ` 4 lacs Profit ` 6 lacs

Opening Capital 20 lacs


own fund of Mrs. X

4 lacs Clubbing Provision will


For Year 2020-21= x Nil = Nil
20 lacs not apply

6 lacs Will be clubbed with the


For Year 2021-2= x 10 lac = 2,00,000
30 lacs income of Mr. X
CLUBBING OF INCOME 93

Illustration 1: Vishal holds 20% equity share capital in Y Ltd. Mrs. Vishal is employed by Y
Ltd. (salary being ` 40,000 per month) as general manager (finance). She does not have any
professional qualification to justify remuneration. Ascertain in whose hands salary income
is chargeable to tax. Does it make any difference if Mrs. Vishal was employed by Y Ltd. even
prior to her marriage?
Solution: In this case, Vishal has substantial interest in Y Ltd. where Mrs. Vishal is
employed. Mrs. Vishal does not have any professional qualification to justify the
remuneration of ` 40,000 per month. Her salary income of ` 4,80,000 (i.e., ` 40,000 × 12)
will be taxable in the hands of Vishal. It does not make any difference even if Mrs. Vishal was
employed by Y Ltd. prior to her marriage.

Illustration 2: Dinesh (age: 35 years) gifts ` 10 lakh to Mrs. Dinesh (age: 31 years). She
deposits the same in a bank @ 8% per annum. Yogesh is minor child of Dinesh and Mrs.
Dinesh. Yogesh has a bank deposit of ` 70,000 (rate of interest 8.25%) which was gifted to
him by his grandfather. Other income of Dinesh and Mrs. Dinesh is as follows – Dinesh:
` 3,00,000 [salary: ` 2,10,000, bank interest: ` 90,000], Mrs. Dinesh: ` 2,00,000 (interest as
company deposits). Out of interest income, Mrs. Dinesh deposits ` 1,000 in Public Provident
Fund. Dinesh’s contribution to the recognized provident fund is ` 40,000.
Find out the income chargeable to tax for the assessment year 2021-22.
Solution:
Dinesh Mrs. Yogesh
Dinesh
Salary 2,10,000 -- --
Income from other sources
- Bank interest of Mrs. Dinesh (8% × ` 10 lakh) 80,000 -- --
- Bank interest of Yogesh (8.25% of ` 70,000 (-) ` 1,500) 4,275 -- --
- Bank interest of Dinesh 90,000 -- --
- Interest of company deposit -- 2,00,000 --
Gross total income 3,84,275 2,00,000
Less: Deduction under section 80C 40,000 1,000 --
Net income (rounded off) 3,44,280 1,99,000
Tax Nil Nil --

Illustration 3: Sanjay and Mrs. Sanjay hold 20% and 30% equity shares in C Ltd.
respectively. They are also employed from April 1, 2020 in Mumbai branch of C Ltd.
(monthly salary being ` 80,000 and ` 40,000 respectively) without any technical /
professional qualification.
Other incomes of Sanjay and Mrs. Sanjay are ` 1,60,000 and ` 1,90,000 respectively. Find
out the net income of Sanjay and Mrs. Sanjay for the assessment year 2021-22.
94 INCOME TAX

Solution: Sanjay and Mrs. Sanjay have substantial interest in C Ltd. which employs them
without any professional/technical qualification. In this case, the salary of husband and wife
shall be included in the income of Mrs. Sanjay whose other income is higher as explained
under.
Sanjay Mrs. Sanjay
` `
Salary of Sanjay (` 80,000 × 12) -- 9,60,000
Salary of Mrs. Sanjay (` 40,000 × 12) -- 4,80,000
Other income 1,60,000 1,90,000
Net income 1,60,000 16,30,000

Illustration 4: Amit and Bolu are minor sons of Nanku and Mrs. Nanku. Business income of
Nanku is ` 3,40,000. Income from house property of Mrs. Nanku is ` 1,90,000. Income of
Amit and Bolu from stage acting is ` 60,000 and ` 70,000 respectively. Interest on company
deposits of Amit and Bolu (deposit was made out of income from acting) is ` 30,000 and
` 1,000, respectively. Amit and Bolu have received the following birthday gifts – on May 20,
2020, gift received by Bolu from his grandfather: ` 80,000; on September 14, 2020, gift
received by Amit - ` 60,000 from Nanku’s friend and ` 35,000 from a relative. Find out the
income of Nanku, Mrs. Nanku, Amit and Bolu for the assessment year 2021-22.

Solution:
Nanku Mrs. Amit Bolu
` Nanku ` `
`
Income from house property -- 1,90,000 -- --
Business income 3,40,000 -- -- --
Income from stage acting -- -- 60,000 70,000
Income from other sources
- Gift received by Bolu on May 20, 2020 from
grandfather (gift from a relative is not taxable) -- -- -- --
- Gift received by Amit on September 14, 2020
from Nanku’s friend (to be clubbed in the
hands of Nanku after giving exemption of
` 1,500)
- Gift received by Amit on September 14, 2020 58,500 -- -- --
from relatives (gift from a relative is not
taxable)
- Interest from company deposit received by -- -- -- --
Amit (to be clubbed in the hands of Nanku)
30,000 -- -- --
CLUBBING OF INCOME 95

- Interest from company deposit received by


Amit (to be clubbed in the hands of Nanku
after giving exemption of ` 1,500, amount to
be clubbed is ` 1,000 – ` 1,000) Nil -- -- --
Net income 4,28,500 1,90,000 60,000 70,000

Illustration 5: X, an individual, is engaged in the business of money-lending. On April 1,


2020, he gave advance of ` 10,00,000 to his HUF at the market rate of interest of 12% per
annum. During the previous year 2020-21, HUF earns ` 4,00,000 as profit on the money
advanced by X (before paying interest). Determine:
1. Is the amount of net income of HUF (i.e., ` 4,00,000 minus 12% of ` 10,00,000)
includible in the income of X under section 64(2)?
2. Does it make any difference if X is not engaged in the business of money-lending?
3. Does it make any change in the applicability of Section 64(2), if money is advanced at
the rate of 4% whereas the market rate of interest is 12%?

Solution: By advancing loan to the HUF, it cannot ordinarily be said that the lender has
transferred any asset to HUF without adequate consideration, if the money is advanced at
the market rate of interest.
1. Section 64(2) is not applicable, as it is not the case of transfer or conversion of separate
property into HUF’s property. As such, no part of HUF’s income can be clubbed with X’s
income.
2. If the money is advanced at the market rate of interest, it does not make any difference
whether X carries on money-lending business or not. In the given case, Section 64(2) is,
therefore, not attracted even if X does not carry on money-lending business.
3. If money is advanced at lower than market rate of interest, Section 64(2) cannot be
invoked, as the advance/loan does not amount to throwing of assets in common stock
of the family or transfer of assets to the family.

Illustration 6: Mr. Dhaval has an income from salary of ` 3,50,000 and his minor children’s
income are as under:
`
Minor daughter has earned the following income:
From a TV show 1,50,000
From interest on FD with a bank (deposited by Mr. Dhaval from his income) 5,000
Minor son has earned the following income:
From the sale of a own painting 1,10,000
From interest on FD with a bank (deposited by Mr. Dhaval from his income) 1,000
96 INCOME TAX

Solution: Computation of Gross Total Income of Mr. Dhaval


Amount Amount
(`) (`)
Income from Salary 3,50,000
Income from other sources:
Minor Daughter’s income
Income from T.V. show (See Note below) Nil
Less: Exempt under section 10(32) 5,000
Minor son’s income 1,500 3,500
Income from sale of self made painting (See Note 1) Nil
Interest income from FD with a Bank 1,000
Less: Exempt under section 10(32) 1,000 Nil
Gross Total Income 3,53,500
Note: The income derived by the minor from manual work or from any activity involving
exercise of his skill, talent or specialized knowledge or experience will not be included in
the income of his parent. Hence, in the given case `1,50,000 being the income of the minor
daughter from TV show and `1,10,000 being the income of minor son shall not be clubbed
in the hands of Mr. Dhaval.

Illustration 7: X submits the following information for the year ending March 31, 2021:
1. Son of X (date of birth: August 31, 2001) has a fixed deposit of ` 40,00,000 in PNB (rate
of interest 7%).
2. Minor daughter of X owns a business. For the previous year ending March 31, 2021, her
income from business is ` (-) 70,000.
3. On October 4, 2013, X gifted ` 5,00,000 to Mrs. X. This amount (along with her own
funds) is used in setting up a sole proprietary business by Mrs. X. On April 1, 2020, her
total investment in the business is ` 11,00,000 and for the year ending March 31, 2021
income from the business is ` 2,78,000.
4. Salary income of X is ` 11,45,000.
5. X holds 10% shares in A Ltd. (a closely held manufacturing company). On June 10,
2020, X transfers these shares by way of gift to Mrs. X. Mrs. X takes a loan of ` 2,00,000
on January 1, 2021 from the company. Accumulated profit of the company on this date
is ` 40,00,000.
6. X is entitled for a deduction of ` 90,000 under section 80CCC.
7. Income of Mrs. X for the assessment year 2021-22 is ` 3,00,000.
Determine the net income and tax liability of X for the assessment year 2021-22.
Solution:
1. Son of X becomes major on August 31, 2020. Interest income from April 1, 2020 to
August 31, 2020 is taxable in the hands of X. It comes to ` 1,16,667 (i.e., ` 40,00,000
CLUBBING OF INCOME 97

×0.07 × 5 ÷ 12). Interest income from August 31, 2020 onwards will be taxable as
income of X’s son.
2. Income (or loss) of minor child is taxable in the hands of parents. Consequently, ` (-)
70,000 will be clubbed in the hands of X.
3. On April 1, 2020, total investment of Mrs. X in her business is ` 11,00,000. Out of
` 11,00,000, ` 5,00,000 was gifted by X in 2013-14. Proportionate income from
business will be taxable in the hands of X. It comes to ` 1,26,364 (i.e., ` 2,78,000 ×
` 5,00,000 ÷ ` 11,00,000).
4. If a closely held company gives a loan or advance to a shareholder (who holds 10% of
equity share in the company, such loan or advance (to the extent it does not exceed
accumulated profit of the company) is treated as deemed dividend under section
2(22)(e) in the hands of the shareholder. In this case, ` 2,00,000 will be deemed as
dividend in the hands of Mrs. X.
Computation of income and tax liability
`
Salary 11,45,000
Business income
Χ Business of minor daughter (-)70,000
Χ Business of Mrs. X 1,26,364
Income from other sources
Deemed Dividend u/s 2(22)(e) 2,00,000
Χ Interest income of son (` 1,16,667 – exemption of ` 1,500) 1,15,167
Gross total income 15,16,531
Less: Deduction under section 80CCC 90,000
Net income (rounded off) 14,26,530
Tax on net income
Income-tax 2,40,459
Add: Health &Education cess 9,618
Tax liability (rounded off) 2,50,077

Illustration 8: X is a salaried employee. For the year ending March 31, 2021, he submits the
following information:
1. On May 3, 2020, X purchases 2,000 shares in A Ltd. at the rate of ` 60 per share. On May
5, 2020, these share are gifted to Mrs. X. A Ltd. allots 1,000 bonus shares to Mrs. X on
January 30, 2020. On March 1, 2021, Mrs. X transfers 3,000 shares in A Ltd. for ` 400
per share to B outside stock exchange. On March 1, 2021, the lowest quotation of shares
in A Ltd. at Bombay Stock Exchange is ` 700 per share.
2. On June 1, 2020, X purchases a house property for ` 18,00,000 (however, he pays stamp
duty at the rate of 10% on ` 30,00,000). The house property is gifted to Mrs. X on
98 INCOME TAX

February 1, 2021. Mrs. X transfers this property to C on March 31, 2021 for ` 40,00,000.
Stamp duty value is ` 50,00,000.
Discuss the tax consequences of these transactions and calculate income from these
transactions in the hands of X, Mrs. X, B and C.
Solution:
1. 2,000 shares in A Ltd. are purchased by X. Later on these shares are gifted to Mrs. X.
Mrs. X gets bonus shares. Original as well as bonus shares are transferred by Mrs. X.
Capital gain which arises on transfer of original shares will be included in the income of
X, as shares were transferred without consideration to Mrs. X by X. Capital gain which
arises on transfer of bonus shares will be taxable as income of Mrs. X, as bonus shares
are not transferred by X. Shares are purchased by B outside stock exchange at a price
which is lower than the lowest market quotation on the date of transaction. The
difference between market value and purchase price will be taxable as income from
other sources under section 56(2)(vii) in the hands of B.
2. House property is transferred by Mrs. X. However, capital gain will be taxable in the
hands of X. The income will be calculated as under:
X Mrs. X B C
` ` ` `
Short-term capital gain on transfer of
original shares 6,80,000 -- -- --
[(` 400 - ` 60) × 2,000]
Short-term capital gain on transfer of
bonus shares [(` 400 - ` 0) × 1,000] -- 4,00,000 -- --
Short-term capital gain on transfer of
house [` 50,00,000 – (` 30,00,000 +
stamp duty of 10% of ` 30,00,000)] 17,00,000 -- -- --
Purchase of 3,000 shares for less than fair
market value [(` 700 -` 400) × 3,000] -- -- 9,00,000 --
Purchase of house property for
inadequate consideration 12,00,000 -- -- 10,00,000
Total 35,80,000 4,00,000 9,00,000 10,00,000

Illustration 9: X gifted debentures of a company to Miss Y on March 15, 2018. Miss Y


married X’s son Z on February 1, 2019. Interest on debenture for the previous year 2020-21
is ` 3,00,000. It is added by the Assessing Officer in the hands of X under the provisions of
Section 64(1)(vi). Is this inclusion justified in law.
Solution: According to Section 64(1)(vi), if an individual, directly or indirectly, transfers
assets, without adequate consideration to son’s wife, income arising from such assets will
be included in the total income of the transferor. However, the relationship of the father-in-
law and daughter-in-law should subsist both at the time of transfer of asset and at the time
CLUBBING OF INCOME 99

of accrual of income. It means transfer of asset before son’s marriage by an individual to his
prospective daughter-in-law is outside the scope of clubbing even if income is accrued after
son’s marriage.
In the present problem, in view of the aforesaid provision, the inclusion in the income of X
by the Assessing Officer is unjustified.

Illustration 10: Discuss the following:


1. X transferred agricultural lands permanently to Y with a condition that 25% of the
income from the land should be handed over to him. Y earned an income of ` 40,000
from the transferred property.
2. Mrs. X holds 4,000 out of 10,000 equity shares of ABC Ltd., a company in which X,
holding a BA degree, is working as finance manager on a salary of ` 50,000 per month.
3. X transferred a sum of ` 2 lakh to his wife in 1989 and Mrs. X constructed a property
out of this amount. She has since been using the property for her own residence. The
transfer deed stipulates that in the case of X and his wife agreeing to live apart, the
property will belong to her absolutely.
Solution:
1. According to Section 61, all income arising to any person by virtue of transfer of assets
is deemed to be the income of the transferor and chargeable to tax in hands if the
transfer contains any provision for the re-transfer, directly or indirectly of the whole or
any part of the income or assets to the transferor. However, it may be noted that
agricultural income is exempt from tax under section 10(1).
2. According to Section 64(1)(ii), an individual is chargeable to tax in respect of any
remuneration received by the spouse from a concern in which the individual has
substantial interest. An individual is deemed to have substantial interest if he
(individually or along with his relatives) beneficially holds equity shares carrying not
less than 20% voting power in the case of a company. However, remuneration which is
solely attributable to the application of technical or professional knowledge and
experience of the spouse will not be clubbed. In the present problem, in view of the
aforesaid provision, ` 50,000 per month being salary of X will be included in the total
income of Mrs. X by virtue of Section 64(1)(ii). She holds 40% equity share capital in
ABC Ltd. It may be noted that X does not possess any professional or technical
qualification to justify his position as a finance manager in ABC Ltd.
3. According to Section 64(1)(iv), where an asset (other than house property) is
transferred by an individual to his or her spouse, directly or indirectly, otherwise than
for adequate consideration or in connection with an agreement to live apart, any
income from such asset will be deemed to be the income of the transferor. In the
present problem, in view of the aforesaid provision, the income from property shall be
included in the total income of X by virtue of Section 64(1)(iv). However, according to
Section 23 annual value of a house property which is self occupied is taken as nil and if
100 INCOME TAX

there is no interest deduction under section 24, then nothing will be included in the
total income of X.
The provisions of Section 64(1)(iv) are applicable is this problem since what is
transferred is cash and not a property and consequently, Section 27(i) is not applicable.
Thus, though Mrs. X will be considered as the owner of the house property yet the
income from the property will be included in the total income of X. The provision in the
transfer deed that his property would belong to Mrs. X in case there is an agreement to
live apart does not make any difference.

Illustration 11: Compute the total income of Mr. A & Mrs. A from the following information.
`
(a) Salary income (computed) of Mrs. A 2,30,000
(b) Income from profession of Mr. A 3,90,000
(c) Income of minor son B from company deposit 15,000
(d) Income of minor daughter C from special talent 32,000
(e) Interest from bank received by C on deposit made out of her special talent 3,000
(f) Gift received by C on 30.9.2020 from friend of Mrs. A 2,500

Solution: As per the provisions of Section 64(1A) of the Income-tax Act, 1961, all the
income of minor child has to be clubbed in the hands of that parent whose total income
(excluding the income of the minor) is greater. The income of Mr. A is ` 3,90,000 and
income of Mrs. A is ` 2,30,000. Since the income of Mr. A is greater than that of Mrs. A, the
income of the minor children have to be clubbed in the hands of Mr. A. It is assumed that
this is the first year when clubbing provisions are attracted.
Income derived by a minor child from any activity involving application of his/her skill,
talent, specialized knowledge and experience is not to be clubbed. Hence, the income of
minor child C from exercise of special talent will not be clubbed.
However, interest from bank deposit has to be clubbed even when deposit is made out of
income arising from application of special talent. The total income of Mrs. A is ` 2,30,000.

Computation of total income of Mr. A for the A.Y. 2021-22


Particulars ` `

Income from profession 3,90,000


Income of minor son B from company deposit 15,000
Less: Exemption u/s 10(32) 1,500 13,500

Income of minor daughter C


From special talent – not to be clubbed --
Interest from bank 3,000
CLUBBING OF INCOME 101

Gift of ` 2,500 received from a non-relative is not taxable u/s


56(2)(vii) being less than the aggregate limit of ` 50,000. Nil
3,000
Less: Exemption u/s 10(32) 1,500 1,500
Gross Total Income 4,05,000

Illustration 12: A proprietary business was started by Smt. Rani in the year 2018. As on
1.4.2019 her capital in business was ` 3,00,000. Her husband gifted ` 2,00,000 on
10.4.2019, which amount Smt. Rani invested in her business on the same date. Smt. Rani
earned profits from her proprietory business for the Financial Year 2019-2020, ` 1,50,000
and Financial Year 2020-21 ` 3,90,000. Compute the income, to be clubbed in the hands of
Rani’s husband for the Assessment Year 2021-22 with reasons.
Solution: Section 64(1) of the Income-tax Act, 1961 provides for the clubbing of income in
the hands of the individual, if the income earned is from the assets transferred directly or
indirectly to the spouse of the individual, otherwise than for adequate consideration. In this
case Smt. Rani received a gift of ` 2,00,000 from her husband which she invested in her
business. The income to be clubbed in the hands of Smt. Rani’s husband for A.Y. 2021-22 is
computed as under:

Particulars Smt Rani’s Capital Total


Capital Contribution
Contribution out of gift from `
` husband
`
Capital as at 1.4.2019 3,00,000 -- 3,00,000
Investment on 10.4.2019 out of gift received
from her husband ________ 2,00,000 2,00,000
3,00,000 2,00,000 5,00,000
Profit for F.Y. 2019-20 to be apportioned on
the basis of capital employed on the first day
of the previous year i.e. on 1.4.2019 1,50,000 _______ 1,50,000
Capital employed as at 1.4.2020 4,50,000 2,00,000 6,50,000
Profit for F.Y. 2020-21 to be apportioned on
the basis of capital employed as at 1.4.2020
(i.e. 45 : 20) 2,70,000 1,20,000 3,90,000
Therefore, the income to be clubbed in the hands of Smt. Rani’s husband for A.Y. 2020-21 is
` 1,20,000.

Illustration 13: Mr. Ghose has four minor children consisting 2 daughters and 2 sons. The
annual income of 2 daughters was ` 7,500 and ` 5,000 and of sons was ` 5,500 and ` 1,250
102 INCOME TAX

respectively. The daughter who was having income of ` 5,000 was suffering from a
disability specified under section 80U. Work out the amount of income earned by minor
children to be clubbed in the hands of Mr. Ghose.
Solution:
Income earned by minor children to be clubbed with the income of Mr. Ghose
(i) Income of two daughters (` 7,500 + Nil) 7,500
Less: Income exempt u/s 10(32) 1,500
Total (A) 6,000
(ii) Income of two sons (` 5,500 + ` 1,250) 6,750
Less: Income exempt u/s 10(32) (` 1,500 + ` 1,250) 2,750
Total (B) 4,000
Total income to be clubbed as per Section 64(1A) (A+B) 10,000
The income of daughter suffering from disability specified under section 80U is not to be
clubbed with the income of Mr. Ghose.

Illustration 14: Mr. Dhaval and his wife Mrs. Hetal furnish the following information:
`
(i) Salary income (computed) of Mrs. Hetal 4,60,000
(ii) Income of minor son ‘B’ who suffers from disability specified in Section 1,08,000
80U
(iii) Income of minor daughter ‘C’ from singing 86,000
(iv) Income from profession of Mr. Dhaval 7,50,000
(v) Cash gift received by ‘C’ on 2.10.2020 from friend of Mrs. Hetal on winning
of singing competition 48,000
(vi) Income of minor married daughter ‘A’ from company deposit 30,000
Compute the total income of Mr. Dhaval and Mrs. Hetal for the Assessment Year 2021-22.
Solution:
Computation of Total Income of Mr. Dhaval and Mrs. Hetal for the A.Y. 2021-22
Particulars Mr. Dhaval Mrs. Hetal
` `
Salaries 4,60,000
Profits and gains of business or profession 7,50,000
Income from other sources
Income by way of interest from company deposit
earned by
Minordaughter A [See Note (d)] 30,000
Less: Exemption under section 10(32) 1,500 28,500
Total Income 7,78,500 4,60,000
CLUBBING OF INCOME 103

Notes:
(a) The income of a minor child suffering from any disability of the nature specified in
Section 80U shall not be included in the hands of the parents. Hence, ` 1,08,000, being
the income of minor son ‘B’ who suffers from disability specified under section 80U,
shall not be included in the hands of either of his parents.
(b) The income derived by the minor from annual work or from any activity involving
exercise of his skill, talent or specialized knowledge or experience will not be included
in the income of his parent. Hence, in the given case, ` 86,000 being the income of the
minor daughter ‘C’ shall not be clubbed in the hands of the parents.
(c) Under section 56(2)(vii), cash gifts received from any person/persons exceeding
` 50,000 during the year in aggregate is taxable. Since the cash gift in this case does not
exceed ` 50,000, the same is not taxable.
(d) The clubbing provisions are attracted even in respect of income of minor married
daughter. The income of the minor will be included in the income of that parent whose
total income is greater. Hence, income of minor married daughter ‘A’ from company
deposit shall be clubbed in the hands of the Mr. Dhaval and exemption under section
10(32) of ` 1,500 per child shall be allowed in respect of such income.
104 INCOME TAX

MCQ

1. Income of a minor child suffering from any disability of the nature specified in
section 80U is –
(a) To be assessed in the hands of the minor child
(b) To be clubbed with the income of that parent whose total income, before
including minor’s income, is higher
(c) Completely exempt from tax
(d) To be clubbed with the income of father

2. Income arising to a minor married daughter is –


(a) To be assessed in the hands of the minor married daughter
(b) To be clubbed with the income of that parent whose total income, before
including minor’s income, is higher
(c) Completely exempt from tax
(d) To be clubbed with the income of her husband

3. Where a member of a HUF has converted or transferred his self-acquired property


for inadequate consideration into joint family property, income arising therefrom is-
(a) Taxable as the income of the transferor-member.
(b) Taxable in the hands of the HUF.
(c) Taxable in the hands of the karta of the HUF.
(d) Exempt from tax.

4. If the converted property is subsequently partitioned among the members of the


family, the income derived from such converted property as is received by the
spouse of the transferor will be taxable –
(a) As the income of the karta of the HUF
(b) As the income of the spouse of the transferor
(c) As the income of the HUF.
(d) As the income of the transferor-member

5. Exemption of a certain amount (not exceeding the income clubbed) is available


under section 10(32), where a minor’s income is clubbed with the income of the
parent. The maximum exemption available is –
(a) Upto 1,200 in respect of each minor child
(b) Upto ` 1,500 in respect of each minor child
CLUBBING OF INCOME 105

(c) Upto ` 2,000 in respect of each minor child


(d) Upto ` 1,000 in respect of each minor child

6. Mr. A gifts cash of ` 1,00,000 to his brother’s wife Mrs. B. Mr. B gifts cash of
` 1,00,000 to Mrs. A From the cash gifted to her, Mrs. B invests in a fixed deposit,
income there from is ` 10,000. Aforesaid ` 10,000 will be included in the total
income of ………
(a) Mr. A
(b) Mrs. A
(c) Mrs. B
(d) Mr. B

7. Scholarship received by a minor child is –


(a) To be assessed in the hands of the minor child
(b) To be clubbed with the income of that parent whose total income, before
including minor’s income, is higher
(c) Completely exempt from tax
(d) To be clubbed with the income of father

8. Income of a minor child from a fixed deposit with a bank, made out of income earned
from scholarship is-
(a) To be assessed in the hands of the minor child
(b) To be clubbed with the income of that parent whose total income, before
including minor’s income, is higher
(c) Completely exempt from tax
(d) To be clubbed with the income of father

9. Mr. X transfers income of ` 51,000 from rent to his major son without transfer of
house property. Rent of ` 51,000 is –
(a) Taxable in the hands of the transfer-father
(b) Taxable in the hands of his son
(c) Taxable in the hands of that parent whose total income is higher
(d) Exempt from tax

10. Interest from a fixed deposit received by a minor married daughter is –


(a) To be assessed in the hands of the minor child
(b) To be clubbed with the income if that parent whose total income, before
including minor’s income, is higher
(c) Completely exempt from tax
(d) To be clubbed with the income of her husband
SET OFF & CARRY
4
FORWARD OF LOSSES
Set off in the year of loss Carry forward & Time Whether
Set off in limit for Return
Head of Income subsequent c/f and to be
previous years set off of filed
Under Under Under Under losses before
same head Other head same head other due date
(Sec. 70) (Sec. 71) head u/s
139(1)
Income from Yes (Upto `
House Property 2,00,000
Yes only, balance Yes No 8 A.Y. No
carried
forward)
Profits and Gains
from Business or
Profession
Non – Speculation Yes (Both
Business from
Speculation
Yes, except
Business & Yes No 8 A.Y. Yes
from Salary
Non
Speculation
Business)
Speculation Yes (Only Yes (Only
Business from from
No No 4 A.Y. Yes
Speculation Speculation
Business) Business)
Unabsorbed
Depreciation,
Yes
Unabsorbed Yes
Except Indefinite
Scientific Research, Yes Except from Yes No
from Period
Family Planning Salary
Salary
expenditure
INCOME TAX 107

Agricultural Yes (only Yes (only


from from
No No 4 years No
agricultural agricultural
income) income)
Specified Business Yes (Only Yes (only
u/s 35 AD from from Indefinite
No No Yes
Specified Specified Period
Business) Business)
Capital Gains
Short term Only from
Yes Yes No 8 A.Y. Yes
LTCG
Long Term Only from Only from
No No 8 A.Y. Yes
LTCG LTCG
Income from
Other Sources
Lotteries, Loss of
It cannot
Crossword puzzles, same
No N.A. N.A. be carry N.A.
card games, cannot be
forward
gambling etc. set off
Loss from activity
Yes (Only Yes (Only
of owning and
from such No from such No 4 A.Y. Yes
maintaining race
activity) activity)
horses
Other Income It cannot
Yes Yes N.A. N.A. N.A.
be c/f
Note: Loss from the activity of owning & maintaining animal for races other than horse race
can be set off & carry forward like any other business loss.

Carry Forward of Losses of an Eligible Start-up [Sec. 79]


In the case of a company, not being a company in which the public are substantially
interested but being an eligible start-up as referred to in section 80-IAC, the loss incurred in
any year prior to the previous year shall be carried forward and set off against the income
of the previous year, if, all the shareholders of such company who held shares carrying
voting power on the last day of the year or years in which the loss was incurred, ̶
(i) Continue to hold those shares on the last day of such previous year; and
(ii) Such loss has been incurred during the period of 7 years beginning from the year in
which such company is incorporated.
108 SET OFF & CARRY FORWARD OF LOSSES

Loss on sale of shares, securities or units [Sec. 94 (7)]


If the three conditions given below are satisfied, Section 94(7) is applicable:
1. Any person buys any securities/ shares/ units within a period of 3 months before the
record date.
2. Such a person transfers such securities/ shares/ units within a period of 3 months (9
months in the case of units) after the record date.
3. The dividend or income on such securities / shares / units received (or receivable) by
such person is exempt from tax.
According to Sec. 94(7), any loss from such transaction can be set off only to the extent it
exceeds the amount of income received / receivable which is exempt from tax.

Loss arising in the case of bonus stripping [Sec. 94(8)]


Section 94(8) is applicable if the following conditions are satisfied:
1. The taxpayer buys any unit within a period of 3 months prior to the record date.
2. Such person is allotted bonus units.
3. Such person transfers all (or any) of the original units within a period of 9 months after
such record date.
4. But he continues to hold all (or any) of the bonus units.

Consequences if the above conditions are satisfied:


1. The loss (if any) arising to the taxpayer on account of purchase and sale of all (or any)
of the aforesaid original units shall be ignored for the purposes of computing his
income chargeable to tax.
2. The amount of loss so ignored shall be deemed to be the cost of purchase or acquisition
of bonus units as are held by him on the date of such sale or transfer.

Illustration 1: From the following information submitted to you, compute the taxable
income in the following situation.
Situation I Situation II
` `
Long term capital gain/loss (+) 1,70,000 (-) 3,00,000
Short term capital gain/loss (-) 50,000 (+) 1,10,000
Business income/loss (-) 80,000 (-) 90,000
INCOME TAX 109

Solution:
Situation I Situation II
` `
Capital gain
Long term capital gain/loss (+) 1,70,000 (-) 3,00,000
(set off not possible)
Short term capital gain/loss (-) 50,000 (+) 1,10,000
Capital gain/loss after set off 1,20,000 1,10,000
Set off of business income/loss (-) 80,000 (-) 90,000
Total income 40,000 20,000

In situation II, long-term capital loss of ` 3,00,000 will be carried forward and the total
income shall be ` 20,000.

Illustration 2: From the following information submitted to you, compute the total income
of A for the Assessment Year 2021-22.
`
Income from salary 1,80,000
Income from house property 40,000
Business loss (-) 1,90,000
Loss from a specified business referred to in Section 35AD (-) 60,000
Short-term capital loss (-) 60,000
Long-term capital gain 2,40,000

Solution: Computation of total income of A for the Assessment Year 2021-22


` `
Income from salary 1,80,000
Income from House Property
Income 40,000
Less: Business loss adjusted (-) 10,000 30,000

Loss from specified business not allowed to be set off (-) 60,000

Income from Capital Gain


Long-term capital gain 2,40,000
Less: Short-term capital loss 60,000
1,80,000
Less: Business loss adjusted 1,80,000 Nil
Gross total income 2,10,000
Less: Deductions Nil
Total Income 2,10,000
110 SET OFF & CARRY FORWARD OF LOSSES

1. Business loss should first be set off from long-term capital gain as the long-term capital
gain is taxable @ 20% where as the income from house property, in this case, is taxable
@ 10%.
2. It may be noted that business loss cannot be set off against income under head salary.

Illustration 3: Compute the taxable income in the following two situations.


Situation I Situation II
` `
Income/loss from manufacturing business 1,50,000 (-) 3,60,000
Income/loss from speculation business (-) 80,000 (+) 3,50,000
Loss from a specified business referred to in Sec. 35AD (-) 40,000 (+) 40,000
Short-term capital gains/loss (-) 1,70,000 (-) 1,70,000
Agricultural income/loss (-) 40,000 (+) 60,000
Solution:
Situation I Situation II
` `
Income/loss from manufacturing business 1,50,000 (-) 3,60,000
Income/loss from speculation business No set off (+) 3,50,000
Loss from specified business No set off (+) 40,000
Income 1,50,000 30,000
STCL cannot be set off from any other head. No loss can be set off from Agriculture Income.

Illustration 4: Rohit furnishes the following particulars of his income for the PY 2020-21:
`
1. Income from salary (computed) 68,000
2. Income from house A 36,000
3. Loss from house B 24,000
4. Loss from house C 22,000
5. Profit from business A 60,000
6. Profit from business B 70,000
7. Profit from share business (speculative) 82,000
8. Loss from silver business (speculative) 94,000
9. Long-term capital gain on sale of shares on which transaction tax
has been paid 22,000
10. Short-term capital loss on sale of land 44,000
11. Income from card games 22,000
12. Winnings from lotteries (gross) 60,000
13. Income from horse races in Delhi (Gross) 40,000
14. Loss from horse races in Bangalore 21,000
Compute the Gross Total Income of Rohit for the Assessment Year 2021-22.
INCOME TAX 111

Solution:
` ` `
Income from Salary 68,000
Income from House Property
Income from House A 36,000
Loss from House B (-) 24,000
Loss from House C (-) 22,000 (-) 10,000
Income from Business
Business A 60,000
Business B 70,000 1,30,000
Profit from speculation business 82,000
Less: Loss from speculation business (-) 94,000
Carried forward speculation loss (-) 12,000
Capital Gain
Long term capital gain Exempt
Less: Short term capital loss (-) 44,000
Capital loss carried forward (-) 44,000
Income from other sources
Incoming from card games 22,000
Winning from lotteries 60,000
Income from Horse races 40,000 1,22,000
Gross Total Income 3,10,000
Note: Loss from horse race in Bangalore cannot be set off against any income.

Illustration 5: Munna submits the following information for previous year 2020-21:
`
1. Profit from Business A situated in Delhi 1,50,000
2. Profit from Business B situated in Bombay 1,00,000
3. Loss from Business C carried in New York (the business is controlled from
India but profits are not received in India) 60,000
4. Unabsorbed depreciation of business C 35,000
5. Income from house property situated in India 10,000
6. Income from house property situated in London (rent received in London) 20,000

Find out the Gross Total Income of Munna for the Assessment Year 2021-22 if he is (a)
Resident in India (b) Not ordinarily resident in India and (c) Non-resident in India.
112 SET OFF & CARRY FORWARD OF LOSSES

Solution:
Not
Non-
Resident ordinarily
Resident
` Resident
`
`
Business Income
Business A (Profit) 1,50,000 1,50,000 1,50,000
Business B (Profit) 1,00,000 1,00,000 1,00,000
2,50,000 2,50,000 2,50,000
Business C (Loss);
(controlled from India but received out of India) (-) 60,000 (-) 60,000 Nil
1,90,000 1,90,000 2,50,000
Unabsorbed depreciation of business C (-) 35,000 (-) 35,000 Nil
1,55,000 1,55,000 2,50,000
Income from house property
Property in India 10,000 10,000 10,000
Property in London 20,000 -- --
Gross Total Income 1,85,000 1,65,000 2,60,000

Illustration 6: From the following details, compute the Gross Total Income of Amit for the
assessment year 2021-22:
`
1. Taxable income from salary 80,000
2. Income from house property House ‘A’ (let out) (-) 95,000
House ‘B’ (self-occupied, interest on borrowed money) (-) 9,000
3. Short-term capital gain 12,000
4. Loss from long-term assets 25,000
5. Interest on securities (Gross) 10,000
Solution:
` `
1. Taxable income from salary 80,000
2. Income from house property House A (Let out) (-) 95,000
House B (Self-occupied) (-) 9,000 (-) 1,04,000
3. Income from capital gains: Short-term capital gain 12,000
4. Long-term capital loss to be carried forward to be carried
forward (-) 25,000
5. Income from other Sources: Interest on securities (Gross) 10,000
Gross Total Income Nil
Note: Loss from long-term capital assets cannot be set off against short-term capital gain or
income under other heads of income. Such a loss of ` 25,000 which could not be adjusted in
INCOME TAX 113

the Assessment Year 2021-22 will be carried forward to the subsequent Assessment Years.
Loss from house property amounting to ` 2,000 shall be carried forward.

Illustration 7: X purchases on May 10, 2020, 1,000 preference shares of ` 10 each in A Ltd.
@ ` 55.55. On October 20, 2020, he transfers 800 shares @ ` 37 per share and remaining
200 shares are transferred on December 20, 2020 @ ` 20 per share. A Ltd. declares 50%
dividend (record date: August 3, 2020). During previous year 2020-21, he has generated
long term capital gain of ` 76,000 on sale of gold.
Solution:
800 shares 200 shares
` `
Sale consideration 29,600 4,000
Less: Cost of acquisition 44,440 11,110
Short-term capital gain (-) 14,840 (-) 7,110
Dividend 4,000 1,000
Whether Section 94(7) is applicable Yes No
Computation of income
Long-term capital gain on sale of gold 76,000
Less:
Short-term capital loss on sale of 800 shares [` 14,840 - `
4,000] (-) 10,840
Short-term capital loss on sale of 200 shares (-) 7,110
Long-term capital gain 58,050

Illustration 8: Compute capital gains in the following cases for the AY 2021-22:
Name of the units: Growth units of PNI Mutual Fund (debt based) (face value: ` 10)
Record date for allotment of bonus unit- December 5, 2020 (a person holding 2 units will
get 1 bonus unit). X purchases 1,000 above-mentioned units on October 1, 2020 at the rate
of ` 23 per unit. On December 5, 2020, he gets 500 bonus units.

Find out the tax consequences in the following different situations:


1. He transfers 800 original units on March 10, 2021 at the rate of ` 26 per unit. He does
not transfer remaining original units and bonus units till the expiry of 9 months from
the record date (i.e., September 5, 2021).
2. He transfers 800 original units on March 10, 2021 at the rate of ` 17 per unit. He does
not transfer remaining original units and bonus units.
3. He transfers 900 original units on March 10, 2021 at the rate of ` 18 per unit. He does
not transfer remaining original units till the expiry of 9 months from the record date
(i.e., September 5, 2021). On May 1, 2021, he transfers 100 bonus units at the rate of
` 17 per unit.
114 SET OFF & CARRY FORWARD OF LOSSES

4. He transfers 700 original units on March 10, 2021 at the rate of ` 14 per unit. He does
not transfer remaining original units till the expiry of 9 months from the record date
(i.e., September 5, 2021). On May 1, 2021, he transfers 400 bonus units at the rate of
` 13 per unit.
5. He transfers 400 original units on January 1, 2021 at the rate of ` 24 per unit. On March
1, 2021, he further transfers 200 original units at the rate of ` 19 per unit. He does not
transfer remaining original units till the expiry of 9 months from the record date (i.e.,
September 5, 2021). However, 200 bonus units are transferred on September 10, 2021
at the rate of ` 15 per unit.
Solution:
Case 1 Case 2 Case 3 Case 4 Case 5
` ` ` ` `
Original units (case of loss)
Sale consideration -- 13,600 16,200 9,800 3,800
Less: Cost of acquisition -- 18,400 20,700 16,100 4,600
Short-term capital gain -- (-) 4,800 (-) 4,500 (-) 6,300 (-) 800
Short-term capital loss which cannot
be adjusted against any other capital
gain (a) NA 4,800 4,500 6,300 800

Original units (case of gain)


Sale consideration 20,800 -- -- -- 9,600
Less: Cost of acquisition 18,400 -- -- -- 9,200
Short-term capital gain 2,400 -- -- -- 400
Units held by X after the aforesaid
transactions Bonus units (b) 500 500 500 500 500
Cost of acquisition of bonus units (per
unit) [(a) ÷ (b)] Nil 9.6 9.0 12.6 1.6

Bonus units
Sale consideration -- -- 1,700 5,200 3,000
Less: Cost of acquisition -- -- 900 5,040 320
Short-term capital gain -- -- 800 160 2,680

Illustration 9: Simran, engaged in various types of activities, gives the following particulars
of her income for the year ended 31.3.2021:

`
(a) Profit of business of consumer and house-hold products 50,000
(b) Loss of business of readymade garments 10,000
INCOME TAX 115

(c) Brought forward loss of catering business which was closed in Asst. year
2020-21. 15,000
(d) Short-term loss on sale of securities and shares 15,000
(e) Profit of speculative transactions entered into during the year 12,500
(f) Loss of speculative transactions of Asst. Year 2016-17 not set off till Asst. Year
2020-21 15,000
Compute the total income of Simran for the A.Y. 2021-22.
Solution:
Computation of total income of Simran for the A.Y. 2021-22
Particulars ` `
Profit of business of consumer and house-hold products 50,000
Less: Loss of business of readymade garments for the year 10,000
40,000
Less: Brought forward loss of catering business closed in A.Y. 2020-21 set
off against business income for the current year 15,000 25,000
Profit of speculative transaction 12,500
Total Income 37,500
Note:
1. Loss of speculative transaction of A.Y. 2016-17 is not allowed to be set off against the
profit of speculative transaction of the A.Y. 2021-22, since, as per the provisions of
Section 73(4), such loss can be carried forward for set-off for a maximum period of 4
years only i.e. up to A.Y. 2020-21.
2. Short term capital loss of ` 15,000 on sale of securities and shares has to be carried
forward as per Section 74 since there is no income under the head Capital Gains for the
A.Y. 2021-22. The loss is to be carried forward for set off in future years against income
chargeable under the head Capital Gains. Such loss can be carried forward for a
maximum period of 8 Assessment Years.

Illustration 10: M/s. Vivitha & Co., a partnership firm, with four partners A, B, C and D
having equal shares, furnishes the following details, summarized from the valid returns of
income filed by it:
Assessment Year Item eligible for carry forward & set off
2019-20 Unabsorbed business loss ` 1,20,000
2020-21 Unabsorbed business loss ` 1,90,000
2020-21 Unabsorbed depreciation ` 1,20,000
2020-21 Unabsorbed long-term capital losses:
-from shares ` 1,10,000
-from building ` 1,90,000
C who was a partner during the last three years, retired from the firm with effect from
1.4.2020. The summarized results of the firm for the Assessment Year 2021-22 are as
under:
116 SET OFF & CARRY FORWARD OF LOSSES

`
Income from house property 70,000
Income from business:
Speculation 2,20,000
Non-speculation (-) 50,000
Capital Gain
Short-term (from sale of shares) 40,000
Long-term (from sale of building) 2,10,000
Income from other sources 60,000
Briefly discuss, how the items brought forward from earlier years can be set off in the hands
of the firm for the Assessment Year 2021-22, in the manner most beneficial to the assessee.
Also show the items to be carried forward.
Solution: According to Section 78(1), where there is a change in the constitution of the firm,
the loss relatable to outgoing partner (whether by way of retirement or death) has to be
excluded for the purposes of carry forward. However, this provision does not apply in the
case of unabsorbed depreciation. Accordingly, M/s. Vivitha & Co. is entitled to carry forward
the losses to the extent detailed here below:
Item Loss Relatable to C Bal. eligible for
(`) (`) carry forward
Business loss A.Y. 2019-20 1,20,000 30,000 90,000
Business loss of A.Y. 2020-21 1,90,000 47,500 1,42,500
Long-term capital loss of A.Y. 2020-21 3,00,000 75,000 2,25,000

Set off of items in the hands of M/s. Vivitha & Co. for the A.Y. 2021-22
Particulars Amount Amount
(`) (`)
1. Income from house property
Current year income 70,000
Less: Brought forward unabsorbed depreciation (See Note 1) 70,000 Nil
2. Profits and gains of business or profession
Current year speculative business profits 2,20,000
Less: Current year non-speculation loss set off (See Note 2) 50,000
1,70,000
Less: Brought forward business losses of earlier year
(2019-20 ` 90,000 and balance of 2020-21 ` 80,000) 1,70,000 Nil
(See Note 3)
3. Capital gain
Short term (from sale of shares) 40,000
Long-term (from sale of building) 2,10,000
Less: Brought forward long term capital loss of A.Y. 2020-21 2,10,000 Nil
(See Note 4)
INCOME TAX 117

4. Income from other sources


Current year before set off 60,000
Less: Brought forward depreciation (See Note 1) 50,000 10,000
Total Income 50,000
Losses to be carried forward to A.Y. 2022-23
Business loss (` 1,42,500 - ` 80,000) 62,500
Long term capital loss (` 2,25,000 - ` 2,10,000) 15,000
Both these losses relate to A.Y. 2020-21.

Notes:
(1) Unabsorbed depreciation can be set off against income from any head. Hence, it will be
advantageous to set off unabsorbed depreciation against income from house property
and income from other sources.
(2) In the current year, non-speculation business loss can be set off against speculation
business income.
(3) Brought forward non-speculation business loss can also be set off against speculation
business income of current year.
(4) According to Section 74, brought forward long-term capital losses shall be set off only
against long-term capital gains of current year.
(5) The set-off and carry forward of losses should be most beneficial to the assessee. If off
brought forward depreciation is set off against current year’s business income first,
then the quantum of brought forward business loss which can set off against current
year’s business income will be lower. This will not be beneficial to the assessee.

Illustration 11: Mr. P, a resident individual, furnishes the following particulars of his
income and other details for the Previous Year 2020-21:
`
(i) Income from salary 18,000
(ii) Net annual value of house property 70,000
(iii) Income from business 80,000
(iv) Income from speculative business 12,000
(v) Long term capital gain on sale of land 15,800
(vi) Loss on maintenance of race horse 9,000
(vii) Loss on gambling 8,000
Depreciation allowable under the Income-tax Act, 1961, comes to ` 8,000 for which no
treatment is given above.
The other details of unabsorbed depreciation and brought forward losses are:
`
(i) Unabsorbed depreciation 9,000
(ii) Loss from speculative business 16,000
118 SET OFF & CARRY FORWARD OF LOSSES

(iii) Short term capital loss 7,800


(iv) Unrealized rent 17,000
Compute the gross total income of Mr. P, for the Assessment Year 2021-22 and the amount
of loss that can or cannot be carried forward.
Solution:
Computation of Gross Total Income of Mr. P for the A.Y. 2021-22
` `
(i) Income from salary 18,000
(ii) Income from House Property
Net annual value 70,000
Less: Deduction under section 24 (30% of ` 70,000) 21,000 49,000
(iii) Income from business and profession
(a) Income from business 80,000
Less: Current year depreciation 8,000
72,000
Less: Unabsorbed depreciation 9,000 63,000
(b) Income from speculative business 12,000
Less: Brought forward loss from speculative business 12,000 Nil
(Balance loss of ` 4,000 (i.e. ` 16,000 - ` 12,000) can be carried
forward to the next year)
(iv) Income from capital gain
Long term capital gain on sale of land 15,800
Less: Brought forward short term capital loss 7,800 8,000
Gross Total Income 1,38,000

Amount of loss to be carried forward to the next year


Particulars `
Loss from speculative business (to be carried forward as per Section 73) 4,000
Loss on maintenance of race horses (to be carried forward as per Section 74A) 9,000
Note:
(i) Loss on gambling can neither be set-off nor be carried forward.
(ii) As per Section 74A(3), the loss incurred on maintenance of race horses cannot be set-
off against income from any other source other than the activity of owning and
maintaining race horses. Such loss can be carried forward for a maximum period of 4
Assessment Years.
(iii) Speculative business loss can set off only against income from speculative business of
the current year and the balance loss can be carried forward to A.Y. 2022-23. It may
be noted that speculative business loss can be carried forward for a maximum of four
years as per Section 73(4).
INCOME TAX 119

MCQ

1. According to section 80, no loss which has not been determined in pursuance of a
return filed in accordance with the provisions of section 139(3), shall be carried
forward. The exceptions to this are –
(a) Loss from specified business under section 73A
(b) Loss under the head “Capital Gains” and unabsorbed depreciation carried
forward under section 32(2)
(c) Loss from house property and unabsorbed depreciation carried forward under
section 32(2)
(d) Loss from speculation business under section 73

2. Section 70 enables set off of losses under one source of income against income from
any other source under the same head. The exceptions to this section are –
(a) Loss under the head “Capital Gains”, Loss from speculative business, Loss from
house property and loss from the activity of owning and maintaining race horses
(b) Long-term capital loss, Loss from speculative business, Loss from specified
business and loss from the activity of owning and maintaining race horses
(c) Short-term capital loss and loss from speculative business
(d) Loss from specified business and short-term capital loss

3. Mr. X incurred long-term capital loss from sale of listed shares in recognized stock
exchange and STT is paid at the time of acquisition and sale of such share. Such loss–
(a) Can be set-off only against long-term capital gains
(b) Can be set-off against both short-term capital gains and long-term capital gains
(c) Can be set-off against any head of income.
(d) Not allowed to be set-off.

4. The maximum period for which speculation loss can be carried forward is –
(a) 4 years
(b) 8 years
(c) Indefinitely
(d) Not allowed to be carry forward

5. Mr. A incurred short-term capital loss of ` 10,000 on sale of shares through the
National Stock Exchange. Such loss –
(a) Can be set-off only against short-term capital gains
(b) Can be set-off against both short-term capital gains and long-term capital gains
120 SET OFF & CARRY FORWARD OF LOSSES

(c) Can be set-off against any head of income.


(d) Not allowed to be set-off

6. The maximum period for which loss from specified business can be carried forward
is –
(a) 4 years
(b) 8 years
(c) Indefinitely
(d) Not allowed to be carry forward

7. Loss from house property of ` 3,10,000 of A.Y. 2019-20 is allowed to be set-off


against income from house property of A.Y. 2021-22 of ` 5,00,000 to the extent of –
(a) ` 2,00,000
(b) Fully allowed i.e. ` 3,10,000
(c) ` 2,50,000
(d) ` 1,00,000

8. Any loss from the specified business referred to in section 35AD can be set off
against –
(a) Only profit and gains of same specified business of the assessee
(b) Profits and gains of any business of the assessee
(c) Profit and gains of any other specified business of the assessee
(d) Income from any other head

9. Business loss of the current year cannot be set-off against –


(a) Any income other than business income
(b) Long-term capital gain
(c) Either long-term capital gain or short-term capital gain
(d) Salary income

10. Brought forward loss from house property can be set-off –


(a) Against any head of income to the extent of ` 2,00,000
(b) Against income from house property to the extent of ` 2,00,000
(c) Against income from house property without any limit
(d) Against any head of income without any limit.
5 DEDUCTIONS

Deduction u/c VI A not available from LTCG u/s 112, LTCG u/s 112A, STCG u/s 111A,
lottery income, races etc.

Sec. Available to Condition Deduction


• Subscription to notified bond of NABARD.
• ≥ 5 years term deposit in bank/ post office.
• LIP – Self, Spouse, Children (Dependent,
Independent Married, Unmarried, Major,
Minor)
- Maximum deduction 20% of sum
assured.
- In case of Insurance Policy issued on or
after 1-4-2012 maximum deduction is
10% of sum assured.
- In case of Insurance Policy issued on or
after 1-4-2013 maximum deduction is
15% of sum assured if policy issued in
respect of person referred to in Sec.
80U or 80DDB.
• Contribution to Provident Fund, Up to `
Superannuation Fund. 1,50,000 For
• 6 years NSC & Accrued interest for initial 5 80C, 80CCC &
years. 80CCD(1)
Individual / • Tuition Fees (for full time education of two (Sec. 80CCE)
80C HUF children in India)
• 15 years PPF [Self, Spouse, Children]. Date
of Clearing of cheque is considered as date
of payment. The annual ceiling for
investment in PPF is ` 1,50,000
• Payment towards Cost of purchase /
construction of Residential property
including repayment of housing loan (Only
Principal Component)
• Notified Unit (ELSS) of UTI, Mutual fund,
LIC
• Subscription to Public Deposit Scheme of
INCOME TAX 122

HUDCO (Housing & Urban Development


Corporation Ltd.)
• Senior Citizen Saving Scheme
• Amount deposited in Sukanya Samriddhi
Account scheme
Deposit to Pension Fund of LIC or other
80CCC Individual insurer

Contribution to NPS (National Pension Covered in


Scheme) overall
Upto 10% of Basic Salary & DA (Employee ceiling of Sec.
80CCD(1) Individual
contribution), or 80CCE
Upto 20% of GTI (Individual other than
Employee)
Contribution to NPS (National Pension
Scheme)
Deductions
80CCD(1B) Individual Deduction available to assessee, whether or
up to 50,000
not any deduction is allowed u/s 8CCD(1)

Contribution to NPS (National Pension


Scheme)
Not covered
Upto 14% of Basic Salary & DA (Employer
in overall
80CCD(2) Individual contribution, if employer is CG)
ceiling of Sec.
Upto 10% of Basic Salary & DA (Employer
80CCE
contribution, in case of any other employer)

Mediclaim Policy of GIC or Central


Government Health Scheme or contributory
Health Service Scheme paid by any mode other Up to
than cash and medical expenditure in ` 25,000
respect of senior citizen without any health Upto `50,000
insurance. (if anyone is
• Insurance taken on the health of Self, Senior
Individual Spouse and Dependent Children and Citizen)
80D medical expenditure incurred if anyone is
/HUF
senior citizen.
• Insurance taken on the health of Parents Upto ` 25,000
(Father & Mother whether dependent or Upto ` 50,000
not) and medical expenditure incurred if (if anyone is
any parent is senior citizen. Senior Citizen
i.e. 60 years
or more)
123 DEDUCTIONS

Upto ` 5,000.
• Preventive healthcheck up of Self, Spouse, However, the
Parents & dependent children paid by any overall
mode (cash payment allowed). qualifying
limit given
Note: Limit of ` 5,000 is for self, spouse, above
dependent children & parents all taken remains
together. unchanged

Maintenance for Medical Treatment including


(nursing), training and rehabilitation of Fixed
dependent with disability. Deduction
Resident
• Medical Treatment. ` 75,000 or
80DD Individual/
• Deposit in Scheme of LIC or other insurer ` 1,25,000 (if
HUF
• Disable dependent must not claim deduction disability >
u/s 80U 80%)
• Spouse, Parent, Children, Brother and Sister.
Medical Treatment of Specified disease of Self, Up to
Spouse, Parent, Children, Brother and Sister. ` 40,000 or
• Amount received from Insurance Up to
Resident ` 1,00,000 if
Company shall be reduced from deduction
80DDB Individual/ incurred in
computed. respect of
HUF
Senior
Citizen

Payment of Interest on loan taken for higher


studies from any financial Institution or
approved charitable Institution.
Amount of
• Self, Spouse, Children (Dependent,
interest paid
Independent, Married, Unmarried, Major,
80E Individual (Not the
Minor)
Principal
• Available for 8 AY beginning with AY year
sum)
in which payment of interest begins.
• Any Studies after passing Xth including
Vocational Studies.
Interest on loan taken from a financial
institution for purchasing a residential house
property. This deduction shall be available
Up to
80EE Individual subject to the following conditions:
₹ 50,000
• Amount of loan up to 35,00,000
• Value of Residential property up to
₹ 50,00,000
INCOME TAX 124

• Assessee does not own any other


residential house property
• Loan sanctioned during PY 2016-17
Interest on loan taken from a financial
institution for purchasing a residential house
property. This deduction shall be available
subject to the following conditions:
• Stamp duty value of property does not
exceed ₹ 45 lakh
80EEA Individual • Assessee does not own any other
Up to
residential property on the date of
₹ 1,50,000
sanction of loan.
• Loan sanctioned during PY 2019-20 or PY
2020-21
• Should not be eligible to claim deduction
u/s 80EE
Interest on loan taken from a financial
institution for purchase of electric vehicle.
• Loan sanctioned during 1.4.2019 to Up to
80EEB Individual
31.3.2023 ₹ 1,50,000
• Deduction available till the repayment of
loan continues
Donation (any sum of money)
• Available in respect of fund set up for
Charitable purpose not religious purpose
GTI xx
See Note
Less: LTCG, STCG u/s 111A xx
80G Any assessee after the
Less: All ded u/s 80C to 80U except 80G xx
Chart
Adjusted GTI xx
Any payment exceeding ` 2,000 shall only be
allowed as deduction if such sum is paid in
any mode other than cash.
Rent Paid in case of non-receipt of HRA
Least of the following amount is deductible:
• Rent Paid (-) 10% of Adjusted GTI
• ` 5,000 per month
• 25% of Adjusted GTI
Conditions: Lower of
80GG Individual
• The person should not own any the 3
residential house at the place of
business or employment.
• The assessee should not own any
residential house at any other place
and claim benefit of SOP.
125 DEDUCTIONS

Sum paid to following institutions:-


• Approved research Association,
University, College or other institution to
be used for scientific research. (similar to
Sec. 35)
• Approved research association which
has as its object the undertaking of
Assessee not
research in social science or statistical 100% of the
80GGA having PGBP
research or to a university, college or sum paid
income
other institution for research in social
science or statistical research. (similar to
Sec. 35)
Any payment exceeding ₹ 10,000 (₹ 2,000
w.e.f. 1.6.2020) shall only be allowed as
deduction if such Sum is paid in any mode
other than cash.
Donation for any sum to Indian Political Party
Indian or Electoral Trust by any mode other than Amount
80GGB
Company Cash. donated

Any assessee
other than
Indian Co., Donation for any sum to Indian Political Party
Amount
80GGC local or Electoral Trust by any mode other than
donated
authority & Cash.
AJP funded
by the Govt.
Additional emoluments paid to new Workmen
• Deduction available only to business
covered u/s 44AB.
• Cost incurred on any employee whose
total emoluments are less than or equal 30% of
to ` 25,000 p.m. is additional wages. Additional
• Minimum number of days of Wages Paid
80JJAA Any Assessee employment in a financial year is 240 (No
days (150 days in case of manufacture deduction if
of Apparel, foot wear & Leather) ROI not filed
• Employee should participate in the by due date)
recognized PF.
• Emoluments paid by account payee
cheque/ draft or ECS
INCOME TAX 126

Royalty Income of Author Royalty


Case 1 : Copyright transferred for lumpsum Received less
consideration expenses
Case 2 : Copyright not transferred, Royalty or
Received whether in Lumpsum or otherwise ` 3,00,000
• For the purpose of Deduction in Case 2, whichever is
Resident
80QQB royalty received not to exceed 15% of lower
Individual
value of book sold. (No
• Amount Received in India in convertible deduction if
foreign exchange within 6 months from ROI not filed
end of P.Y. shall be taken as Royalty by due date)
received

Royalty
Received less
expenses
or
Royalty on Patent
` 3,00,000
• Amount Received in India in convertible
Resident whichever is
80RRB foreign exchange within 6 months from
Individual lower
end of PY shall be taken as Royalty
(No
received
deduction if
ROI not filed
by due date)

Individual
except Senior Interest on Saving Account with a Scheduled Upto
80TTA
Citizen/ Bank, or a Co-operative Bank or Post Office ` 10,000
HUF
Individual
Interest on any Deposit with a Scheduled Upto
80TTB (Senior
Bank, or a Co-operative Bank or Post Office ` 50,000
Citizen)
Fixed
Suffering from disability (Blindness, Low Deduction
vision, Leprosy-cured, Hearing impairment, ` 75,000 or
80U Individual
Locomotors disability, Mental retardation, ` 1,25,000 (if
Mental illness) disability >
80%)
127 DEDUCTIONS

Note:
Deduction under Section 80G
(A) Donations eligible for deduction without any limit:
1. 100% Deduction will be allowed if donations are given to-
(a) National Children’s Fund
(b) Prime Minister’s National Relief Fund
(c) Armenia Earthquake Relief Fund
(d) Africa (Public contribution-India) Fund
(e) National Foundation for Communal Harmony
(f) National Illness Assistance Fund
(g) National Cultural Fund set up by Government
(h) Gujarat Govt.’s Earthquake Relief Fund
(i) National Defence Fund set up by Government
(j) National Sports Fund set up by Govt
(k) Zila Sahakarta Samiti
(l) An approved University or other educational institution of national eminence
(m) Maharastra Chief Minister’s Earthquake Relief Fund
(n) The National Blood Transfusion Council or any State Blood Transfusion
Council
(o) Any fund set up by a State Government to provide medical relief to the poor
(p) Central Welfare fund for Army or Air Force or Indian Naval Benevolent Fund
(q) Andhra Pradesh Chief Minister’s Cyclone Relief Fund
(r) Fund for Technology Development and Application set up by Government
(s) Chief Minister’s Relief fund or Lieutenant Governor’s Relief Fund of any State
or Union Territory
(t) Welfare trust for persons with Autism, Cerebral Palsy and Multiple Disabilities
(u) National Fund for Control of Drug Abuse.
(v) Swachh Bharat Kosh
(w) Clean Ganga Fund
(x) Prime Ministers Citizen Assistance and Relief in Emergency Situation Fund
(PM Cares Fund)

2. 50% Deduction will be allowed if donations made to-


(a) Jawaharlal Nehru Memorial Fund
(b) Rajiv Gandhi Foundation
(c) Indira Gandhi Memorial Trust
(d) Prime Minister’s Drought Relief Fund

(B) Donations eligible for deduction subject to qualifying limit (i.e. 10% of Adjusted
GTI):
1. 100% deduction shall be allowed subject to the qualifying amount if
donations are made:
INCOME TAX 128

(a) To Government or any approved local authority, institution or association to


be utilized for promoting family planning;
(b) By company to Indian Olympic Association or other notified association /
institution in India for development of infrastructure for sports and games, or
sponsorship thereof in India.

2. 50% deduction shall be allowed subject to the qualifying amount if donations


are made:
(a) To Government or any approved local authority, for charitable purpose except
promoting family planning.
(b) To approved charitable institution, which satisfies the conditions of Section
80G(5).
(c) For renovation or repair of any temple, mosque, gurudwara, church or other
place notified to be of historic / archaeological artistic importance or as a
place of public worship of renown.
(d) To housing development authority or city / town / village development
authority constituted in India by or under any law.
(e) Corporation for promoting interests of minority community as given under
Section 10(26BB).
(f) Chartered Accountants Benevolent Fund.

Illustration 1: Compute the eligible deduction under section 80C for A.Y. 2021-22 in
respect of life insurance premium paid by Mr. Ganesh during the P.Y., the details of which
are given hereunder –
Date of issue of Person insured Actual capital Insurance
policy sum assured premium
(`) 2020-21 (`)
(i) 1/4/2011 Self 3,00,000 40,000
(ii) 1/5/2014 Spouse 1,50,000 20,000
(iii) 1/6/2015 Handicapped Son (Section 4,00,000 80,000
80U disability)
Solution:
Date of Person Actual Insurance Deduction Remark
issue of insured capital sum premium u/s 80C (restricted
policy assured (`) paid during for A.Y. to % of
2020-21 (`) 2021-22 sum
(`) assured)
(i) 1/4/2011 Self 3,00,000 40,000 40,000 20%
(ii) 1/5/2014 Spouse 1,50,000 20,000 15,000 10%
(iii) 1/6/2015 Handicapped 4,00,000 80,000 60,000 15%
son (section
80U disability)
Total 1,15,000
129 DEDUCTIONS

Illustration 2: Mr. A, aged about 61 years, has earned a lottery income of ` 1,20,000 (gross)
during the P.Y. 2020-21. He also has interest on Fixed Deposit of ` 30,000. He invested an
amount of ` 10,000 in Public Provident Fund account and ` 24,000 in National Saving
Certificates. What is the total income of Mr. A for the A.Y. 2021-22?

Solution: Computation of total income of Mr. A for A.Y. 2021-22


Particulars ` `
Income from other sources
- Interest on Fixed Deposit 30,000
- Lottery income 1,20,000
Gross Total Income 1,50,000
Less: Deductions under Chapter VIA
Under section 80C
- Deposit in Public Provident Fund 10,000
- Investment in National Saving Certificate 24,000
Restricted to 34,000 30,000
Total Income 1,20,000

Note: Though the value of eligible investments is ` 34,000, however, deduction under
Chapter VIA cannot exceed the gross total income exclusive of long term capital gains,
short-term capital gains covered under section 111A, winnings of lotteries etc of the
assessee.
Therefore, the maximum permissible deduction u/s 80C = ` 1,50,000 - ` 1,20,000 =
` 30,000.

Illustration 3: The gross total of Mr. X for the A.Y. 2021-22 is ` 5,00,000. He has made the
following investments/payments during the F.Y. 2020-21–
Particulars `
(1) Contribution to PPF 1,10,000
(2) Payment of tuition fees to Apeejay School, New Delhi, for 45,000
education of his son studying in Class XI
(3) Repayment of housing loan taken from Standard Chartered 25,000
Bank
(4) Contribution to approved pension fund of LIC 1,05,000
Compute the eligible deduction under Chapter VI-A for the A.Y. 2021-22
INCOME TAX 130

Solution:
Computation of deduction under Chapter VI-A for the A.Y. 2021-22
Particulars `
Deduction under section 80C
(1) Contribution to PPF – fully allowed, since it is within the limit 1,10,000
of ` 1,50,000
(2) Payment of tuition fees to Apeejay School, New Delhi, for 45,000
education of his son studying in Class XI
(3) Repayment of housing loan 25,000
1,80,000
Restricted to ` 1,50,000, being the maximum permissible deduction
u/s 80C 1,50,000
Deduction under section 80CCC
(4) Contribution to approved pension fund of LIC ` 1,05,000 1,05,000
2,55,000
As per section 80CCE, the aggregate deduction under section 80C,
80CCC and 80CCD(1) has to be restricted to ` 1,50,000
Deduction allowable under Chapter VIA for the A.Y. 2020-21 1,50,000

Illustration 4: Mr. A, aged 40 years, paid medical insurance premium of ` 20,000 during
the P.Y. 2020-21 to insure his health as well as the health of his spouse. He also paid medical
insurance premium of ` 37,000 during the year to insure the health of his father, aged 63
years, who is not dependent on him. He contributed ` 3,600 to Central Government Health
Scheme during the year. He has incurred ` 3,000 in such on preventive health check-up of
himself and his spouse and ` 4,000 by cheque on preventive health check-up of his father.
Compute the deduction allowable under section 80D for the A.Y. 2021-22.

Solution:
Deduction allowable under section 80D for the A.Y. 2021-22
Particulars Actual Maximum
Payment deduct ion
(`) allowable (`)
A. Premium paid and medical expenditure
incurred for self and spouse
(i) Medical insurance premium paid for self and 20,000 20,000
spouse
(ii) Contribution to CGHS 3,600 3,600
(iii) Exp. On preventive health check-up of self & 3,000 1,400
spouse
26,600 25,000
131 DEDUCTIONS

B. Premium paid and medical expenditure


incurred for father, who is a senior citizen
(i) Medi-claim premium paid for father, who is over 37,000 37,000
60 years of age
(ii) Expenditure on preventive health check-up of 4,000 3,600
father
41,000 40,600
Total deduction under section 80D 65,600
Notes:
(1) The total deduction under 4.(i), (ii) and (iii) above should not exceed ` 25,000.
Therefore, the expenditure on preventive health check-up for self and spouse would
be restricted to ` 1,400, being (` 25,000 - ` 20,000 - ` 3,600).
(2) In this case, the total deduction allowed on account of expenditure on preventive
health check-up of self, spouse and father is `5,000 (i.e., ` 1,400 + ` 3,600), which is
the maximum permissible limit.

Illustration 5: Mr. Y, aged 40 years, paid medical insurance premium of ` 22,000 during the
P.Y. 2020-21 to insure his health as well as the health of his spouse and dependent children.
He also paid medical insurance premium of ` 33,000 during the year to insure the health of
his father, aged 67 years, who is not dependent on him. He contributed ` 2,400 to Central
Government Health Scheme during the year. Compute the deduction allowable under
section 80D for the A.Y. 2021-22.

Solution: Deduction allowable under section 80D for the A.Y. 2021-22
Particulars `
(i) Medical insurance premium paid for self, spouse and dependent children 22,000
(ii) Contribution to CGHS 2,400
(iii) Medi-claim premium paid for father, who is over 60 years of age 33,000
57,400
Note – The total deduction under (i) and (ii) above should not exceed ` 25,000. In this case,
since the total of (i) and (ii) (i.e. ` 24,400) does not exceed ` 25,000, the same is fully
allowable under section 80D.
However, had the medical insurance premium paid for self, spouse and children been
` 24,000 instead of ` 22,000, then, the total of ` 26,400 (i.e., ` 24,000 + ` 2,400) under (i)
and (ii) above would be restricted to ` 25,000.

Illustration 6: Mr. B has taken three education loans on April 1, 2020, the details of which
are given below:
Loan 1 Loan 2 Loan 3
For whose education loan was taken B Son of B Daughter of B
Purpose of loan MBA B.Sc. B.A.
Amount of loan (`) 5,00,000 2,00,000 4,00,000
INCOME TAX 132

Annual repayment of loan (`) 1,00,000 40,000 80,000


Annual repayment of interest (`) 20,000 10,000 18,000
Compute the amount deductible under section 80E for the A.Y. 2021-22.
Solution: Deduction under section 80E is available to an individual assessee in respect of
any interest paid by him in the previous year in respect of loan taken for pursuing his
higher education of higher education of his spouse or children. Higher education means any
course of study pursued after senior secondary examination.
Therefore, interest repayment in respect of all the above loans would be eligible for
deduction.
Deduction under section 80E = ` 20,000 + ` 10,000 + ` 18,000 = ` 48,000.

Illustration 7: Mr. Shiva aged 61 years, has gross total income of ` 7,75,000 comprising of
income from salary and house property. He has made the following payments and
investments:
(i) Premium paid to insure the life of her major daughter (policy taken on 1.4.2014)
(Assured value ` 1,80,000) - ` 20,000.
(ii) Medical Insurance premium for self - ` 12,000; Spouse - ` 14,000.
(iii) Donation to a public charitable institution registered under 80G ` 1,50,000 by way
of cheque.
(iv) LIC Pension Fund - ` 60,000.
(v) Donation to National Children’s Fund - ` 25,000 by way of cheque
(vi) Donation to Jawaharlal Nehru Memorial Fund - ` 25,000 by way of cheque
(vii) Donation to approved institution for promotion of family planning - ` 40,000 by
way of cheque
Compute the total income of Mr. Shiva for A.Y. 2021-22

Solution:
Computation of Total Income of Mr. Shiva for A.Y. 2021-22
Particulars ` `
Gross Total Income 7,75,000
Less: Deduction under section 80C
Life insurance Premium paid for insurance of major 18,000
daughter (Maximum 10% of the assured value `
1,80,000, as the policy is taken after 31.3.2012)
Deduction under section 80CCC in respect of LIC 60,000 78,000
pension fund
Deduction under section 80D
Medical Insurance premium in respect of self and 26,000
spouse
Deduction under section 80G (See Working Note 91,050
below)
Total Income 5,79,950
133 DEDUCTIONS

Working Note: Computation of deduction under section 80G


Particulars of donation Amount % of deduction Deduction
donated (`) u/s 80G (`)
(i) National Children’s Fund 25,000 100% 25,000
(ii) Jawaharlal Nehru Memorial 25,000 50% 12,500
Fund
(iii) Approved institution for 40,000 100%, subject to 40,000
promotion of family planning qualifying limit
(iv) Public Charitable Trust 1,50,000 50% subject to 13,550
qualifying limit
(See Note below)
91,050
Note – Adjusted total income = Gross Total Income – Amount of deductions under section
80C to 80U except section 80G i.e., ` 6,71,000, in this case.
` 67,100, being 10% of adjusted total income is the qualifying limit, in this case.
Firstly, donation of ` 40,000 to approved institution for family planning qualifying for 100%
deduction subject to qualifying limit, has to be adjusted against this amount. Thereafter,
donation to public charitable trust qualifying for 50% deduction, subject to qualifying limit
is adjusted. Hence, the contribution of ` 1,50,000 to public charitable trust is restricted to
` 27,100 (being, ` 67,100 - ` 40,000), 50% of which would be the deduction under section
80G. Therefore, the deduction under section 80G in respect of donation to public charitable
trust would be ` 13,550, which is 50% of ` 27,100.

Illustration 8: Mr. A has commenced the business of manufacture of computers on


1.4.2020. He employed 350 new employees during the P.Y. 2020-21, the details of whom
are as follows –
No. of Date of Regular/Casual Total monthly
employees employment emoluments per
employees (`)
(i) 75 1.4.2020 Regular 24,000
(ii) 125 1.5.2020 Regular 26,000
(iii) 50 1.8.2020 Casual 25,000
(iv) 100 1.9.2020 Regular 24,000

The regular employees participate in recognized provident fund while the casual employees
do not. Compute the deduction, if any, available to Mr. A for A.Y. 2021-22, if the profits and
gains derived from manufacture of computers that year is ` 75 lakhs and his total turnover
is ` 2.16 crores.
What would be your answer if Mr. A has commenced the business of manufacture of apparel
on 1.4.2020?
INCOME TAX 134

Solution: Mr. A is eligible for deduction under section 80JJAA since he is subject to tax audit
under section 44AB for A.Y. 2021-22, as his total turnover from business exceeds ` 1 crore
and he has employed “additional employees” during the P.Y. 2020-21.
I If Mr. A is engaged in the business of manufacture of computers
Additional employee cost = ` 24,000 × 12 × 75 [See Working Note below] =
` 2,16,00,000
Deduction under section 80JJAA = 30% of ` 2,16,00,000 = ` 64,80,000.
Working Note:
Number of additional employees
Particulars No. of workmen
Total number of employees employed during the year 350
Less: Casual employees employed on 1.8.2020 who do not 50
participate in recognized provident fund
Regular employees employed on 1.5.2020, since their total 125
monthly emoluments exceed ` 25,000
Regular employees employed on 1.9.2020 since they have 100 275
been employed for less than 240 days in the P.Y. 2020-21.
Number of “additional employees” 75
Note – Since casual employees do not participate in recognized provident fund, they do
not qualify as additional employees. Further, 125 regular employees employed on
1.5.2020 also do not qualify as additional employees since their monthly emoluments
exceed ` 25,000. Also, 100 regular employees employed on 1.9.2020 do not qualify as
additional employees for the P.Y. 2020-21, since they are employed for less than 240
days in that year.
Therefore, only 75 employees employed on 1.4.2020 qualify as additional employees,
and the total emoluments paid or payable to them during the P.Y. 2020-21 is deemed to
be the additional employee cost.

II If Mr. A is engaged in the business of manufacture of apparel, footwear or leather.


If Mr. A is engaged in the business of manufacture of apparel, then, he would be entitled
to deduction under section 80JJAA in respect of employee cost of regular employees
employed on 1.9.2020, since they have been employed for more than 150 days in the
previous year 2020-21.
Additional employee cost = ` 2,16,00,000 + ` 24,000 × 7 × 100 = ` 3,84,00,000
Deduction under section 80JJAA = 30% of ` 3,84,00,000 = ` 1,15,20,000
135 DEDUCTIONS

MCQ

1. Mr. Srivastav, aged 72 years, paid medical insurance premium of ` 32,000 by cheque
and ` 4,000 by cash during May, 2020 under a Medical Insurance Scheme of the
General Insurance Corporation. The above sum was paid for insurance of his own
health. He would be entitled to a deduction under section 80D of a sum of –
(a) ` 25,000
(b) ` 32,000
(c) ` 20,000
(d) ` 36,000

2. Mr. Ramesh pays a rent of ` 5,000 per month. His total income is ` 2,80,000 (i.e.
Gross Total Income as reduced by deductions under Chapter VI-A except section
80GG). He is also in receipt of HRA. He would be eligible for a deduction under
section 80GG of an amount of –
(a) ` 60,000
(b) ` 32,000
(c) ` 70,000
(d) Nil

3. This deduction under section 80QQB in respect of royalty income of authors of


certain books is subject to a maximum limit of –
(a) ` 1,00,000
(b) ` 3,00,000
(c) ` 5,00,000
(d) ` 2,00,000

4. Under section 80GGB, deduction is allowable in respect of contribution to political


parties by –
(a) Any person other than local authority and every artificial juridical person wholly
or partly funded by the Government
(b) Local authority and every artificial juridical person wholly or partly funded by
the Government
(c) An Indian company
(d) Any assessee

5. As per section 80CCE, ` 1.5 lakh is the maximum qualifying limit for deduction
under–
(a) Section 80C and 80CCD
(b) Sections 80CCC and 80CCD
INCOME TAX 136

(c) Sections 80C, 80CCC and 80CCD(1)


(d) Sections 80C, 80CCC and 80CCD

6. Deduction u/s 80C in respect of LIP, Contribution to provident fund, etc. is allowed
to:
(a) Anyassessee
(b) An individual
(c) An individual of HUF
(d) An individual or HUF who is resident in India

7. An assessee has paid life insurance premium of ` 25,000 during the previous year
for a policy of ` 1,00,000 taken on 1.4.2016. He shall:
(a) Not be allowed deduction u/s 80C
(b) Be allowed deduction of ` 20,000 u/s 80C
(c) Be allowed deduction of ` 25,000 under section 80C
(d) Be allowed deduction of ` 10,000 u/s 80C

8. The payment for Insurance premium under section 80D should be paid:
(a) In cash
(b) By any mode other than cash
(c) Cash/by cheque
(d) Through account payee cheque/account payee bank draft

9. The maximum deduction allowable under section 80EE for A.Y. 2021-22 is –
(a) ` 50,000
(b) ` 2,50,000
(c) ` 1,00,000
(d) ` 1,50,000

10. The maximum amount which can be donated in cash for claiming deduction under
section 80G is –
(a) ` 5,000
(b) ` 10,000
(c) ` 1,000
(d) ` 2,000
6 TOTAL INCOME
What is total income and how is it computed
Total income of an assessee is gross total income as reduced by the amount permissible as
deduction under sections 80C to 80U. The scheme of computation of total income and tax
liability thereon can be easily understood with the help of the following chart:
COMPUTATION OF INCOME FOR AN ASSESSMENT YEAR
Particulars ` `
Income from Salaries
Income from salary …………
Income by way of allowances …………
Taxable value of perquisites …………
Gross Salary ……….
Less: Deduction under section 16
Standard Deduction …………
Entertainment allowance …………
Professional tax …………
Income from Salaries …………
Income from House Property
Gross Annual Value …………
Less: Municipal Taxes …………
Net annual value ………..
Less: Deduction under section 24 ………..
Income from House Property …………
Profits and Gains of Business or Profession
Net profit as per profit and loss account …………
Add: Amounts which are debited to P & L A/c but are not allowable as
Deduction under the Act …………
…………
Less: Expenditure which are not debited to P & L A/c but are allowable
as Deduction under the Act …………
…………
Less: Income which are credited to P & L A/c but are exempt under …………
section 10 or are taxable under other heads of income
Add: Those income which are not credited to P& L A/c but are taxable
under the head “profits and gains of business or profession” …………
Profits and Gains of Business or Profession ……….
138 INCOME TAX

Capital Gains
Short term Capital Gain …………
Long term Capital Gain …………

Income from Capital Gains …………


Income from Other Sources
Gross income …………
Less: Deduction under section 57 …………
Income from Other Sources …………
Total …………
Less: Adjustment on account of set-off and carry forward of losses …………
Gross Total Income …………
Less: Deductions under sections 80C to 80U …………
Total Income or Net Income …………

COMPUTATION OF TAX LIABILITY


Particulars `
Tax on Net Income …………
Less: Rebate u/s 87A or …………
Add: Surcharge …………
…………
Add: Health & Education cess [4 %] …………
Tax …………
Less: Pre-paid taxes
 Advance Tax …………
 TDS / TCS …………
 Self – Assessment Tax …………
Tax Liability …………

Illustration 1: Discuss the tax treatment of the following items which belong to different
taxpayers-
1. A salaried employee has received medical allowance of ` 18,000 which is fully used for
meeting medical expenses.
2. A salaried employee has received reimbursement of ` 18,000 on account of medical
facilities in a private clinic.
3. A salaried employee gets ` 2,000 per month as transport allowance for meeting
expenditure to cover the journey between office and residence (actual expenditure:
` 190 per month).
TOTAL INCOME 139

4. A company provides free conveyance to its employees for the journey between office
and residence.
5. X is employed by A Ltd. on salary of ` 25,000 per month. He holds 18% equity share
capital in A Ltd. He does not have any professional qualification to justify the
remuneration. Mrs. X holds 2% equity share capital in A Ltd.
6. A firm (having two equal partners) gets a loan of ` 40,000 from a private limited
trading company whose general reserve is more than its share capital of ` 20 lakh. X,
one of the partners of the firm, holds 10 % equity share capital in the company.
7. A prize of ` 15,000 received from a TV contest.
8. A gift received from father.
9. A gift received by a lawyer from his client in appreciation of his service.
10. Gift of ` 16,000 received by an individual from his friend. Another gift of ` 20,000 from
the same person is received later on in the same year.
11. Electricity bills of the year 2020-21 paid on December 1, 2021 by a manufacturing
company.
12. Bonus received from LIC at the time of maturity of an endowment policy.
13. Money received from LIC on the maturity of a Keyman insurance policy by a company.
14. Leave encashment received by the legal heirs of a deceased employee.
15. X (age: 32 years) dies on June 15, 2019 (normal date of retirement is March 31, 2031).
Gratuity is received by the legal heirs of X on July 2, 2019.
16. Share of profit received from a firm.
17. Interest received on income-tax refund.
18. Interest paid on capital borrowed for payment of income-tax.
19. Income received by a minor from stage acting.
20. Interest received by a minor from a bank in which he deposits his earning from stage
acting.
21. Loss from the activity of owning and maintaining race horses.
22. Loss from the activity of owning and maintaining camel for races.
23. A company pays LTC to its general manager (being air fare of the employee and family :
` 80,000 and boarding lodging expenses : ` 20,000).
24. A person borrows money at the rate of 8% per annum and the same is given as in
interest-free loan to his wife.
25. A person has agricultural income in India (` 20,000) as well as outside India (` 60,000).
26. A farmer transfers rural agricultural land in India at a price higher than its cost of
acquisition.
27. A farmer transfers rural agricultural land in Nepal at a price higher than its cost of
acquisition.
Solution:
1. Medical allowance is always chargeable to tax (there is no exception).
2. Reimbursement of medical expenses is fully taxable.
140 INCOME TAX

3. Transport allowance for meeting the expenditure between office and residence is fully
taxable regardless of actual expenditure. Therefore, in this case ` 2,000 per month will
be chargeable to tax.
4. It is not taxable. Conveyance facility for covering the distance between the office and
residence is not taken as perquisite chargeable to tax.
5. X does not have any technical/ professional qualification and Mrs. X has a substantial
interest in A Ltd. (i.e., shareholding of Mrs. X : 2% + shareholding of her husband who is
a “relative” : 18%), salary will be taxable in the hands of Mrs. X (irrespective of the fact
whether or not other incomes of Mrs. X is higher). If, however, Mrs. X is also employed
by A Ltd. without any technical/professional qualification, then the salary of X and Mrs.
X shall be taxable in the hands of X or Mrs. X whose other income is higher.
6. X has a substantial interest in the firm. He holds 10% equity share capital in the private
company. The company gives a loan of ` 40,000 to the firm out of accumulated profit.
The loan will be treated as Deemed dividend. The company will have to pay CDT on it.
7. It is taxable @ 30% + Cess.
8. Gift from father (or any other relative) is not income. There is no tax liability.
9. Gift from a client in appreciation of professional service is nothing but a professional
income.
10. Since the total gift received does not exceed ` 50,000 nothing is chargeable to tax.
11. It is deductible for the previous year 2020 -21. Section 43B is not applicable.
12. Bonus on an endowment policy is not an income chargeable to tax.
13. Money received from LIC on Keyman Insurance Policy is taxable as business income.
14. Leave encashment received by the legal heirs of a deceased employee is not taxable.
15. Gratuity received by the legal heirs of a deceased employee is not taxable. It cannot be
taxed in the hands of deceased employee as it becomes due and is paid after his death.
16. Share of profit received from a firm is exempt.
17. Interest received on income-tax refund is taxable under head Income from other
Sources.
18. Interest on capital borrowed for payment of income-tax is not deductible.
19. Income received by a minor child from stage acting is taxable as his own income.
20. Interest from a bank received by a minor child is taxable as income of his father or
mother whose other income is higher. This rule is applicable even if interest is received
on accumulated earnings from stage acting (income from stage acting is not taxable as
income of father/mother).
21. Loss from the activity of owning and maintaining race horses can be set off only from
income from the same activity. It can be carried forward for the next 4 years for being
set off against the same income.
22. Loss from the activity of owning and maintaining camels for races can be set off against
any income of the current year and unadjusted amount can be carried forward for
being set off against any business income of the next 8 assessment years.
TOTAL INCOME 141

23. While the company can claim deduction of ` 1 lakh under section 37(1), the general
manager will pay tax on ` 20,000.
24. As there is not transfer of asset, clubbing provisions are not applicable. Interest payable
may not be allowed as deduction.
25. Agricultural income in India is exempt under section 10(1). It is, however, taken into
account while finding out tax on non-agricultural income, if the recipient is an
individual, HUF, AOP/BOI.
Agricultural income from outside India is taken as non-agricultural income.
26. Agricultural land in India in a rural area is not a “capital asset”. Gain on its transfer is
not chargeable to tax.
27. The aforesaid rule is applicable only if the agricultural land is in India. Rural
agricultural land outside India is a capital asset and gain on its transfer is chargeable to
tax under section 45.

Illustration 2: Dr. Niranjana, a resident individual, aged 60 years is running a clinic. Her
Income and Expenditure Account for the year ending March 31st, 2021 is as under:
Expenditure ` Income `
To Medicine consumed 35,38,400 By Consultation and 58,85,850
medical charges
To Staff salary 13,80,000 By Income-tax refund 5,450
(Principal ` 5,000,
interest ` 450)
To Clinic consumables 1,10,000 By Dividend from units of 10,500
UTI
To Rent paid 90,000 By Winning from game 35,000
show on T.V. (net of
TDS of ` 15,000)
To Administrative expenses 2,55,000 By Rent 27,000
To Amount paid to scientific 1,50,000
research association
approved under section 35
To Net profit 4,40,400
59,63,800 59,63,800

(i) Rent paid includes ` 30,000 paid by cheque towards rent for her residential house in
Surat.
(ii) Clinic equipments are:
1.4.2020 Opening W.D.V. - ` 5,00,000
7.12.2020 Acquired (cost) by cheque - ` 2,00,000
142 INCOME TAX

(iii) Rent received relates to property situated of Surat. Gross Annual Value ` 27,000. The
municipal tax of ` 2,000, paid in December, 2020, has been included in
“administrative expenses”.
(iv) She received salary of ` 1,30,000 p.a. from “Full Cure Hospital” which has not been
included in the “consultation and medical charges”.
(v) Dr. Niranjana availed a loan of ` 5,50,000 from a bank for higher education of her
daughter. She repaid principal of ` 1,00,000, and interest thereon ` 55,000 during the
year 2020-21.
(vi) She paid ` 1,00,000 as tuition fee (not in the nature of development fees/ donation)
to the university for full time education of her daughter.
(vii) An amount of ` 28,000 has also been paid by cheque on 27th March, 2021 for her
medical insurance premium.
From the above, compute the total income of Dr. Smt. Niranjana for the A. Y. 2021-22.
Solution:
Computation of total income and tax liability of Dr. Niranjana for A.Y. 2021-22
Particulars ` ` `
I Income from Salary
Basic Salary – Std. deduction (1,30,000-50,000) 80,000
II Income from house property
Gross Annual Value (GAV) 27,000
Less: Municipal taxes paid 2,000
Net Annual Value (NAV) 25,000
Less: Deduction u/s 24 @ 30% of ` 25,000 7,500 17,500
III Income from profession
Net profit as per Income and Expenditure 4,40,400
account
Less: Items of income to be treated
separately
(i) Rent received 27,000
(ii) Dividend from units of UTI 10,500
(iii) Winning from game show on T.V. (net 35,000
of TDS)
(iv) Income tax refund 5,450 77,950
3,62,450
Less : Allowable expenditure
Depreciation on Clinic equipments
On ` 5,00,000 @ 15% 75,000
On ` 2,00,000 @ 7.5% 15,000
(On equipments acquired during the year
in December 2019 she is entitled to
TOTAL INCOME 143

depreciation @ 50% of normal


depreciation, since the same are put to use
for less than 180 days during the year)
Additional deduction of 50% for amount
paid to scientific research association
(Since weighted deduction of 150% is
available in respect of such payment) 75,000 1,65,000
1,97,450
Add: Items of expenditure not allowable
while computing business income
(i) Rent for her residential 30,000
accommodation included in Income
and Expenditure A/c
(ii) Municipal tax paid relating to
residential houses at Surat included in 2,000 32,000 2,29,450
administrative expenses
IV Income from other sources
(a) Interest on income-tax refund 450
(b) Dividend from UTI 10,500
Less: Exempt under section 10(35) 10,500 Nil
(c) Winnings from the game show on T.V.
(` 35,000 + ` 15,000) 50,000 50,450
Gross Total Income 3,77,400
Less: Deductions
(a) Section 80C – Tuition fee paid to
university for full time education of 1,00,000
her daughter
(b) Section 80D – Medical insurance
premium (fully allowed since she is a 28,000
senior citizen)
(c) Section 80E – Interest on loan taken
for higher education is deductible 55,000 1,83,000

Total Income 1,94,400


Notes:
(i) The principal amount received towards income-tax refund will be excluded from
computation of total income. Interest received will be taxed under the head “Income
from other sources”.
(ii) Winnings from game show on T.V. should be grossed up for the chargeability under
the head “Income from other sources” (` 35,000 + ` 15,000). Thereafter, while
144 INCOME TAX

computing tax liability, TDS of ` 15,000 should be deducted to arrive at the tax
payable. Winnings from game show are subject to tax @ 30% as per section 115BB.
(iii) Since Dr. Niranjana is staying in a rented premise in Surat itself, she would not be
eligible for deduction u/s 80GG, as she owns a house in Surat which she has let out.

Illustration 3: Ms. Purvi, aged 55 years, is a Chartered Accountant in practice. She


maintains her accounts on cash basis. Her Income and Expenditure account for the year
ended March 31, 2021 reads as follows:

Expenditure (`) Income (`) (`)


Salary to staff 15,50,000 Fees earned:
Stipend to articled Audit 27,88,000
assistants 1,37,000 Taxation services 15,40,300
Incentive to articled Consultancy 12,70,000 55,98,300
assistants 13,000 Dividend on shares
Office rent 12,24,000 of Indian companies
Printing and stationery 12,22,000 (Gross) 10,524
Meeting, seminar and Income from UTI 7,600
Conference 31,600 Honorarium
Purchase of car 80,000 received from
Repair, maintenance and various institutions
petrol of car 4,000 for valuation of
Travelling expenses 5,25,000 answer papers 15,800
Municipal tax paid in Rent received from
respect of house property 3,000 residential flat let
Net Profit 9,28,224 out 85,600
57,17,824 57,17,824

Other Information:
(i) Allowable rate of depreciation on motor car is 15%.
(ii) Value of benefits received from clients during the course of profession is ` 10,500.
(iii) Incentives to articled assistants represent amount paid to two articled assistants for
passing Find Examination in first attempt.
(iv) Repairs and maintenance of car include ` 2,000 for the period from 1-10-2019 to 30-
09-2020.
(v) Salary include ` 30,000 to a computer specialist in cash for assisting Ms. Purvi in one
professional assignment.
(vi) The travelling expenses include expenditure incurred on foreign tour of ` 32,000
which was within the RBI norms.
TOTAL INCOME 145

(vii) Medical Insurance Premium on the health of dependent brother and major son
dependent on her amounts to ` 5,000 paid in cash and ` 10,000 in National Saving
Certificate.
Compute the total income and tax payable of Ms. Purvi for the assessment year 2021-22.

Solution:
Computation of total income and tax liability of Ms. Purvi for the A.Y. 2021-22
Particulars ` `
Income from house property (See Working Note 1) 57,820
Profit and gains of business or profession (See Working Note 2) 9,20,200
Income from other sources (See Working Note 3) 33,924
Gross Total Income 10,11,944
Less: Deductions under Chapter VI-A (See Working Note 4) 10,000
Total Income 10,01,944
Tax on total income
Upto ` 2,50,000 Nil
` 2,50,001 - ` 5,00,000 @5% 12,500
` 5,00,001 - ` 10,00,000 @20% 1,00,000
` 10,00,001 - ` 10,01,944 @ 30% 583 1,13,083
Add: Health & Education cess @ 4% 4,523
Total tax liability 1,17,606
Tax Payable 1,17,610

Working Notes:

(1) Income from House Property


Particulars ` `
Gross Annual Value under section 23(1) 85,600
Less: Municipal taxes paid 3, 000
Net Annual Value (NAV) 82,600
Less: Deduction under section 24 @ 30% of NAV 24,780 57,820

Note – Rent received has been taken as the Gross Annual Value in the absence of other
information relating to Municipal Value, Fair Rent and Standard Rent.
146 INCOME TAX

(2) Income under the head “Profit & Gains of Business or Profession”
Particulars ` `
Net profit as per Income and Expenditure account 9,28,224
Add: Expenses debited but not allowable
(i) Salary paid to computer specialist in cash disallowed
under section 40A(3), since such cash payment exceeds 30,000
` 10,000
(ii) Amount paid for purchase of car is not allowable under
section 37(1) since it is a capital expenditure 80,000
(iii) Municipal Taxes paid in respect of residential flat let 3,000 1,13,000
out
10,41,224
Add: Value of benefit received from clients during the course
of profession [taxable as business income under section 10,500
28(iv)]
10,51,724
Less: Income credited but not taxable under this head:
(i) Dividend on shares of Indian companies 10,524
(ii) Income from UTI 7,600
(iii) Honorarium for valuation of answer papers 15,800
(iv) Rent received from letting out of residential flat 85,600 1,19,524
9,32,200
Less: Depreciation on motor car @15% (See Note (i) below) 12,000
9,20,200
Notes:
(i) It has been assumed that the motor car was put to use for more than 180 days during
the previous year and hence, full depreciation @ 15% has been provided for under
section 32(1)(ii).
Note: Alternatively, the question can be solved by assuming that motor car has been
put to use for less than 180 days and accordingly, only 50% of depreciation would be
allowable as per the second proviso below section 32(1)(ii).
(ii) Incentive to articled assistants for passing IPCC examination in their first attempt is
deductible under section 37(1).
(iii) Repairs and maintenance paid in advance for the period 1.4.2020 to 30.9.2020 i.e. for
6 months amounting to ` 1,000 is allowable since Ms. Purvi is following the cash
system of accounting.
(iv) ` 32,000 expended on foreign tour is allowable as deduction assuming that it was
incurred in connection with her professional work. Since it has already been debited
to income and expenditure account, no further adjustment is required.
TOTAL INCOME 147

(3) Income from other sources


Particulars `
Dividend on shares of Indian companies 10,524
Income from UTI 7,600
Honorarium for valuation of answer papers 15,800
33,924

(4) Deduction
Particulars `
Deduction under section 80C (Investment in NSC) 10,000
Deduction under section 80D (See Notes (i) & (ii) below) Nil
Total deduction 10,000
Notes:
(i) Premium paid to insure the health of brother is not eligible for deduction under
section 80D, even though he is a dependent, since brother is not included in the
definition of “family” under section 80D.
(ii) Premium paid to insure the health of major son is not eligible for deduction, even
though he is a dependent, since payment is made in cash.

Illustration 4: Mr. Y carries on his own business. An analysis of his trading and profit & loss
for the year ended 31-3-2021 revealed the following information:
(1) The net profit was ` 11,20,000.
(2) The following incomes were credited in the profit and loss account:
(a) Dividend from UTI ` 22,000.
(b) Interest on debentures ` 17,500.
(c) Winnings from races ` 15,000.
(3) It was found that some stocks were omitted to be included in both the opening and
closing stocks, the value of which were:
Opening stock ` 8,000
Closing stock ` 12,000.
(4) ` 1,00,000 was debited in the profit and loss account, being contribution to a
University approved and notified under section 35(1)(ii).
(5) Salary includes ` 20,000 paid to his brother which is unreasonable to the extent of
` 2,500.
(6) Advertisement expenses include 15 gifts packets of dry fruits costing ` 1,000 per
packet presented to important customers.
(7) Total expenses on car was ` 78,000. The car was used both for business and personal
purposes. 3/4th is for business purposes.
(8) Miscellaneous expenses included ` 30,000 paid to A & Co., a goods transport operator
in cash on 31-1-2020 for distribution of the company’s product to the warehouses.
148 INCOME TAX

(9) Depreciation debited in the books was ` 55,000. Depreciation allowed as per Income-
tax Rules, 1962 was ` 50,000.
(10) Drawings ` 10,000.
(11) Investment in NSC ` 15,000.
Compute the tot al income of Mr. Y for the assessment year 2021-22.
Solution:
Computation of total income of Mr. Y for the A.Y. 2021-22
Particulars `
Profits and gains of business or profession (See Working Note 1 below) 10,71,500
Income from other sources (See Working Note 2 below) 54,500
Gross Total Income 11,26,000
Less: Deduction under section 80C (Investment in NSC) 15,000
Total Income 11,11,000

Working Notes:
1. Computation of profits and gains of business or profession
Particulars ` `
Net profit as per profit and loss account 11,20,000
Add: Expenses debited to profit and loss account but not
allowable as deduction
Salary paid to brother disallowed to the extent considered 2,500
unreasonable [Section 40A(2)]
Motor car expenses attributable to personal use not 19,500
allowable (` 78,000 × ¼)
Depreciation debited in the books of account 55,000
Drawings (not allowable since it is personal in nature) 10,000
[See Note (III)]
Investment in NSC [See Note (iii)] 15,000 1,02,000
12,22,000
Add: Under statement of closing stock 12,000
12,34,000
Less: Under statement of opening stock 8,000
12,26,000
Less: Contribution to a University approved and notified under
section 35(1)(ii) is eligible for weighted deduction@
150%. Since only the actual contribution (100%) has been
debited to profit and loss account, the additional 50% has 50,000
to be deducted.
11,76,000
Less: Incomes credited to profit and loss account but not taxable
TOTAL INCOME 149

as business income
Income from UTI (taxable under the head “Income from
other sources”) 22,000
Interest on debentures (taxable under the head “Income 17,500
from other sources”)
Winnings from races (taxable under the head “Income
from other sources”) 15,000 54,500
10,21,500
Less: Depreciation allowable under the Income-tax Rules, 1962 50,000
10,71,500
Notes:
(i) Advertisement expenses of revenue nature, namely, gift of dry fruits to important
customers, is incurred wholly and exclusively for business purposes. Hence, the same
is allowable as deduction under section 37.
(ii) Disallowance under section 40A(3) is not attracted in respect of cash payment
exceeding ` 10,000 to A & Co., a goods transport operator, since, in case of payment
made for plying, hiring or leasing goods carriages, an increased limit of ` 35,000 is
applicable (i.e. payment of upto ` 35,000 can be made in cash without attracting
disallowance under section 40A(3))
(iii) Since drawings and investment in NSC have been given effect to in the profit and loss
account, the same have to be added back to arrive at the business income.

2. Computation of “Income from other sources”


Particulars `
Interest on debentures 17,500
Winnings from races 15,000
Income from UTI 22,000
54,500

Notes:
The following assumptions have been made in the above solution:
1. The figures of interest on debentures and winnings from races represent the gross
income (i.e., amount received plus tax deducted at source).
2. In point no. 9 of the question, it has been given that depreciation as per Income-tax
Rules, 1962 is ` 50,000. It has been assumed that, in the said figure of ` 50,000, only
the proportional depreciation (i.e., 75% for business purposes) has been included in
respect of motor car.
150 INCOME TAX

Illustration 5: Balamurugan furnishes the following information for the year ended 31-03-
2021:
Particulars `
Income from business (1,35,000)
Income from house property (15,000)
Lottery winning (Gross) 5,00,000
Speculation business income 1,00,000
Net salary 60,000
Long term capital gain 70,000
Compute his total income & tax liability.
Solution:
Computation of total income of Balamurugan for the year ended 31.03.2021
Particulars ` `
Salaries 60,000
Less: Loss from house property (15,000)
Net Salary (after set off of loss from house property) 45,000
Profits and gains of business of profession
Speculation business income 1,00,000
Less: Business loss set-off (1,35,000)
Net business loss to be set-off against long-term capital gain (35,000)
Capital Gains
Long term capital gain 70,000
Less: Business loss set-off (35,000)
Long term capital gain after set off of business loss 35,000
Income from other sources
Lottery winnings (Gross) 5,00,000
Total Income 5,80,000

Computation of tax liability


Particulars `
On total income of ` 80,000 (excluding lottery winning) Nil
On lottery winnings of ` 5,00,000 @ 30% 1,50,000
Add: Health & Education Cess @ 4% 6,000
Total tax liability 1,56,000
Notes:
1. The basic exemption limit of ` 2,50,000 has to be first exhausted against salary income
of ` 45,000. The unexhausted basic exemption limit of ` 2,05,000 cannot be adjusted
against lottery winnings which are taxable at a flat rate of 30% under section 115BB.
2. The first proviso to section 234C(1) provides that since it is not possible for the
assessee to estimate his income from lotteries, the entire amount of tax payable (after
TOTAL INCOME 151

considering TDS) on such income should be paid in the remaining installments of


advance tax which are due. Where no such installment is due, the entire tax should be
paid by 31st March, 2020. The first proviso to section 234C(1) would be attracted only
in case of non-deduction or short-deduction of tax at source under section 194B.

Illustration 6: Mr. Rajiv, aged 50 years, a resident individual and practicing Chartered
Accountant, furnishes you the receipts and payments account for the financial year 2020-
21.
Receipts and Payments Account
Receipts ` Payments `
Opening balance 12,000 Staff salary, bonus and stipend to 21,50,000
(1.4.2020) Cash on hand articled clerks
and at Bank Other administrative expenses 11,48,000
Fee from professional 59,38,000 Office rent 30,000
services Housing loan repaid to SBI 1,88,000
Rent 50,000 (includes interest of ` 88,000)
Motor car loan from Life insurance premium 24,000
Canara Bank (@ 9% p.a.) 2,50,000 Motor car (acquired in Jan. 2021
by cheque) 4,25,000
Medical insurance premium (for
self and wife) 18,000
Books bought (annual
publications by cheque) 20,000
Computer acquired on 1.11.2020
(for professional use) 30,000
Domestic drawings
Public provident fund 2,72,000
subscription 20,000
Motor car maintenance
Closing balance (31.3.2021) 10,000
Cash on hand and at Bank 19,15,000
62,50,000 62,50,000

Following further information is given to you:


(1) He occupies 50% of the building for own residence and let out the balance for
residential use at a monthly rent of ` 5,000. The building was constructed during the
year 1997-98, when the housing loan was taken.
(2) Motor car was put to use both for official and personal purpose. One-fifth of the motor
car use is for personal purpose. No car loan interest was paid during the year
(3) The written down value of assets as on 1-4-2020 are given below:
152 INCOME TAX

Furniture & Fittings ` 60,000


Plant & Machinery ` 80,000
(Air-conditioners, Photocopiers, etc.)
Computers ` 50,000
Note: Mr. Rajiv follows regularly the cash system of accounting.
Compute the total income of Mr. Rajiv for the assessment year 2021-22.
Solution:
Computation of total income of Mr. Rajiv for the assessment year 2021-22
Particulars ` ` `
Income from house property
Self-occupied
Annual value Nil
Less: Deduction under section 24(b)
Interest on housing loan
50% of ` 88,000 = 44,000 but limited to 30,000
Loss from self occupied property (30,000)
Let out property
Annual value (Rent receivable has been taken
as the annual value in the absence of other 60,000
information)
Less: Deductions under section 24
30% of Net Annual Value 18,000
Interest on housing loan
(50% of ` 88,000) 44,000 62,000 (2,000)
Loss from house (32,000)
Profits and gains of business or profession
Fees from professional services 59,38,000
Less: Expenses allowable as deduction
Staff salary, bonus and stipend 21,50,000
Other administrative expenses 11,48,000
Office rent 30,000
Motor car maintenance (10,000 × 4/5) 8,000
Car loan interest – not allowable (since the
same has not been paid and the assessee
follows cash system of accounting) Nil 33,36,000
26,02,000
Less Depreciation
Motor car ` 4,25,000 × 7.5% × 4/5 25,500
Books being annual publications @ 40% 8,000
Furniture and fitting @ 10% of ` 60,000 6,000
TOTAL INCOME 153

Plant and machinery @ 15% of ` 80,000 12,000


Computer @ 40% of ` 50,000 20,000
Computer (New) ` 30,000 @ 40% × ½ 6,000 77,500 25,24,500
thereon
Gross Total income 24,92,500
Less: Deduction
Deduction under section 80C
Housing loan principal repayment 1,00,000
PPF subscription 20,000
Life insurance premium 24,000
Total amount of ` 1,44,000 is allowed as 1,44,000
deduction since it is within the limit of `
1,50,000
Deduction under section 80D
Medical insurance premium paid ` 18,000 18,000 1,62,000
Total income 23,30,500

Illustration 7: From the following details, compute the total income of Siddhant of Delhi for
A.Y. 2021-22:
Particulars `
Salary including dearness allowance 3,35,000
Bonus 11,000
Salary of servant provided by the employer 12,000
Rent paid by Siddhant for his accommodation 49,600
Bills paid by the employer for gas, electricity and water provided free of cost at 11,000
the above flat
Siddhant purchased a flat in a co-operative housing society in Delhi for ` 4,75,000 in April,
2013, which was financed by a loan from Life Insurance Corporation of India of ` 1,60,000
@ 15% interest, his own savings of ` 65,000 and a deposit from a nationalized bank for `
2,50,000 to whom this flat was given on lease for ten years. The rent payable by the bank
was ` 3,500 per month. The following particulars are relevant:
(a) Municipal taxes paid by Mr. Siddhant ` 4,300 (per annum)
(b) House Insurance ` 860
(c) He earned ` 2,700 in share speculation business and loss ` 4,200 in cotton speculation
business.
(d) In the year 2014-15, he had gifted ` 30,000 to his wife and ` 20,000 to his son who was
aged 11. The gifted amounts were advanced to Mr. Rajesh, who was paying interest @
19% per annum.
(e) Siddhant received a gift of ` 25,000 each from four friends.
(f) He contributed ` 50,000 to Public Provident Fund.
154 INCOME TAX

Solution: Computation of total income of Siddhant for the A.Y. 2021-22


Particulars ` `
Salary Income
Salary including dearness allowance 3,35,000
Bonus 11,000
Value of perquisites:
(i) Salary of servant 12,000
(ii) Free gas, electricity and water 11,000 23,000
3,69,000
Income from house property
Gross Annual Value (GAV) (Rent receivable is 42,000
taken as GAV in the absence of other information
(` 3,500 × 12)
Less: Municipal taxes paid 4,300
Net Annual Value (NAV) 37,700
Less: Deductions under section 24
(i) 30% of NAV ` 11,310
(ii) Interest on loan from LIC @15% of
` 1,60,000 [See Note 2] ` 24,000 35,310 2,390
Income from speculative business
Income from share speculation business 2,700
Less: Loss from cotton speculation business 4,200
Net Loss 1,500
Net loss from speculative business has to be
carried forward as it cannot be set off against any
other head of income.
Income from Other Sources
(i) Income on account of interest earned from 3,800
advancing money gifted to his minor son is
includible in the hands of Siddhant as per
section 64(1A)
Less: Exempt under section 10(32) 1,500
2,300
(ii) Interest income earned from advancing
money gifted to wife has to be clubbed with
the income of the assessee as per section
64(1) 5,700
(iii) Gift received from four friends (taxable
under section 56(2)(x) as the aggregate
amount received during the year exceeds
TOTAL INCOME 155

` 50,000) 1,00,000 1,08,000


Gross Total Income 4,79,390
Less: Deduction under section 80C
Contribution to Public Provident Fund 50,000
Total Income 4,29,390

Notes:
(1) It is assumed that the entire loan of ` 1,60,000 is outstanding as on 31.3.2021;
(2) Since Siddhant’s own flat in a co-operative housing society, which he has rented out to
a nationalized bank, is also in Delhi, he is not eligible for deduction under section
80GG in respect of rent paid by him for his accommodation in Delhi, since one of the
conditions to be satisfied for claiming deduction under section 80GG is that the
assessee should not own any residential accommodation in the same place.

Illustration 8: Ramdin working as Manager (Sales) with Frozen Foods Ltd., provides the
following information for the year ended 31.03.2021:
- Basic Salary ` 15,000 p.m.
- DA (50% of it is meant for retirement benefits) ` 12,000 p.m.
- Commission as a percentage of turnover of the Company 0.5%
- Turnover of the Company ` 50 lacs
- Bonus ` 50,000
- Gratuity ` 30,000
- Own Contribution to R.P.F. ` 30,000
- Employer’s contribution to R.P.F. 20% of basic salary
- Interest credited in the R.P.F. account @ 15% p.a. ` 15,000
- Gold Ring worth ` 10,000 was given by employer on his 25 wedding anniversary.
th

- Music System purchased on 01.04.2020 by the company for ` 85,000 and was given to
him for personal use.
- Two old heavy goods vehicles owned by him were leased to a transport company
against the fixed charges of ` 6,500 p.m. Books of account are not maintained.
- Received interest of ` 5,860 on bank FDRs, dividend of ` 1,260 from shares of Indian
Companies and interest of ` 7,540 from the debentures of Indian Companies.
- Made payment by cheques of ` 15,370 towards premium of Life Insurance policies and
` 12,500 for Mediclaim Insurance policy.
- Invested in NSC ` 30,000 and in FDR of SBI for 5 years ` 50,000.
- Donations of ` 11,000 to an institution approved u/s 80G and of ` 5,100 to Prime
Minister’s National Relief Fund were given during the year by way of cheque.
Compute the total income for the A.Y. 2021-22.
156 INCOME TAX

Solution: Computation of Total Income for the A.Y. 2021-22


Particulars ` `
Income from Salaries
Basic Salary (` 15,000 × 12) 1,80,000
Dearness Allowance (` 12,000 × 12) 1,44,000
Commission on Turnover (0.5% of ` 50 lacs) 25,000
Bonus 50,000
Gratuity (Note 1) 30,000
Employer’s contribution to recognized provident fund
Actual contribution [20% of ` 1,80,000] 36,000
Less: Exempt (Note 2) 33,240 2,760
Interest credited in recognized provident fund account 15,000
@15% p.a.
Less: Exempt upto 9.5% p.a. 9,500 5,500
Gift of gold ring worth ` 10,000 on 25th wedding 5,000
anniversary by employer (See Note 3)
Perquisite value of music system given for personal use
(being 10% of actual cost) i.e. 10% of ` 85,000 8,500
Gross Salary 4,55,760
(-) Standards deduction 50,000
Profits and Gains of Business or Profession 4,05,760
Lease of 2 trucks on contract basis against fixed charges of
` 6,500 p.m. In this case, presumptive tax provisions of 1,80,000
section 44AE will apply i.e. ` 7,500 p.m. for each of the two
trucks (` 7,500 × 2 × 12). He cannot claim lower pro fits
and gains since he has not maintained books of account.
Income from Other Sources
Interest on bank FDRs 5,860
Interest from debentures 7,540
Dividend on shares 1,260 14,660
Gross total Income 6,00,420
Less: Deductions under Chapter VI-A
Section 80C
Premium on life insurance policy 15,370
Investment in NSC 30,000
FDR of SBI for 5 years 50,000
Employee’s contrition to recognized provident fund 30,000 1,25,370
Section 80D – Mediclaim Insurance 12,500
Section 80G (Note 4) 10,600
Total Income 4,51,950
TOTAL INCOME 157

Notes:
1. Gratuity received during service is fully taxable.
2. Employer’s contribution in the recognized provident fund is exempt up to 12% of the
salary i.e. 12% of (Basic Salary + DA for retirement benefits + Commission based on
turnover)
= 12% of (` 1,80,000 + (50% of ` 1,44,000) + ` 25,000)
= 12% of 2,77,000 = ` 33,240
3. Deduction under section 80G is computed as under:
Particulars `
Donation to PM National Relief Fund (100%) 5,100
Donation to institution approved under section 80G (50% of ` 11,000) 5,500
(amount contributed ` 11,000 or 10% of Adjusted Gross Total Income
i.e. ` 46,255, whichever is lower)
Total deduction 10,600
Adjusted Gross Total Income = Gross Total Income – Deductions under section 80C
and 80D = ` 6,00,420 - ` 1,37,870 = ` 4,62,550

Illustration 9: Mr. Ashok owns a property consisting of two blocks of identical size. The
first block is used for business purposes. The other block has been let out from 1.4.2020. to
his cousin for ` 10,000 p.m. The cost of construction of each block is ` 5 lacs (fully met from
bank loan), rate of interest on bank loan is 10% p.a. The construction was completed on
31.3.2020. During the year ended 31.3.2021, he had to pay a penal interest of ` 2,000 in
respect of each block on account of delayed payments to the bank for the borrowings. The
normal interest paid by him in respect of each block was ` 42,000. Principal repayment for
each block was ` 23,000 made at the end of the year. An identical block in the same
neighbourhood fetches a rent of ` 15,000 per month. Municipal tax paid in respect of each
block was ` 12,000.
The income computed in respect of business prior to adjustment towards depreciation on
any asset is ` 2,20,000.
Depreciation on equipments used for business is ` 30,000.
On 23.3.2021, he sold shares of B Ltd., a listed share in BSE for ` 2,30,000. The share had
been purchased 10 months back for ` 1,80,000. Securities transaction tax at sale paid may
be taken as ` 220.
Brought forward business loss of a business discontinued on 12.1.2020 is ` 80,000. This
loss has been determined in pursuance of a return of income filed in time and the current
year is the seventh year.
The following payments were effected by him during the year;
(i) LIP of ` 20,000 on his life and ` 12,000 for his son aged 22, engaged as a software
engineer and drawing salary of ` 25,000 p.m.
158 INCOME TAX

(ii) Mediclaim premium of ` 6,000 for himself and ` 5,000 for above son. The premiums
were paid by cheque.
You are required to compute the total income for the assessment year 2021-22. The various
heads of income should be property shown. Ignore the interest on bank loan for the period
prior to 1.4.2020, as the bank had waived the same.
Solution:
Computation of total income of Mr. Ashok for the A.Y. 2021-22
Particulars ` ` `
Income from house property [See Note I]
House block 2 let out (higher of fair rent and 1,80,000
rent receivable)
Less: Municipal tax paid 12,000
Net annual value (NAV) 1,68,000
Less: Deductions under section 24
(a) 30% of NAV 50,400
(b) Interest on bank loan @ 10% on 50,000 1,00,400 67,600
` 5,00,000

Profits and gains of business or profession


[See Note II]
Income prior to adjustment for depreciation 2,20,000
Less: Depreciation on equipments used for
business 30,000
Depreciation on building ` 5,00,000 @ 10%
50,000 80,000
Less; Set off of brought forward business loss 1,40,000
relating to discontinued business
[See Note III] 80,000 60,000
Capital Gains [See Note IV]
Short term capital gains from sale of listed
shares
Full value of consideration 2,30,000
Less: Cost of acquisition 1,80,000 50,000
Gross Total Income 1,77,600
Less: Deduction under section 80C in respect
of LIP ` 32,000 and housing loan repayment in
respect of II block ` 23,000 55,000
Deduction under section 80D (for self) 6,000 61,000
Total Income 1,16,600
TOTAL INCOME 159

Notes:
I- On computation of Income from house property
(i) The annual value of the house property which is used for business would not
fall under the head “Income from house property”. Therefore, the annual value
of the first block is not chargeable to tax under the head “Income from house
property”. However, depreciation there on at 10% has been claimed while
computing the income from business.
(ii) As regards the second block, the sum for which the property may be reasonably
expected to be let is ` 15,000 per month. The Gross Annual Value (GAV) of the
block is the higher of fair rent (i.e., ` 15,000 p.m.) or the actual rent received
(` 10,000 p.m.) Hence, the GAV of the second block is ` 1,80,000 (i.e. ` 15,000
p.m.)
(iii) Under section 24(b), interest on bank loan for construction of house is
deductible. However, penal interest is not deductible. Interest due during the
year in respect of the second block is ` 50,000 p.m.) (i.e. 10% of ` 5 lakhs),
which is allowable as deduction under section 24(b).
II- On computation of Profits and gains of business or profession: Mr. Ashok can
claim depreciation @ 10% on the building used by him for business purposes. The
depreciation on the first block is ` 50,000 (being 10%, of ` 5,00,000) and depreciation
on equipments used for business is ` 30,000. Hence the depreciation allowable during
the year is ` 80,000.
III- On set off of business loss: As per section 72, business loss relating to discontinued
business is eligible for set off.
IV- On treatment of short-term capital gains (STCG): The listed shares have been sold
and securities transaction tax is paid, hence it is taxable at 15% as per section 111A.
For the purpose of providing deduction under Chapter VI, the gross total income
should be reduced by the STCG on listed shares.
V- On computation of deductions under sections 80C and 80D: Deduction under
section 80C can be claimed in respect of life insurance premium paid for major son,
even though he is not dependent on the assessee. It is assumed Block 2 let out to
cousin was used for residential purpose and accordingly principal repayment was
considered for deduction under section 80C.
However, deduction under section 80D cannot be claimed in respect of mediclaim
premium paid for non-dependant son. Mediclaim premium paid for self of ` 6,000 is
eligible for deduction.
160 INCOME TAX

Illustration 10: Mr. Venus provides the following details for the previous year ending
31-3-2021
(i) Salary from HNL Ltd. ` 50,000 per month
(ii) Interest on FD with SBI for the financial year 2020-21 ` 96,000
(iii) Brought forward long term capital loss of A Y 2018-19 ` 96,000
(iv) Long term capital gain ` 75,000
(v) Loss of minor son ` 90,000 computed in accordance with the provisions of income-
tax Act, 1961. Mr. Venus transferred his own house to his minor son without
adequate consideration few years back and minor son let it out and suffered loss.
(vi) Loss of his wife’s business ` (2,00,000)
She carried business with funds which Mr. Venus gifted to her.
You are required to compute taxable income of Mr. Venus for the AY 2021-22.
Solution:
Computation of Taxable Income of Mr. Venus for the A.Y. 2021-22
Particulars ` `
Salaries
Income from Salary (` 50,000 × 12) 6,00,000
Less: Loss from house property in respect of which Mr.
Venus is the deemed owner to be set off against his salary
income as per section 71(1) [See Note 1] 90,000 5,10,000
Capital Gains
Long term capital gain 75,000
Less: Brought forward long term capital loss of A.Y.
2018-19 set off against current year long-term capital gain Nil
as per section 74(1) & (2) [See Note 2] 75,000
Balance long-term capital loss of ` 21,000 (` 96,000 - `
75,000) of A.Y. 2018-19 to be carried forward to A.Y. 2022-
23 [See Note 2]
Income from Other Sources
Interest on fixed deposit with SBI (` 72,000 × 100/90) 80,000
Less: Business loss incurred by wife includible in Mr. 80,000 Nil
Venus’s hands set off against interest income as per section
71(1) [See Notes 3 & 4]
Balance business loss of ` 1,20,000 (` 2,00,000 - ` 80,000)
to be carried forward to A.Y. 2022-23
Taxable Income 5,10,000
Notes:
(1) As per section 27(i), Mr. Venus is the deemed owner of the house transferred to his
minor son without adequate consideration. Hence, the income from house property
would be assessable in Mr. Venus’s hands. Since there is a loss from house property
TOTAL INCOME 161

transferred to minor son without adequate consideration, Mr. Venus can set-off the
same against salary income, since he is the deemed owner of such property.
(2) As per section 74(1) and 74(2), brought forward long-term capital loss can be set-off
only against long-term capital gains. Unabsorbed long-term capital loss can be
carried forward for a maximum of eight assessment years (upto A.Y. 2025-26, in this
case) for set-off against long-term capital gains.
(3) As per section 64(1)(iv), income from funds gifted to spouse by an individual and
invested in business by the spouse is includible in the hands of t he individual. As per
Explanation 2 to section 64, income includes “loss”. Hence, in the given case, loss
arising out of t he business carried on by Mr. Venus’s wife is to be included in the
income of Mr. Venus, as she has carried on business with the funds gifted to her by
Mr. Venus.
(4) As per section 71(2A), business loss cannot be set-off against salary income.
However, the same can be set-off against income from other sources (consisting of
interest on fixed deposit).
162 INCOME TAX

MCQ

1. Income under the Income-tax Act, 1961, is to be computed under –


(a) Five heads
(b) Six heads
(c) Four heads
(d) Seven heads

2. What is the basic exemption limit for a woman assessee for A.Y. 2021-22, who
turned 60 years on 2.4.2021?
(a) ` 2,00,000
(b) ` 3,00,000
(c) ` 2,50,000
(d) ` 5,00,000

3. What is the rate of surcharge applicable to individuals having total income exceeding
` 1 crore?
(a) 15%
(b) 12%
(c) 10%
(d) 2%

4. What is the basic exemption limit for Mrs. X, a resident individual who is of the age
of 80 years as on 30.3.2021?
(a) ` 5,00,000
(b) ` 2,40,000
(c) ` 3,00,000
(d) ` 2,50,000

5. Share of profit of Mr. P, who is a partner is M/s PQR is -


(a) Exempt from tax
(b) Taxable as his business income
(c) Taxable as his salary
(d) Taxable as other sources

6. What is the basic exemption limit for Mr. X, a resident individual who is of the age of
60 years as on 1.4.2021?
(a) ` 5,00,000
TOTAL INCOME 163

(b) ` 2,40,000
(c) ` 3,00,000
(d) ` 2,50,000

7. The maximum amount of rebate allowable under section 87A for A.Y. 2021-22 is –
(a) ` 12,500, if the total income does not exceed ` 5 lakh
(b) ` 5,000, if the total income does not exceed ` 5 lakh
(c) ` 2,500, if the total income does not exceed ` 3.5 lakh
(d) ` 5,000, if the total income does not exceed ` 3.5 lakh

8. If Mr. Y’s total income for A.Y. 2021-22 is ` 52 Lakhs, surcharge is payable at the rate
of –
(a) 15%
(b) 12%
(c) 10%
(d) 2%

9. Unexhausted basic exemption limit of a resident individual can be adjusted against-


(a) Only LTCG taxable @ 20% u/s 112
(b) Only STCG taxable @ 15% u/s 111A
(c) Both (a) and (b)
(d) Casual income taxable @ 30% u/s 115BB

10. Unexhausted basic exemption limit of a non resident individual can be adjusted
against –
(a) Only LTCG taxable @ 20% u/s 112
(b) Only STCG taxable @ 15% u/s 111A
(c) Both (a) and (b)
(d) Neither (a) nor (b)
7 RETURN OF INCOME

Section Content
Statutory filing of Return
 For Firm & Company mandatory
 For other if Taxable Income before deduction under chapter VI A, & capital
gain exemption u/s 54, 54B, 54EC, 54F, 54GA & 54GB > Maximum amount
not chargeable to tax
 Individual who deposited 1 Crore or more in current account.
 Individual who incurred expenditure of 2 lakh or more on foreign travel.
 Individual who incurred expenditure of 1 lakh or more an electricity
consumption.
 Any resident who is otherwise not required to furnish a return of Income,
will be required to furnish a return if he has any asset located outside India
including any financial interest in any entity, or has signing authority in any
account located outside India.
Due date of
submission of
139(1) Different Situations
return
1. Where the assessee is a company October 31
2. Where the assessee is a person other than a company October 31
2.1 In case where accounts of the assessee are required
to be audited under any law
2.2 Where the assessee is a partner in a firm whose
accounts are required to be audited under any law
3. Assesses required to file Transfer Pricing Report under
Sec. 92E pertaining to International Transaction November 30
4. In any other case July 31
Note: Where the last day for filing return is a day on which the office is closed,
the assessee can file the return on the next day afterwards on which the office is
open and, in such cases, the return will be considered to have been filed within
the specified time limit.
139(1A) Salaried employee to furnish return through their employer.
INCOME TAX 165

139(1B) Option to file return through computer readable media (CD, floppy).
139(1C) Exempted from filing Return
For reducing the compliance burden of small taxpayers, the Central Government
has been empowered to notify the class or classes of persons who will be
exempted from the requirement of filing of return of income, subject to
satisfying the prescribed conditions.
139(3) Loss Return
Loss cannot be carried forward if Return of Income not filed within due date u/s
139(1). Exception: Income from HP, Unabsorbed Depreciation, Agricultural
Income.
139(4) Belated Return
Upto the end of the relevant AY or before the completion of assessment
Whichever is earlier. Belated Return can also be revised.
139(5) Revised Return
Upto the end of the relevant AY or before the completion of assessment
whichever is earlier. A return may be revised any number of times.
139(6) Particulars to be furnished with the return
The prescribed form of the return shall, in certain specified cases, require the
assessee to furnish the particulars of-
(i) Income exempt from tax
(ii) Assets of the prescribed nature and value and belonging to him
(iii) His bank account and credit card held by him
(iv) Expenditure exceeding the prescribed limits incurred by him under
prescribed heads
(v) Such other outgoing as may be prescribed.
139(6A) Particulars to be furnished with return of income in the case of an assessee
engaged in business or profession
The prescribed form of the return shall, in the case of an assessee engaged in any
business or profession also require him to furnish –
(i) The report of any audit referred to in section 44AB.
(ii) The particulars of the location and style of the principal place where he
carries on the business or profession and all the branches thereof.
(iii) The names and addresses of his partners, if any, in such business or
profession.
(iv) If he is a member of an association or the body of individuals; and
(a) The names of the other members of the association or the body of
individuals; and
(b) The extent of the share of the assessee and the shares of all such
166 RETURN OF INCOME

partners or members, as the case may be, in the profits of the business
or profession.
139(9) Defective Return
Non-payment of self-assessment tax together with interest, if any, payable in
accordance with the provisions of Section 140A, by the date of furnishing the
return of income shall make the return of income a defective return. Department
shall intimate defect to assessee to rectify it within 15 days.
139A PAN
The following persons are required to obtain a permanent account number:
1. If income exceeds exemption limit or turnover exceeds or is likely to exceed
5,00,000. Application should be submitted to obtain the permanent
account number before May 31 of the assessment year.
2. Non-individual entities which enters into a financial transaction of an
amount aggregating to 2,50,000 or more in a financial year.
3. Non-individual entities with the natural persons, it is also proposed that the
managing director, director, partner, trustee, author, founder, karta or any
person competent to act on behalf of such entities shall also apply for PAN.
4. Charitable Trust
5. Person specified by the Central Government (such as importers and
exporters)
6. Every person who intends to enter into certain prescribed transactions.
Every person who has been allotted a PAN and who has linked his Aadhar with
PAN, may disclose his Aadhaar Number in lieu of a PAN.
139AA Every person who is eligible to obtain Aadhar Number is required to quote
Aadhar Number in:
(a) PAN Application Number
(b) Return of Income
Person can quote the Enrollment ID of Aadhar Application form in case he does
not possess the Aadhar Number.
139B Submission of Return through TRP
It provides as follows:
 For the purpose of enabling any specified class or classes of persons to
prepare and furnish returns of income through a Tax Return Preparer.
 The Scheme shall specify the manner in which the Tax Return Preparer shall
assist the persons furnishing the return of income, and shall also affix his
signature on such return.
 A tax Return Preparer may be an individual other than an employee of the
specified class or classes of persons.
INCOME TAX 167

 The above Scheme shall also provide the manner in which a TRP shall be
authorized, the educational qualifications, the training and other conditions
required to be fulfilled, the code of conduct for the Tax Return Preparer, the
duties and obligations of the Tax Return Preparer, the manner in which the
authorization may be withdrawn.
 Practicing CA and Advocate cannot act as TRP.
 Cannot submit return of income of an assessee whose books of accounts are
required to be audited u/s 44AB.

139C Annexure Less Return


The Central Board of Direct Taxes has notified the following forms:
New ITR Subject
Forms
ITR – 1 For an individual if his total income is upto 50 lakh and includes:
(a) Salary and family pension;
(b) Income from one house property (excluding losses);
(c) Income from other sources but does not include:
• Winnings from lottery;
• Winnings from horse races; and
• Loss under this head.
ITR – 2 For individuals and HUFs not having income from profit and gains
from business or profession
ITR – 3 For individual and HUF having Income from profit and gains of
business or profession
ITR – 4 Income computed on presumptive basis u/s 44AD, 44ADA & 44AE
ITR – 5 For Firms, LLP, AOPs, BOIs, artificial judicial persons,
co-operative society or local authority.
ITR – 6 For Companies
ITR – 7 For persons required to furnish return under section 139(4A)/
(4B)/ (4C)/ (4D)/ (4E)/ 4F) (i.e. trusts, political parties,
institutions, colleges etc.)

Note:
1. The assessee will have the following 3 options of filling return electronically
(a) E-filing with digital signature
(b) E-filing with electronic verification code (EVC)
(c) E- filing and submitting ITR-V to CPC
2. Return in ITR 1 cannot be filed by a person, who:
168 RETURN OF INCOME

(a) Is a resident person (other than not ordinarily resident in India), & has:
• Any asset (including financial interest) located outside India;
• Signing authority in any account located outside India;
 Earned any Income from source outside India.
(b) Has claimed any relief of tax under sections 90, 90A or 91;
(c) If assessee claims exemption in respect of any income exceeding
5,000 under sections 10, 10A, 10AA, etc.
3. Return in ITR 4 cannot be filed by a person, who:
(a) Is a resident Individual or a HUF (other than not ordinarily resident in
India) deriving income as referred to in Section 44AD, 44ADA or 44AE,
and has:
• Any asset (including financial interest) located outside India;
• Signing authority in any account located outside India;
(b) Has claimed any relief of tax under sections 90, 90A or 91;
(c) If assessee claims exemption in respect of any income exceeding
5,000 under sections 10, 10A, 10AA, etc

139D e-return
All the ITR Forms are to be filed electronically. However, where return is
furnished in ITR 1 or ITR – 4, the following person have an option to file return
in paper form:
(i) Super Senior citizen or
(ii) Individual or HUF having income upto 5 lakh and not claiming any
refund.

e-Audit Report
E-filing audit reports shall be mandatory in following cases:
 Audit report under sec. 44AB in respect of books of account;
 Audit report under sec. 92E in respect of international transaction; or
 Audit report under sec. 115JB in respect of MAT computation.
140 Who shall verify the return
Assessee Who should verify the return
Individual By the individual himself or where the individual
concerned is absent from India or mentally incapacitated
from attending to his affair and where for any other reason,
it is not possible for the Individual to sign the return, by any
person duly authorized by him in this behalf.

HUF By the karta, or where the karta is absent from India or is


INCOME TAX 169

mentally incapacitated from attending to his affairs, by any


other adult member (male/female) of the family.
Company By the managing director thereof, or where, for any
unavoidable reason, such managing director is not able to
sign, or where there is no managing director, by any director
of the company.

Firm By the managing partner thereof or where, for any


unavoidable reason, such managing partner is not able to
sign or where there is no managing partner, by any partner
thereof, not being a minor.

Limited By the designated partner thereof, or where for any


Liability unavoidable reason such designated partner is not able to
Partnership sign or where there is no designated partner, by any partner
thereof, not being a minor.

Local By the principal officer thereof.


Authority

Political By the chief executive officer of such party.


Party

Any other By any member of the association or the principal officer thereof.
association

Any other By that person or by some other person competent to act


person on his behalf.

143(3A) E-assessment Scheme


1. Removal of existing human interface in the assessment procedure.
2. Assessment to be handled by specific functional units on the basis of
automated allocation system.
3. Assessment unit will be anonymous.
4. All communication to be made exclusively in electronic mode.
5. No personal appearance of any assessee.
6. This scheme covers assessment u/s 143(3) & 144.
170 RETURN OF INCOME

Centralized Processing Centers


A Centralized Processing Centre (CPC) has been set up at Bangalore for processing all e-filed
income-tax returns as well as paper returns filed in the State of Karnataka.

Obligation to furnish Specified Financial Transaction Reporting [Sec. 285BA]


The SFTR shall be furnished by every person specified in respect of all transactions of the
nature and value specified in form 61A on or before May 31st of the following financial year.

S. Class of person Nature and value of transaction


No.
1. A banking company Cash deposits aggregating to 10 lakhs or more in a
year in any savings account of a person maintained
in that bank.
2. A banking company or any Payments made by any person against bills raised in
other company or institution respect of a credit card issued to that person,
issuing credit card. aggregating to 2 lakh or more in the year.
3. A trustee of a Mutual Fund. Receipt from any person of an amount of 2 lakh or
more for acquiring units of that fund.
4. A company or institution Receipt from any person of an amount of 5 lakh or
issuing bonds or debentures. more for acquiring bonds or debentures issued by
the company or institution.
5. A company issuing shares Receipt from any person of an amount of 1 lakh or
through a public or rights more for acquiring shares issued by the company.
issue.
6. Registrar or Sub-Registrar Purchase or sale by any person of immovable
appointed under Registration property valued at 30 lakh or more.
Act.
7. A person being an officer of Receipt from any person of an amount or amounts
the RBI or a person aggregating to 5 lakh or more in a year for bonds
authorized by RBI. issued by the Reserve Bank of India.

When a person becomes liable to pay advance tax


Every person is liable to pay advance tax if advance tax payable is 10,000 or more. All
items of income are liable for payment of advance tax. The scheme of advance payment of
tax is also known as ‘pay-as-you-earn’.
An individual, who is of the age of 60 years or more at any time during the previous year
and who does not have any income chargeable under the head ‘Profits and gains of business
or profession’ is not required to pay advance tax. [Sec. 207]
INCOME TAX 171

How is advance tax payable


On or before June 15 of the PY Up to 15% of advance tax payable
On or before September 15 of the PY Up to 45% of advance tax payable
On or before December 15 of the PY Up to 75% of advance tax payable
On or before March 15 of the PY Up to 100% of advance tax payable
 Any payment of advance tax made before March 31 shall be treated as advance tax paid
during the financial year.
 If the last day for payment of any instalment of advance tax is a day on which the
receiving bank is closed, the assessee can make the payment on the next immediately
following working day, and in such case, the mandatory interest leviable under sections
234B and 234C would not be charged.
 Payment of advance tax in one installment on or before 15th March instead of Four for
assesses availing presumptive taxation u/s 44AD & 44ADA.

Interest for late filing of return [Sec. 234A]


An assessee shall be liable to pay interest u/s 234A @ 1% p.m. or part of the month for late
filing of return. Interest is calculated on the amount of tax due i.e. tax assessed on the total
income less advance tax, TDS if any:
Default Period for computing interest
If the return of income is From the date immediately following the due date for filing
furnished after the due return of income to the date of furnishing of return of
date income.
If the return of income is From the date immediately following the due date for filing
not filed return of income to the date of completion of assessment u/s
144.

Interest for non-payment of Advance Tax [Sec. 234B]


1. Interest u/s 234B is attracted for non-payment of advance tax or payment of advance
tax of an amount less than 90% of assessed tax (i.e. Tax on total income less TDS).
2. The interest liability would be 1% per month or part of the month from 1st April
following the financial year up to the date of determination of income i.e. self
assessment u/s. 140A.
3. Such interest is calculated on the amount of difference between the assessed tax and
the advance tax paid.
4. Assessed tax is the tax calculated on total income less TDS& TCS.

Interest for deferment of different installment of Advance Tax [Sec. 234C]


1. Interest u/s 234C is attracted for deferment of installment of advance tax beyond the
due dates.
172 RETURN OF INCOME

2. The interest liability would be 1% per month, for a period of 3 months, for every
deferment.
3. However, for the last installment of 15th March, the interest liability would be 1% for
one month.
4. Such interest is calculated on the amount of difference between the Advance tax
payable up to that date and the actual tax paid.

Payment of Advance Tax in case of Casual income [Proviso to Section 234C]


1. Advance tax is payable by an assessee on his total income, which includes capital gains,
dividend exceeding 10 lac and casual income like income from lotteries, crossword
puzzles etc.
2. Generally it is not possible to estimate his capital gains, dividend and income from
lotteries etc.
3. Therefore it has been provided that if any such income arises after the due date for any
installment, then, the entire amount of tax payable on such capital gains or casual
income should be paid in the remaining installments of advance tax which are due. If
such incomes are earned after 15th March, the entire tax should be paid by 31st march of
the relevant financial year.
4. No interest liability would arise if the entire tax liability is so paid.

Interest on recovery of refund granted earlier [Sec. 234D]


Simple interest at the rate of ½ % for every month or part thereof on the excess amount of
refund granted under section 143(1) if on regular assessment, it is found to be excessive.
Interest is payable for the period starting from the date of refund to the date of regular
assessment.

Fee of delay in filing of Income Tax Returns [Sec. 234F]


1. 5,000 if return is filed after due date but before 31st December of Assessment Year.
2. 10,000 if return is filed after 31st December of assessment year.
3. 1,000 if total income does not exceed 5 lakh.
4. Such fee shall be payable along with amount of taxes payable by the assessee under
section 140A.

Prosecution for non-filing of Income Tax Return [Sec. 276CC]


Prosecution proceeding can be initiated in case of failure to file an income tax return where
tax payable by the assessee is 10,000 or more.

Illustration 1: The accounts of a firm are subject to tax audit under section 44AB. X, is a
working partner of the firm. He is, however, not entitled to receive any remuneration as per
the partnership deed. He files his return of income for the assessment year 2021-22 on
INCOME TAX 173

September 30, 2021. The Assessing Officer wants to charge interest under section 234A for
delay in filing of return. Is the Assessing Officer justified?

Solution: The due date of filing return of income where the assessee is a “working partner”
in a firm whose accounts are required to be audited under any law, is September 30.
Consequently, the Assessing Officer is not justified in charging interest under section 234A.

Illustration 2: Can an individual, who is not in India, sign the return of income from outside
India? Is there any other option?

Solution: As per section 140, return of income can be signed by an individual even if he is
absent from India. Hence, an individual can himself sign the return of income from a place
outside India. Alternatively, any person holding a valid power of attorney and duly
authorized by the individual can also sign the return of income.

Illustration 3: Explain with brief reasons whether the return of income can be revised
under section 139(5) of the Income-Tax Act, 1961 in the following cases:
(i) Defective or incomplete return filed under section 139(9).
(ii) Belated return filed under section 139(4).
(iii) Return already revised once under section 139(5).
(iv) Return of loss filed under section 139(3).

Solution: Any person who has furnished a return under section 139(1) or in pursuance of
notice issued under section 142(1) can file a revised return if he discovers any omission or
any wrong statement in the return filed earlier. Accordingly,
(i) A defective or incomplete return filed under section 139(9) cannot be revised.
However, the defect can be removed.
(ii) A belated return filed under section 139(4) can be revised.
(iii) A return revised earlier can be revised again as the first revised return replaces the
original return. Therefore, if the assessee discovers any omission or wrong statement
in such a revised return, he can furnish a second revised return within the prescribed
time i.e. within one year from the end of the relevant assessment year or before the
completion of assessment, whichever is earlier.
(iv) A return of loss filed under section 139(3) is deemed to be return filed under section
139(1), and therefore, can be revised under section 139(5).

Illustration 4: Tax liability of X (53 years), a resident individual, for the financial year
2020-21 is computed as 1,00,000. X has paid advance tax as follows-

June 12, 2020 8,000


174 RETURN OF INCOME

September 10, 2020 12,000


December 21, 2020 30,000
March 11, 2021 35,000

X intends to file his income-tax return with balance tax &interest payable. There was no TDS
from any of his income. Compute the tax and interest payable if-
1. Balance tax and interest are paid on July 21, 2021 and return is filed on same date.
2. Balance tax and interest are paid on January 4, 2022 and he files return on same date.
3. Balance tax and interest are paid on July 21, 2021, but he forgot to file return and
return is later filed on January 4, 2022.
Solution:
Computation of interest payable under section 234C for deferment of advance tax:
Due date for Amount Amount Difference Interest Interest
payment of which should actual paid chargeable 1% per
advance tax have been months.
paid

June 12, 2020 15,000 8,000 7,000 3 210


September 15, 2020 45,000 20,000 25,000 3 750
December 15, 2020 75,000 20,000 55,000 3 1,650
March 15, 2021 1,00,000 85,000 15,000 1 150
Interest u/s 234C 2,760

Computation of interest payable under section 234B for default in payment of


advance tax

Case 1 (on 15,000 for 4 months @ 1% per month) 600


Case 2 (on 15,000 for 10 months @ 1% per month) 1,500
Case 3 (on 15,000 for 4 months @ 1% per month) 600

Computation of interest payable under section 234A for default in filing return of
income:
Case 1: No interest under section 234A as return for the assessment year 2021-22 is filed
before the due date of filling return of income, i.e., July 31, 2021.
Case 2: There is delay in filing return of income by 6 months, consequently, interest
payable under section 234A is 900 (on 15,000 for 6 months @ 1% per month).
Case 3: Interest under section 234A is not applicable as balance tax and interest are paid
before the due date of filing return of income though the return is actually filed in
January 2021. The answer is summarized as under:
INCOME TAX 175

Case 1 Case 2 Case 3

Balance tax payable 15,000 15,000 15,000


Interest under section 234A Nil 900 Nil
Interest under section 234B 600 1,500 600
Interest under section 234C 2,760 2,760 2,760
Total tax and interest 18,360 20,160 18,360

Illustration 5: State with reasons, whether the following statements are true or false, with
regard to the provisions of the Income-tax Act, 1961:
(i) The Assessing Officer has the power, inter alia, to allot PAN to any person by whom
no tax is payable.
(ii) Where the Karta of a HUF is absent from India, the return of income can be signed by
any male member of the family.
Solution:
(i) True : Section 139A(2) provides that the Assessing Officer may, having regard to the
nature of transactions as may be prescribed, also allot a PAN to any other person,
whether any tax is payable by him or not, in the manner and in accordance with the
procedure as may be prescribed.
(ii) False :Section 140(b) provides that where the karta of a HUF is absent from India, the
return of income can be signed by any other adult member of the family; such
member can be a male or female member.

Illustration 6: Comment on the allow ability of the following claims made by the assessee:
Mrs. Hetal, an individual engaged in the business of Beauty Parlour, has got her books of
account for the Financial year ended on 31st March, 2021 audited under section 44AB. Her
total income for the assessment year 2021-22 is 2,85,000. She wants to furnish her return
of income for assessment year 2021-22 through a tax return preparer.
Solution: Section 139B provides a scheme for submission of return of income for any
assessment year through a tax return preparer. However, it is not applicable to persons
whose books of account are required to be audited under section 44AB. Therefore, Mrs.
Hetal cannot furnish her return of income for A.Y. 2021-22 through a tax return preparer.
176 RETURN OF INCOME

MCQ

1. Akash, who is 32 years old, has long-term capital gains of 25,000 which is exempt
under section 10(38) and deduction of 80,000 under section 80C. He has to file a
return of income for A.Y. 2021-22, only if his total income exceeds –
(a) 1,00,000
(b) 1,45,000
(c) 1,50,000
(d) 2,50,000

2. The due date for filing of a return of income for a company for Assessment Year
2021-22 is –
(a) 31st July, 2021
(b) 30th September, 2021
(c) 31st October, 2021
(d) 31st August, 2021

3. For filing returns of income in respect of various entities, the Income-tax Act, 1961
has prescribed –
(a) One due date
(b) Two due dates
(c) Three due dates
(d) Four due dates

4. The return of a company has to be verified by –


(a) The Managing Director or Director
(b) The General Manager
(c) The Secretary
(d) The Manager

5. An assessee can file a revised return of income at any time before the completion of
assessment or before expiry of the following period, whichever is earlier –
(a) One year from the end of the relevant assessment year
(b) Two years from the end of the relevant assessment year
(c) Six months from the end of the relevant assessment year
(d) End of the relevant assessment year
INCOME TAX 177

6. As per section 139(1), filing of returns is compulsory irrespective of whether profit


is earned or loss is incurred, in case of –
(a) Companies only
(b) Firms only
(c) Both companies and firms
(d) All assesses

7. Mr. X has a total income of 7 lakhs for A.Y. 2021-22. He files his return of income
for A.Y. 2021-22 on 13th January, 2022. He is liable to pay fee of –
(a) 1,000 under section 234F
(b) 5,000 under section 234F
(c) 10,000 under section 234F
(d) Not liable to pay any fee

8. Mr. Y has a total income of 4,50,000 for A.Y. 2021-22. He furnishes his return of
income for A.Y. 2021-22 on 2nd December, 2021. He is liable to pay fee of –
(a) 1,000 under section 234F
(b) 5,000 under section 234F
(c) 10,000 under section 234F
(d) Not liable to pay any fee

9. Mr. Z, a salaried individual, has a total income of 8 lakhs for A.Y. 2021-22. He
furnishes his return of income for A.Y. 2021-22 on 28th August, 2021. He is liable to
pay fee of –
(a) 1,000 under section 234F
(b) 5,000 under section 234F
(c) 10,000 under section 234F
(d) Not liable to pay any fee

10. The due date of filing of return for a company with a business loss of 1,30,000 for
A.Y. 2021-22 is-
(a) 31st July, 2021
(b) 30th September ,2021
(c) 31st October, 2021
(d) 31st August, 2021
8 TDS & TCS
Point of time when TDS has to be deducted at source
1. At the time of payment
In the following cases, tax is deductible at the time of payment –
• Salary [Sec. 192]
• Winnings from lottery/ crossword puzzle [Sec. 194B]
• Winnings from horse race [Sec. 194BB]
2. At the time of payment or at the time of giving credit to the recipient in the books
of the payer, whichever is earlier
In all the other cases, tax is deductible either at the time of payment or at the time of
passing credit entry or book entry in the books of the payer, whichever is earlier-

Person liable to deduct TDS


The person responsible for making payment (or passing book entry) shall deduct tax at
source under the aforesaid sections whether the payment is for business or personal
purposes. However, if
• Payer is an individual/ HUF carrying on business whose turnover is upto ` 1 crore and
upto ` 50 lakh if carrying on profession in the immediately preceding financial year,
TDS need not be deducted, except u/s 194 IA 194 IB & 194M
• Payer is an individual /HUF and payment/credit is of professional fees for personal
purposes of the payer [Sec. 194J] then TDS is not deductible irrespective of turnover in
the immediately preceeding FY.
• Payee is Government, TDS is not deductible.

TDS Rates
Section Nature of Payment Threshold limit to Rate of
deduct tax TDS
192 Salary Basic exemption limit Avg Rate of
Tax
192A Premature withdrawal from ` 50,000 p.a. 10%
employee’s Provident Fund
193 Zero Coupon Bond / Government -- --
Securities
8% GOI Saving (Taxable) Bond 2003 ` 10,000 p.a. 10%
7.75% GOI Saving (Taxable) Bond ` 10,000 p.a. 10%
2018
INCOME TAX 179

Interest on Securities (Listed/ ` 5,000 p.a. 10%


Unlisted)
194 Dividend on shares ` 5,000 p.a 10%
194 A Interest on Fixed Deposit & Recurring
Deposit from Banking Company / Co- ` 50,000 p.a. (for
operative Society engaged in banking/ Senior Citizen) 10%
deposit with Post office ` 40,000 p.a. (for other)

Interest other than from Banking


company ` 5,000 p.a. 10%

194 B Winning from lottery, Cross word


` 10,000 30%
puzzle or other game
194 BB Winning from horse race ` 10,000 30%
194 C Payment to contractor/ sub contractor ` 30,000 per single 1% (2% if
contract or payment
` 1,00,000 in made to
aggregate person
other than
Ind/HUF)
Payment to Transport contractor who -- Nil
is eligible to compute income u/s 44AE

5% (10% if
payment
made to
194 D Insurance Commission ` 15,000 p.a.
person
other than
Ind/HUF)
Sum paid under Life Insurance policies
194 DA ≥ ` 1 lac p.a. 5%
which are not exempted u/s 10(10).
194 G Commission on sale of Lottery ` 15,000 p.a. 5%
194 H Payment of commission or Brokerage ` 15,000 p.a. 5%
194 I Payment of Rent for land, building or
furniture or fitting whether or not ` 2,40,000 p.a. 10%
owned by payee.
Payment of Rent for use of Plant,
Machinery or equipment whether or ` 2,40,000 p.a. 2%
not owned by payee.
180 TDS & TCS

194IA Payment on transfer of immovable


` 50 lakh
Property (other than agricultural land) 1%

194IB Payment of Rent to a resident by an


Individual/ HUF not liable for Tax ` 50,000 p.m. 5%
Audit
194 J (a) Payment of Technical Services Fee ` 30,000 p.a. 2%
194 J(b) Payment of Professional/ Non-
` 30,000 p.a. 10%
compete fee
194 K Dividend on units of mutual fund ` 5,000 10%
194 LA Payment of compensation on
` 2,50,000
compulsory acquisition of immovable 10%
property (other than agricultural land)
194 M Payment to contractor or professional
by Individual or HUF not required to ` 50 lakh p.a. 5%
deduct TDS u/s 194C or 194J
194 N Cash withdrawal exceeding ` 1 crore 2% on the
in a Financial year from any account amount
` 1 crore p.a.
maintained in a bank or Post office. exceeding
` 1 crore
194 O Payment by E-Commerce Operator to
E-commerce participant. Not
applicable for E-commerce participant -- 1%
being individual or Huf having sale
through ECO upto ` 5 lakhs.
Note:
1. TDS rates reduced by 25% w.e.f 14.5.20.
2. Surcharge and Cess is not applicable for TDS. It is applicable only:
• When the recipient is a foreign company or non resident
• On payment of Salary to any individual
3. The Rate of TDS will be 20% in all cases, if PAN is not quoted by the deductee. However,
for the purpose of Sec. 194O if the E-commerce participant does not furnish PAN or
Aadhar, rate of TDS would be 5%.
4. TDS needs to be deducted if the amount is above the limits given.
5. Interest @ 1% p.m. or part of the month from the date TDS was deductible to the date
tax is deducted (i.e. on late deduction).
6. Interest @ 1.5% p.m. or part of the month from the date tax was deducted to the date
tax is actually paid (i.e. on late deposit).
INCOME TAX 181

7. Professional or Technical Services would include any fees, commission or


remuneration by whatever name called, paid or payable to a director of a company
other than Salary income. No threshold limit is provided. Tax will be deductible even if
the Payment / Credit during a FY is less than ` 30,000.
8. Sec. 194IA will apply to all types of purchasers including individuals and HUFs, whether
they are required to get their books of account audited u/s 44AB or not. The buyer of
an immovable property is not required to obtain and quote TAN. The option of
obtaining certificate from the Assessing Officer under section 197 prescribing nil rate
or lower rate of TDS is not available in such a case.
9. For the purpose of Sec. 194IA if the value adopted/assessed/assessable for the stamp
duty purposes is more than the actual consideration paid/ payable, then such value
shall be considered for the purpose of deduction of tax at source. The registering officer
shall not register the document unless the transferee furnishes the proof of deduction
and payment of such tax.
10. For the purpose of Sec. 194 IA, consideration for immovable property shall include all
charges paid towards club membership fee, car parking fee, electricity and water
facility fees, maintenance fee or any other charges of similar nature, which are
incidental to transfer of the immovable property.
11. For the purpose of Sec. 194 IB, deductor shall not be required to obtain TAN and they
shall deduct TDS only once in a previous year.
12. TDS can be deposited under section 194M without any requirement to obtain TAN.
13. Exemption from TDS of 5% being deducted from Commission payable to individual
insurance agents subject to their filling a self-declaration that their income is below
taxable limit.
14. TDS shall be deducted on amount paid or payable without including GST on services
component.

TDS Certificates
Certificate for TDS shall be given by deductor/ payer
Income Form No. Due date
Salary Form 16 Annually by 31 May of following FY
st

Other Income Form 16A Quarterly within 15 days of furnishing the statement of
TDS

Due Dates for deposit of TDS


1. If deductor/payer is other than Govt.
Deduction date Due dates
If amount deductible in April to Feb. 7th of next month
If amount deductible in March April 30th
182 TDS & TCS

2. If deductor/payer is Government
Amount deposited without challan On the same day of deduction
Amount deposited with challan On the 7th day of next month
Due dates for deposit of Statements of TDS
Quarter ending Due dates
June July 31st
September October 31st
December January 31st
March May 31st

Credit for tax deducted at source [Sec. 199]


The whole or any part of the income on which tax has been deducted at source is assessable
in the hands of a person other than the deductee, credit for the whole or any part of the tax
deducted at source, as the case may be, shall be given to the other person and not to the
deductee.

Fee and penalty for delay in furnishing of TDS Statement and penalty for incorrect
information in TDS Statement [Sec. 272A]
In order to provide effective deterrence against delay in furnishing of TDS statement, it is
provided that-
(i) To levy fee of ` 200 per day for late furnishing of TDS statement from the due date of
furnishing of TDS statement to the date of furnishing of TDS statement. However, the
total amount of fee shall not exceed the total amount of tax deductible during the
period for which the TDS statement is delayed, and
(ii) To provide that in addition to said fee, a penalty ranging from ` 10,000 to ` 1,00,000
shall also be levied for not furnishing TDS statement within the prescribed time.
In view of the levy of fee for late furnishing of TDS statement, it is also provided that no
penalty shall be levied for delay in furnishing of TDS Statement if the TDS statement is
furnished within one year of the prescribed due date after payment of tax deducted along
with applicable interest and fee.
In order to discourage the deductors to furnish incorrect information in TDS statement, it is
provided that a penalty ranging from ` 10,000 to ` 1,00,000 shall be levied for furnishing
incorrect information in the TDS statement.

TCS Rates
S. No. Nature of Goods/ Contract % of Purchase
Price
1. Alcoholic Liquor for human consumption 1%
2. Tendu leaves 5%
INCOME TAX 183

3. Timber or any other forest produce not being tendu leaves 2.5%
4. Scrap 1%
5. Parking lot, Toll Plaza, Mining and quarrying (shall not include
mining & quarrying of mineral oil i.e. petroleum and natural
2%
gas)
6. Sale of minerals being coal or lignite or iron ore if the same was
not purchased by buyer for personal consumption or for 1%
manufacturing, processing or producing article or things or
Generation of power
7. Sale of motor vehicle over ` 10 lakh 1%
8. Amount collected by authorized dealer of ` 7 lakh or more in a 5%
financial year for remittance out of India from a buyer under
Liberalized Remittance Scheme of RBI
9. Sale of overseas Tour Program Package 5%
10 Sale of goods exceeding ` 50 lakhs to a buyer in a year by a
seller whose turnover exceeds ` 10 crore in preceding financial 0.1%
year

Note:
1. For transactions without quoting PAN, TCS shall be collected at twice the rate or 5%
whichever is higher.
2. Surcharge and Cess is not applicable for TCS. Surcharge, & Cess applicable if the
purchaser is a foreign company & the amount subject to TCS exceeds ` 1 crore.
3. Every seller shall, at the time debiting of the amount to the account of the buyer, or at
the time of receipt whichever is earlier collect from the buyer a sum equal to the
following percentage of the purchase price as income tax. However in case of sale of
motor vehicle, TCS shall be collected at the time of receipt of consideration.

Illustration 1: ABC Ltd. makes the following payments to Mr. X, a contractor, for contract
work during the P.Y. 2020-21
` 25,000 on 1.5.2020
` 25,000 on 1.8.2020
` 30,000 on 1.12.2020
On 1.3.2021, a payment of ` 28,000 is due to Mr. X on account of a contract work.
Discuss whether ABC Ltd. is liable to deduct tax at source under section 194C from
payments made to Mr. X.
Solution: In this case, the individual contract payments made to Mr. X does not exceed
` 30,000. However, since the aggregate amount paid to Mr. X during the P.Y. 2020-21
exceeds ` 1,00,000 (on account of the last payment of ` 28,000, due on 1.3.2021, taking the
184 TDS & TCS

total from ` 80,000 to ` 1,08,000), the TDS provisions under section 194C would get
attracted. Tax has to be deducted @ 1% on the entire amount of 1,08,000 from the last
payment of ` 28,000 and the balance of ` 26,920 (i.e. ` 28,000 – 1,080) has to be paid to
Mr. X.

Illustration 2: If XYZ Ltd. makes a payment of ` 28,000 to Mr. Ganesh on 2.8.2020 towards
fees for professional services and another payment of ` 25,000 to him on the same date
towards fees for technical services. Determine the liability of TDS.
Solution: TDS under section 194J would not get attracted, since the limit of ` 30,000 is
applicable for fees for professional services and fees for technical services, separately.

Illustration 3: An amount of ` 40,000 was paid to Mr. X on 1.7.2020 towards fees for
professional services without deduction of tax at source. Subsequently, another payment of
` 50,000 was due to Mr. X on 28.2.2021 from which tax @ 10% (amounting to ` 9,000) on
the entire amount of ` 90,000 was deducted. However, this tax of ` 9,000 was deposited
only on 22.6.2021. Compute the interest chargeable under section 201(1A).
Solution: Interest under section 201(1A) would be computed as follows
1% on tax deductible but not deducted i.e., 1% on ` 4,000 for 8 months 320
1 ½ % on tax deducted but not deposited i.e. 1 ½ % on ` 9,000 for 4 months 540

Illustration 4: A TV channel pays ` 11,00,000 on July 1, 2020 as prize money to the winner
of a quiz programme “Who will be a Millionaire”. Is there any TDS liability?
Solution: The TV channel is required to deduct tax at the rate of 30% under section 194B.
Tax has to be deducted at the time of payment of ` 11,00,000. ` 3,30,000 (being 30%) shall
be deducted and paid to the Government. The recipient of prize money will get ` 7,70,000.

Illustration 5: Punjab National Bank pays ` 75,000 per month as rent to the Central
Government for a building in which one of its branches is situated. Is there any TDS
liability?
Solution: As per section 196, there is no requirement to deduct income-tax, at source if the
payee is the Government. Punjab National Bank is not liable to deduct tax while paying rent
to the Central Government.

Illustration 6: A television company pays ` 40,600 to a cameraman for shooting of a


documentary film. Is there any requirement of tax deduction?
Solution: As per section 194J, the television company shall deduct tax at source at the time
of making payment to the cameraman at the rate of 10%.

Illustration 7: X has been running a sole proprietary business whose accounts are audited
under section 44AB during the last financial year i.e. 2019-20. He pays a monthly rent of
` 85,000 for the office premises to Y, the owner of building and an individual. Besides, he
INCOME TAX 185

also pays service charges of ` 10,000 per month to Y towards the use of furniture, fixtures
and vacant land appurtenant thereto and GST at the rate of 18% of monthly rent. Is there
any obligation on X to deduct tax at source in respect of rent and GST payable during the
financial year 2020-21?
Solution: The payer of rent is X, an individual. In the case of payment of rent, tax is
deductible under section 194-I if the aggregate amount of payment is more than ` 1,80,000.
However, an individual/HUF is supposed to deducted tax at source only if in the
immediately preceding financial year books of account are to be audited under section
44AB. In this case, books of account of X of the financial year 2019-20 are audited.
Consequently, payment of rent in the financial year 2020-21 will be subject to tax deduction
under section 194-I. The amount of TDS will be ` 1,14,000 [i.e., 10% of (` 85,000 +
` 10,000) x 12]. TDS will not be deducted on GST.

Illustration 8: X is a sole proprietor. His annual turnover for the financial years 2019-20
and 2020-21 is ` 90,00,000 and ` 2,25,00,000 respectively. Every year he pays (in the
month of March) annual consultancy fees of ` 1,20,000 to a consultant. This amount is
before GST and after GST it comes to ` 1,41,600 (i.e., ` 1,20,000 + GST at the rate of 18% of
` 1,20,000). Find out TDS liability in respect of payment made during the financial years
2020-21 and 2021-22.
Solution: X is an individual. During the financial year 2019-20, audit of books of account is
not required (turnover being ` 90,00,000). Consequently, tax is not deductible by X in the
immediately following financial year (i.e., financial year 2020-21). However, in the financial
year 2020-21, X is required to get his books audited under section 44AB. Therefore, tax has
to be deducted under section 194J in the immediately following financial year 2021-22 as
follows-
Tax to be deducted at source at the rate of 10% of ` 1,20,000 = ` 12,000
Note- TDS is not deductible on GST component.

Illustration 9: A company operating a television channel makes payment of ` 5 lakh to a


former cricketer for making running commentary of a one-day cricket match. Should the TV
channel company deduct tax at source?
Solution: Tax is deducible under section 194J at the rate of 10%.

Illustration 10: X is a sole proprietor. His turnover is more than ` 2,20,00,000 since last 5
years. During the financial year 2020-21, he makes the following payments to a non-
banking company:
1. Interest on loan taken for the marriage of his daughter (amount of interest paid on
March 1, 2021: ` 5,00,000).
186 TDS & TCS

2. Interest on loan for business purposes (amount of interest paid on March 21, 2021:
` 6,00,000). Under section 194A, tax is deductible on interest other than interest on
securities. Discuss whether the aforesaid payments are covered by this provision.
Solution: X, the payer, is an individual. His books of account are required to be audited
under section 44AB. Consequently, tax is deductible under section 194A. Section 194A is
applicable in such a case regardless of the fact whether payment of interest pertains to a
personal loan or a business loan. Tax will be deductible on ` 5,00,000 as well as ` 6,00,000
at the rate of 10%.

Illustration 11: X is a sole proprietor. His annual turnover is more than ` 2,25,00,000 since
last 5 years. During the financial year 2020-21, he makes the following payments of rent –
1. Rent paid to A Ltd. for a residential property for his personal use (amount of rent paid
on March 1, 2021: ` 5,00,000).
2. Rent paid to B for taking a machinery on rent (amount of rent paid on March 21, 2021:
` 6,00,000). Under section 194-I, tax is deductible on rent payment. Discuss whether
the aforesaid payments are covered by this provision.
Solution: X, the payer, is an individual. His books of account are required to be audited
under section 44AB. Consequently, tax is deductible on payment/credit of rent under
section 194-I. Section 194-I is applicable in such a case regardless of the fact whether
payment/credit of rent is for personal purposes or business purposes. Tax will be
deductible on ` 5,00,000 as well as ` 6,00,000. TDS rate is 2% of rent for use of machinery,
plant or equipment. It is 10% of rent for use of land, building, furniture or fixture.

Illustration 12: X Ltd. makes the following payments during the financial year 2020-21
1. Payment to A, a resident transport contractor eligible to compute income u/s 44AE:
` 11,50,000 (PAN is intimated by A to X Ltd.).
2. Payment to B, a resident transport contractor eligible to compute income u/s 44AE:
` 1,00,000 (PAN is not intimated by B to X Ltd.).
3. Payment to C, a resident catering contractor: ` 21,50,000 (PAN is intimated by C to X
Ltd.).
4. Payment to D, a resident catering contractor: ` 2,00,000 (PAN is not intimated by D).
Determine the amount of tax deductible under section 194C in this case.
Solution: TDS will be deducted at the rate of 1% (if recipient is an individual/HUF) or 2%
(if recipient is any other person). If PAN of the contractor is not available, tax will be
deducted at the rate of 20%. However, in the case payment or credit to transport
contractors (i.e., the business of plying, hiring or leasing goods carriages), no tax is
deductible if the recipient contractor gives his PAN to the payer.
Amount of tax deductible under section 194C in this case shall be calculated as follows-
`
Payment to A (transport contractor eligible to compute income u/s 44AE
INCOME TAX 187

and PAN is intimated, no TDS) Nil


Payment to B (transport contractor eligible to compute income u/s 44AE
and PAN is not intimated, Tax will be deducted @ 20%) 20,000
Payment to C (catering contractor and PAN is intimated, tax will be
deducted @ 1%) 21,500
Payment to D (catering contractor and PAN is intimated, tax will be
deducted @ 20%) 40,000
Total 81,500
188 TDS & TCS

MCQ

1. Any person responsible for paying to a resident any sum exceeding ` 2.5 lakh towards
compensation for compulsory acquisition of his urban industrial land under any law has
to deduct income-tax at the rate of –
(a) 10%
(b) 15%
(c) 20%
(d) 25%

2. The rate of TDS on rental payments of plant, machinery or equipment is –


(a) 2%
(b) 5%
(c) 10%
(d) 1%

3. Mr. X, a resident Indian, wins ` 10,000 in a lottery. Which of the statement is true?
(a) Tax is deductible u/s 194B @ 30%
(b) Tax is deductible u/s 194B @ 30.9%
(c) No tax is deductible at source
(d) None of the above

4. Mr. X paid fees for professional services of ` 40,000 to Mr. Y, who is engaged only in the
business of operation of call centre, on 15.7.2020. Tax is to be deducted by Mr. X at the
rate of –
(a) 1%
(b) 2%
(c) 10%
(d) 20%

5. Mr. A, a salaried individual, pays rent of ` 51,000 per month to Mr. B from June, 2020.
Which of the statement is true?
(a) No tax is deductible at source since Mr. A is not liable to tax audit u/s 44AB.
(b) Tax is deductible at source every month @ 10% on rent paid to Mr. B.
(c) Tax is deductible at source every month @ 5% on rent paid to Mr. B.
(d) Tax is deductible at source @ 5% on annual rent from the rent paid for March 2021
.
ALTERNATE MINIMUM
9 TAX (AMT)
Alternate Minimum Tax (AMT) [Sec. 115JEE(1)]
Chapter XII-BA contains the special provisions for levy of alternate minimum tax in case of
persons other than a company. Any person other than a company, who has claimed deduction
under any section (other than section 80P) included in Chapter VI–A under the heading
“C” or under section 10AA or investment-linked deduction under section 35AD would be subject
to AMT.

The provisions of AMT would, however, not be applicable to an individual, HUF, AOPs, BOIs,
whether incorporated or not, or artificial juridical person, if the adjusted total income of such
person does not exceed ` 20 lakh [Sec. 115JEE(2)].

Accordingly, where the regular income-tax payable by an individual for a previous year
computed as per the provisions of the Income – tax Act, 1961 is less than the AMT payable for
such previous year, the adjusted total income shall be deemed to be the total income of the
person. Such person shall be liable to pay income-tax on the adjusted total income @ 15% [Sec.
115JC].

“Adjusted total income” would mean the total income before giving effect to Chapter XII-BA as
increased by
(i) The deductions claimed, if any, under section 10AA;
(ii) The deduction claimed under section 35AD, as reduced by the depreciation allowable
under section 32, as if no deduction under section 35AD was allowed in respect of the
asset for which such deduction is claimed; and
(iii) Deduction under any section included in Chapter VI-A under the heading C-Deductions
in respect of certain incomes.

Tax credit for AMT [Sec. 115JD]


Tax credit is the excess of AMT paid over the regular income-tax payable under the provisions of
the Income-tax Act, 1961 for the year. Such tax credit shall be carried forward and set-off
against income-tax payable in the later year to the extent of excess of regular income-tax
payable under the provisions of the Act over the AMT payable in that year. The balance tax
credit, if any, shall be carried forward to the next year for set-off in that year in a similar
manner.
190 ALTERNATE MINIMUM TAX (AMT)

AMT credit can be carried forward for set-off upto a maximum period of 15 assessment years
succeeding the assessment year in which the credit becomes allowable.

Tax Credit allowable even if Adjusted Total Income does not exceed ` 20 lakh in the year of
set-off [Sec. 115JEE(3)]
In case where the assessee has not claimed any deduction under section 10AA or section 35AD
or deduction under section 80JJAA, 80QQB & 80RRB in any previous year and the adjusted total
income of that year does not exceed ` 20 lakh, it would still be entitled to set-off his brought
forward AMT credit in that year.

Section 115JC(3)
Every person to whom this section applies shall obtain a report, in prescribed from, from an
accountant, certifying that the adjusted total income and the alternate minimum tax have been
computed in accordance with the provisions of this Chapter and furnish such report on or
before the due date of furnishing of return of income under section 139(1).

Section 115JD(1)
No interest shall be payable on tax credit allowed.

Summary: Applicability of AMT


Person Deductions u/s 10AA, 80H to 80RRB, 35AD
Not Claimed Claimed
Firm/Limited Liability Partnership Not Applicable Applicable
Individual/HUF/ AOP/BOI or artificial juridical Not Applicable Applicable
person whose adjusted total income exceeding
` 20 lakh (> 20 lakhs)

Question 1: From the following information of Raj Kaj LLP, compute tax liability.
Sr. No. Particulars `
(1) Income from business A under head PGBP (Eligible for 100% 12,20,300
deduction under section 80IC)
(2) Loss from business B (Set up on 10.04.2020) under head PGBP (5,18,000)
(After claiming deduction of ` 25 lac for building under section
35AD)
(3) Income from business C under the head PGBP 6,15,000
(4) Loss under the head “Income from other sources” (10,000)
(5) Contribution to Prime Minister National Relief Fund by cheque 10,000

You might also like